You are on page 1of 337

-- ~=- - -~-- .

)
1 -= - -=- , 2 =- - ~-- ,-=- , 3 =- - ---- , 4 -- - -=- , 5 -
=- _-~ ,-=- , 6 - - -- , 7 - -- , 8 - - -=- , 9 ;- -
-- , 10 ,~- - -~- )
_-' ,' ,- -- ~- ~--- _-~--- ~-( , =-=~- , ,'-
~-( -, -- - -= '==- . ~- -= _ - ~- J;
^V J> ^---V ^-=- - - _ - ,-V
~-= _=-- . -~- -- ---- ^V .


! "#$
. $ %
1) 25 year teacher have fear attack and worry before enter the class ( I forgot
all the scenario) what is the initial treatment:
a. Selective serotonin reutake inhibitor
b. !ricyclic deressant
c. "eta blocker
d. #####
e. ######
-This is a case of social phobia and the initial treatment is answer a
(SSRIs).
$$$$$$$$$$$$$$$$$$$$$$$$$$$$$$$$$$$$$$$
$$$$$$$$$$$$$$$$
2) !he best initial !!! for deression is:
a. SS%Is
b. !ricyclic deressant
c. &'( inhibitors
d. "eta blocker
e. ###########
-The correct answer is a (SSRIs).
$$$$$$$$$$$$$$$$$$$$$$$$$$$$$$$$$$$$$$$
$$$$$$$$$$$$$$$$
)) *sing the following classification :
%isk factor +ase (disease) ,on case total
-resent ' " '.b
'bsent + / +.d
!otal '.+ "./
%elative risk of those with risk factor to those without risk factor is:
a. A/A+B , C/C+D
b. '0'."
c. +0+./
-The correct answer is a.
Positive predictive value (PPV) : a0 (a.b)
Neative predictive value (NPV) ! d 0 (+.d)
Sensitivit" ! proportion of people with disease who have a positive test: a 0 (a.c)
Specificit" ! proportion of people without disease who have a neative test ! d0
(b.d)
Relative ris# ( %% ) ! incidence in e$pose 1 incidence in une$posed! ( a 0 (a.b) ) 0
(c0 (c.d) )
%ttributable ris# ! incidence in e$pose --- incidence in une$posed! ( a 0 (a.b) ) -- (c0
(c.d) )
&dd ratio ( (% ) ! &dds that a disease person is e$posed ' &dds that a
nondisease person is e$posed ! ad 0 bc
2) 53 year old &an resented to 4% with sudden headache5 blurred of vision5
and eye ain. !he diagnosis is:
a. 'cute glaucoma
b. 'cute con6unctivitis
c. +orneal ulcer
d. #####
-The correct answer is a.
$$$$$$$$$$$$$$$$$$$$$$$$$$$$$$$$$$$$$$$
$$$$$$$$$$$$$$$$
5) 7hich heart condition is tolerable during regnancy:
a. 4isenmenger syndrome
b. 'ortic stenosis
c. Severe mitral regurge
d. /ilated cardiomyoathy with 48 239
e. &itral stenosis and the mitral area is 1 cm (or mm).
#the correct answer is c.
$$$$$$$$$$$$$$$$$$$$$$$$$$$$$$$$$$$$$$$
$$$$$$$$$$$$$$$$
:) /iffuse abdominal ain ;in wave like< and vomiting. !he diagnosis is:
a. -ancreatitis
b. 'endicitis
c. "owel obstruction
d. +holelithiasis
The correct answer is c. (colic#" pain! is the pain that comes in waves
(wave-li#e) and it is associated with bowel obstruction* cholelithiasis*
nephrolithiasis)
$$$$$$$$$$$$$$$$$$$$$$$$$$$$$$$$$$$$$$$
$$$$$$$$$$$$$$$$
=) 7hich tye of contracetive is contraindicative in lactation:
a. (+-s
b. &ini ills (rogesterone#only birth control ills)
c. I*/
d. +ondom
e. /eo#-rovera (rogesterone#only birth control in6ection)
-The correct answer is a. because combined &+Ps decrease the ,uantit"
of mil#. Proesterone -onl" contraceptive are not believed to decrease
the ,uantit" of mil#.
>) ' long scenario about atient with olydisia ad olyuria. I don?t remember
the scenario but they mention osmolality in urine and serum5 measurement
of ,a.
I encourage you to read about central @S nehrogenic diabetes insiidus.
# A IS ,(! +(&-B4!4 .
(.iabetes Insipidus) ! failure to concentration of urine as a result of central or
nephroenic %./. Present with pol"urea* pol"depsia and h"ponatremia.
.ianosis central or nephroenic b" administration of .esmopressin acetate
(..%VP) * s"nthetic analo of %./* can be used to distinuish central from
nephroenic .I.
0- +entral .I ! ..%VP challene will decrease urine output and increase urine
osmalirit"
1- Nephroenic .I ! ..%VP chane will not sinificant decrease urine output
$$$$$$$$$$$$$$$$$$$$$$$$$$$$$$$$$$$$$$$
$$$$$$$$$$$$$$$$
C) ' 5 year old child came with earache on eDamination there is fluid in middle
ear and adenoid hyertrohy. "eside adenoidectomy on management5
which also you should do:
a. &yringotomy
b. Erommet tube insertion
c. &astidectomy
d. !onsillectomy
e. #####
#The correct answer is a.
# &yringotomy (is used for bulin acute otitis media)
#Erommet tube insertion (is used for recurrent acute otitis media)
$$$$$$$$$$$$$$$$$$$$$$$$$$$$$$$$$$$$$$$
$$$$$$$$$$$$$$$$
13) Fow the randomiGed control study become strong or of good validity:
( sorry I forgot the answers)
2 IS N&T +&3P45T5
-6" doin blindin and allocation concealment. The best wa" to increase the
power of the stud" is to increase the sample si7e.
$$$$$$$$$$$$$$$$$$$$$$$$$$$$$$$$$$$$$$$
$$$$$$$$$$$$$$$$
11) 7oman with ostartum deression5 beside the medical !!!5 whch should
be include in theray:
a. 8amily theray or suort
b. #######
-Nonpharmacoloical treatment strateies are!conitive behavioral
therap"*ps"cho educational or support roup*increase partner
support*additional help with child care.
12) +lassic Scenario of stroke on diabetic and hyertensive atient. 7hat is
the athoysiology of stroke:
a. 'therosclerosis
b. 'nyresm
c. #####
-The correct answer is a.
$$$$$$$$$$$$$$$$$$$$$$$$$$$$$$$$$$$$$$$
$$$$$$$$$$$$$$$$
1)) &iddle aged atient with an acyanotic congenital heart disease the H#ray
show ventrical enlargement and ulmonary hyertension:
a. @S/
b. 'S/
c. !rancus arteriosus
d. -ulmonary stenosis
-The correct answer is b* bcause the pt. in middle ae and ac"notic.
$$$$$$$$$$$$$$$$$$$$$$$$$$$$$$$$$$$$$$$
$$$$$$$$$$$$$$$$
12) "est food in travelling is:
a. "oiling water
b. 7ater
c. Ice
d. -artial cocked fish and meat
e. ###
-The correct answer is a.
$$$$$$$$$$$$$$$$$$$$$$$$$$$$$$$$$$$$$$$
$$$$$$$$$$$$$$$$
15) &echanism of vitamin + in wound healing :
a. 4ithiliaGation
b. 'erobic fibroblast synthesis
c. +ollagen synthesis
d. 4nhance vasculariGation
e. ####
-The correct answer is c.
$$$$$$$$$$$$$$$$$$$$$$$$$$$$$$$$$$$$$$$
$$$$$$$$$$$$$$$$
1:) (en globe in6ury . !!! is:
a. +ontineuos antibiotic dros
b. +ontinuous wate and ,S dros
c. +ontinuous steroids dros
d. Sterile cover and the refered
e. #####
-The correct answer is d.
1=) 'dolescent female counseling on fast food. 7hat you should give her:
a. +a . folic acid
b. @it + . folic acid
c. Iinc . folic acid
d. Iinc . @it +
-The correct answer is a.
$$$$$$$$$$$$$$$$$$$$$$$$$$$$$$$$$$$$$$$
$$$$$$$$$$$$$$$$
1>) 7hat is the !%*4 about backache with osteoorosis:
a. ,ormal D ray vertebra eDclude the diagnosis
b. Steroid is beneficial !!!
c. @it / defiency is the cause
d. 'nother !!! I don?t remember
-The correct answer is c * because vit. . deficienc" is cause osteomacia.
&steomalcia is considerin secondar" causes of osteoporosis. +hoice d
not mention ma" be more accurate answer.
$$$$$$$$$$$$$$$$$$$$$$$$$$$$$$$$$$$$$$$
$$$$$$$$$$$$$$$$
1C) 'dolescent female with eating disorder and osteoorosis
a. 7eight gain
b. @it /
c. "ishoshonates
d. ########
#The correct answer is a.
$$$$$$$$$$$$$$$$$$$$$$$$$$$$$$$$$$$$$$$
$$$$$$$$$$$$$$$$
23) 8ourth degree hemorrhoids5 &anagement is:
a. Femoridectomy
b. band ligation
c. sclerotheray
d. fiber diet
-The correct answer is a.
# +lassification of internal hemorrhoid :
8irst degree: hemorrhoid do not prolapsed
Second degree! hemorrhoid prolapsed upon defecation but spontaneousl"
reduce
!hird degree! hemorrhoid prolapsed upon defecation* but must be manuall"
reduce
8ourth degree! hemorrhoids are prolapsed and cannot be manuall" reduce
-treatment:
8irst J Second: life st"le modification (fiber dite)
!hird: life st"le modification with (band liation* sclerotherap" or
cr"otherap") * if
8ailed o to surer".
8ourth: surer" (hemorrhiodectom")
21) nulligravida at > weak gestational age5 follow u for genetic screening5 she
refused the invasive rocedure but she agree for once screening 5 what is
the aroriate action now:
a. do ultrasound
b. 1
st
screening
c. 2
nd
screening
d. )
rd
screening
e. 'mncentesis
-The answer is correct 6
$$$$$$$$$$$$$$$$$$$$$$$$$$$$$$$$$$$$$$$
$$$$$$$$$$$$$$$$
22) =3 year old male with chronic Featitis " virus antigen carrier. !he
screening of choice is:
a. 'lfarotien . liver ultrasound
b. 'lfarotien . another tumor marker
c. 'bdominal +! . abdominal ultrasound
d. ####
-The correct answer is a.
$$$$$$$$$$$$$$$$$$$$$$$$$$$$$$$$$$$$$$$
$$$$$$$$$$$$$$$$
2)) )5 year old smoker 5 on eDamination sown white atch on the tongue5
management:
a. 'ntibiotics
b. ,o ttt
c. +lose observation
d. #########
-This is a case of leu#opla#ia and the manaement includes!as# the pt. to
stop smo#in* do a biops" for the lesion9 if there is pre-cancerous
chanes or cancer in the biops" 9 surical e$cision should be done.
$$$$$$$$$$$$$$$$$$$$$$$$$$$$$$$$$$$$$$$
$$$$$$$$$$$$$$$$
22) %egarding E/&:
a. Screening for E/& at 22 t3 2> weeks
b. /iet control is always successful !!!
c. Screening at > weeks
d. #########
-The correct answer is a.
$$$$$$$$$$$$$$$$$$$$$$$$$$$$$$$$$$$$$$$
$$$$$$$$$$$$$$$$
25) 'fter doing +-% on child and the showing asystole:
a. 'troine
b. 'drenaline
c. Bidocane
d. ######
-The correct answer is b.
2:) Scenario of trauma 5 on face eDamination there is shifted mouth angle5 loss
of sensation of ant. !hird of tongue5 which +, is affected:
a. 8acial nerve
b. !rigeminal nerve
c. ####
-The correct answer is b * facial nerve for taste ant. Third of tonue* but
trieminal nerve for sensation .
$$$$$$$$$$$$$$$$$$$$$$$$$$$$$$$$$$$$$$$
$$$$$$$$$$$$$$$$
2=) (n eDamination of newborne the skin show aules or (astules) over
erythema base:
a. transient neonatal ustular melanosis
b. erythema toDicum neonatorum
c. ####
#The correct answer is b.
$$$$$$$$$$$$$$$$$$$$$$$$$$$$$$$$$$$$$$$
$$$$$$$$$$$$$$$$
2>) !he most common causes of recocious uberty:
a. Idioathic
b. 8unctional ovary cysts
c. (vary tumor
d. "rain tumor
e. 'denoma
-The correct answer is a .
$$$$$$$$$$$$$$$$$$$$$$$$$$$$$$$$$$$$$$$
$$$$$$$$$$$$$$$$
2C) &other worry about radiation from microwave if eDosed to her child. 7hat
you tell her:
a. ,ot all radiation are dangerous and microwave one of them
b. &icrowave is dangerous on children
c. &icrowave is dangerous on adult
d. ####
# In international stud" ! all microwave is danerous on children ( ver"
vaue 2 )

$$$$$$$$$$$$$$$$$$$$$$$$$$$$$$$$$$$$$$$
$$$$$$$$$$$$$$$$
)3) 4arlier sign of uberty in male is:
a. 'earance of ubic hair
b. Increase testicular siGe
c. Increase enis siGe
d. Increase rostate siGe
e. ###
-The correct answer is b.
)1) Auestion about nutritional marasmus on definition.(rotein energy
malnutrition).
&arasmus is a form of severe protein-ener" malnutrition characteri7ed
b" ener" deficienc". % child with marasmus loo#s emaciated. 6od"
weiht ma" be reduced to less than :;< of the averae weiht that
corresponds to the heiht. 3arasmus occurrence increases prior to ae 0*
whereas =awash occurrence increases after 0: months. It can be
distinuished from =awash in that =awasa#i is protein wastin with the
presence of edema. The pronosis is better than it is for =awash.
$$$$$$$$$$$$$$$$$$$$$$$$$$$$$$$$$$$$$$$
$$$$$$$$$$$$$$$$
)2) 7hat is the most risk of antihyertensive drugs on elderly atient:
a. Fyotension
b. Fyokalemia
c. +,S side effect
d. K
-The correct answer is a. because orthostatic h"potension will increase
the ris# of fall and fractures in elderl".
$$$$$$$$$$$$$$$$$$$$$$$$$$$$$$$$$$$$$$$
$$$$$$$$$$$$$$$$
))) 'symtomatic woman with trichomonas:
a. !reat if symtomatic
b. !reat if she is regnant
c. !reat her anyway
d. K
-The correct answer is c.
Sinle dose of metradina7ole for s"mptomatic and as"mptomatic pt. and
treatment partner (because it is se$ual transmitted disease). "ut* not
treatment of as"mptomatic trichomniasis in first trimester of prenanc".
(+ontraindication in prenanc")
$$$$$$$$$$$$$$$$$$$$$$$$$$$$$$$$$$$$$$$
$$$$$$$$$$$$$$$$
)2) 7hat is the risk of E/& on her life later:
a. /& tye 1
b. /& tye 2
c. Imaired fasting glucose
-The correct answer is b.
)5) 7hat is the ma6or sign that can tell you that atient have olycythemia vera
rather than secondary olysythemia:
a. Featomegaly
b. Slenomegaly
c. @enous engorgement
d. Fyertension
e. ####
-The correct answer is b.
in pol"c"themia vera there is clonal proliferation of a pluripotent marrow stem
cell. %lso* level of er"thropietine ma" be useful distinuish P+V* level are
low* from other causes of pol"c"themia. Secondar" pol"c"themia is
associated with e$cessive dirusis* severe astroenteritis and burn.
$$$$$$$$$$$$$$$$$$$$$$$$$$$$$$$$$$$$$$$
$$$$$$$$$$$$$$$$
):) 7hat is the deficient vitamin in infantile beri beri :
a. "1(thiamine)
b. +
c. 4
d. ,iacine
-The correct answer is a.
L @I!'&I,4S /48I+I4,+M :
@it. ' : niht blindness > dr" s#in
@it5 "1 (thiamine): 6eriberi (pol"neuritis* dilated cardioma"opath"* edema)
@it. "2 (riboflavin): anular stomatitis* cheilosis. +orneal vascuari7ation.
@it. ") (nacine) ! pellara ( .iarrhea* .ermatitis* .ementia )
@it. "5 (antothenate): dermatitis* enteritis* alopecia* adrenal insufficienc"
@it. ": (yridoDine): convulsion* h"perirritabilit"
@it "12 (cobalamin): macroc"tic mealoblastic anemia* neuroloic
s"mptoms
8(BI+ '+I/ : macroc"tic mealoblastic anemia without neuroloic
s"mptoms.
@it + : scurv" ( swollen ums* brusin* anemia* poor wound healin)
@it / : ric#ets in children * osteomalasia in adult
@it. 4 : increase frailit" of R6+
@it. N : neonatal hemorrhae* increase PT > PTT* normal 6T
$$$$$$$$$$$$$$$$$$$$$$$$$$$$$$$$$$$$$$$
$$$$$$$$$$$$$$$
)=) 1= year boy admit to involve in recurrent illegal drug in6ection 5 what the
screening test to do:
a. FI@
b. Featitis "
c. Featitis +
d. #####
-%ll of the above choices are correct.?
)>) 23 year old male found to have heatitis b surface antibodies :
a. -revious vaccination
b. -revious infection
c. 'ctive infection
d. ####
-The correct answer is a.
F"s'g ! indicate carrier state.
F"s'b ! indicate provide immunit" to /6V
F"c'g! associated with core of /6V
F"c'": durin widow period* /6c%b-I3 indicate recent disease

$$$$$$$$$$$$$$$$$$$$$$$$$$$$$$$$$$$$$$$
$$$$$$$$$$$$$$$$
)C) &ost common cause of secondary hyertension in female adolescent is:
a. +ushing syndrome
b. Fyerthyroidism
c. %enal disease
d. 4ssential F!,
e. -olycystic ovary disease
-The correct answer is c.
3ost common causes of secondar" /TN in "oun female is renal arter"
stenosis ( caused b" fibromuscular d"splasia )
23) clomihene citrate:
a. induce ovulation
b. ######
-!he correct answer is a.
$$$$$$$$$$$$$$$$$$$$$$$$$$$$$$$$$$$$$$$
$$$$$$$$$$$$$$$$
21) Scenario about erimenstrual deression syndrome OOOO
-remenestral syndrome : define as a s"mptoms comple$ of ph"sioloical
emotional s"mptoms sever enouh to interfere with ever"da" life and occur
c"clical durin luteal phase of menses
-remenestral dyshoric disorder : is a sever form of pf premenestral
s"ndrome characteri7ed b" severe recurrent depressive and an$it" s"mptoms
with premenestral ( luteal phase) onest that remit a few da"s after the start of
menses.

$$$$$$$$$$$$$$$$$$$$$$$$$$$$$$$$$$$$$$$
$$$$$$$$$$$$$$$$
22) +hild resent with stiffing neck5 fever5 headach. Mou susect meningitis
what is your initial !!! should be:
a. !obramycin
b. BevoflaDicine
c. -eneciline(amicillin)
d. /oDycycline
I remember there is no vancomycine5 ceftriaDone or
deDamethasone on the choices.
#!he correct answer is c.
'E4 +ausative organism !reatment
P 1 &(,!F @6S* 5 coli %mpicillin A cefota$ime or
entamicin
1#) &(,!FS S.pneumonia* /.infulen7a
3eninocci
Vancom"cin A cefota$ime or
cefria$one
) &(,!FS #
'/*B!
Pneumoccoci*
meninococci
Vancom"cin A cefota$ime or
ceftia$one
Q:3
M4'%Racoholism
Pneumococi*
meninococci
@ram -ve bacilli
%mpicillin A vancom"cin A
ceota$ime or ceftia$one
2)) ' atient have tender5 redness nodule on lacriminal duct site. "efore
referred him to ohthalmologist what you will do:
a. !oical steroid
b. !oical antibiotics
c. (ral antibiotics
d. ,othing
e. K
The correct answer is c.(this is a case of dacroc"stitis).
$$$$$$$$$$$$$$$$$$$$$$$$$$$$$$$$$$$$$$$
$$$$$$$$$$$$$$$$
22) 'bout /& in NS':
a. about P 13 9
b. &ost of the t of insulin deendant tye
c. female more affected with tye 2 /&
d. most of ,I//& are obese
-The correct answer is d.
$$$$$$$$$$$$$$$$$$$$$$$$$$$$$$$$$$$$$$$
$$$$$$$$$$$$$$$$
25) &a6or aim of -F+ in Saudi 'rabia :
a. !o rovide comrehensive maternal J child health
b. ############
-other aims are!communit" participation* immuni7ation* referral* chronic
disease manaement* prescribin* health education* manaement of
communicable disease and environmental health.
$$$$$$$$$$$$$$$$$$$$$$$$$$$$$$$$$$$$$$$
$$$$$$$$$$$$$$$$
2:) 1= y.o5she missed her second dose of varecila vaccine5the first one about 1
y ago what youSll do:
a. give her double dose vaccine
b. give her the second dose only
c. revaccinate from start
d. see if she has antibody and act accordingly
#The correct answer is b.
$$$$$$$$$$$$$$$$$$$$$$$$$$$$$$$$$$$$$$$
$$$$$$$$$$$$$$$$
2=) there is outbreak of difteria and tetanus in community 5 regaring to
regnant woman:
a. contraindication to give /! vaccine
b. if eDosed 5 terminate regnancy immediately
c. if eDosed 5 terminate after =2 hour
d. give /! vaccine anyway
-The correct answer is d.
2>)'ll of the following are live vaccine eDcet:
a. &&%
b. (ral lio
c. @aricella
d. Featitis " vaccine
e. "+E
-The correct answer is d.
$$$$$$$$$$$$$$$$$$$$$$$$$$$$$$$$$$$$$$$
$$$$$$$$$$$$$$$$
2C) -t with scoliosis5 you need to refer him to the ortho when the degree is:
a. 5
b. 13
c. 15
d. 23
-The correct answer is d.
$$$$$$$$$$$$$$$$$$$$$$$$$$$$$$$$$$$$$$$
$$$$$$$$$$$$$$$$
53) >= year old who brought by his daughter5 she said he is forgettable5
doing mess thing in room 5 do not maintain attension 5 neurological
eDamination and the investigation are normal
a. 'lGheimer disease
b. &ulti#Infarct /ementia
c. ###
-The correct answer is a .
$$$$$$$$$$$$$$$$$$$$$$$$$$$$$$$$$$$$$$$
$$$$$$$$$$$$$$$$
51) &echanical intestinal obstruction
a. ,asogastric tube decomression

$$$$$$$$$$$$$$$$$$$$$$$$$$$$$$$$$$$$$$$
$$$$$$$$$$$$$$$$
52) -atient with cancer. Mou want to break bad news5 which of the
following is the answer:
a. Inform his family
b. 8ind out how much the atient know about it
c. Bet social service inform him
d. /on?t tell him
-The correct answer is b.
5)) FI@ atient has hemorrhagic lesion in the mouth and aules in the
face. Skin biosy show sindle cells and vascular structures:
a. Naosi sarcoma
b. $$$$$$$.
Naosi sarcoma : malinant multifocal neoplasm characteri7ed b" vascular
tumors of s#in and viscera * caused b" /uman herpesvirus : (//V:) * most
commom associated with %I.S patient. 4esion usall" papules . in biobs" show
spindle shape of cell.

$$$$$$$$$$$$$$$$$$$$$$$$$$$$$$$$$$$$$$$
$$$$$$$$$$$$$$$$
52) -atient with retrosternal chest ain 5 barium swallow show corkscrew
aearance
a. 'chalasia
b. 4sohagitis
c. E4%/
d. /iffuse esohageal sasm

-The correct answer is d.
# 'lchalasia : show esophaus dilation with (bird bea#) taperin of distal
esophaus
-/iffuse esohagus sasm! show cor#screw appearance.
$$$$$$$$$$$$$$$$$$$$$$$$$$$$$$$$$$$$$$$
$$$$$$$$$$$$$$$$
55) &ost common cause of intra cerebral hemorrhage:
a. rutured aneurysm
b. Fyertension
c. !rauma
d. K
-The correct answer is b.
5:) %t uer Tudrent ain and tenderness 5 fever5 high 7"+ 5 6aundice5
normal heatic marker
a. 'cute cholecyctitis
b. -ancreatitis
c. 'cute heatitis
d. K
-The correct answer is a.
$$$$$$$$$$$$$$$$$$$$$$$$$$$$$$$$$$$$$$$
$$$$$$$$$$$$$$$$
5=) 5 y.o child with h.o fever and swelling of the face ant to the both ears
(arotid gland enlargement) what is the most common comlication:
a. Babrynthitis
b. meningitis
c. encehalitis
d. orchitis.
-The correct answer is b.
&rchitis! (this one is the most common complication in postpubertal
males-adolscents and adults-). !his t. is reuberty .
$$$$$$$$$$$$$$$$$$$$$$$$$$$$$$$$$$$$$$$
$$$$$$$$$$$$$$$$
5>) classic symtoms of tension headache .
!ension headach : is the most common t"pe of headach dianosed in adult.
Present with tiht* band-li#e pain . ma" be enerali7ed or most intense in the
frontal* occipital* and nec# reion. Bsuall" occurs at the end of da". Treatmet
avoidance of e$acerbatin factors* NS%I. and acetaminophen.

$$$$$$$$$$$$$$$$$$$$$$$$$$$$$$$$$$$$$$$
$$$$$$$$$$$$$$$$
5C) rohylaDis of 'sthma
a. oral steroid
b. inhaler steroids
c. inhaler bronchodilator " agonists
-The correct answer is b.
$$$$$$$$$$$$$$$$$$$$$$$$$$$$$$$$$$$$$$$
$$$$$$$$$$$$$$$$
:3) )3 year woman with dysmenorrhea5 menorrhagea5 infertility5 and on
eDamination found immobile mass on uterosacral ligaments :
a. uterine fibroid
b. endometriosis
c. ###
-The correct answer is b
:1) +lassic symtoms of rimary dysmenorrheal
-rimary dysmenorrhea: menestral pain associated with ovulator" c"cle in
abcense of patholoical findin.
Secondary dysmenorrhea: 3enestral pain wich oranic causes e$ist* most
common cused b" endometriosis* adenom"osis* tumor and fibroid.
$$$$$$$$$$$$$$$$$$$$$$$$$$$$$$$$$$$$$$$
$$$$$$$$$$$$$$$$
:2) ,ewly married woman comlain of no regnancy for ) month with
unrotective seDual intercourse :
a. !ry more ( infertility is defined as no regnancy for one year)

$$$$$$$$$$$$$$$$$$$$$$$$$$$$$$$$$$$$$$$
$$$$$$$$$$$$$$$$
:)) Mounger diabetic atient came with abdominal ain5 vomiting and
ketones smelled from his mouth. 7hat is freTuent cause:
a. Insulin mismanagement
b. /iet mismanagement
c. ####
-The correct answer is a.. 6ut if the infection is one of the answers * "ou
should choose it because the most common cause of .=% is infection.
$$$$$$$$$$$$$$$$$$$$$$$$$$$$$$$$$$$$$$$
$$$$$$$$$$$$$$$$
:2) +ause of syncoe in aortic stenosis
a. Systemic hyotension
$$$$$$$$$$$$$$$$$$$$$$$$$$$$$$$$$$$$$$$
$$$$$$$$$$$$$$$
:5) (n stroke management: eDcet
a. I@ fluid not include deDtrose
b. /iaGeam for convulsion
c. ,o indication of anticonvulsive drugs
d. ####
-The correct answer is c.
$$$$$$$$$$$$$$$$$$$$$$$$$$$$$$$$$$$$$$$
$$$$$$$$$$$$$$$$
::) -atient use illegal drug abuse and the blood show %,' virus. 7hich
heatitis
a. '
b. "
c. +
d. /
-The correct answer is c.
:=) +lassic symtoms and signs of infectious mononucleosis (4"@) O
3ost common occur in "oun adult* usuall" cused b" 4"@ * transmission
throuh e$chane of bod" fluid include saliva. Present with triad ( fever*
sorethroat* l"mphadenopath") * also* ma" present tonsillar e$udates*
splenomeal"* maculopapular rash and bilateral upper e"elid edema.
3anaement is mostl" suortive and corticosteroids are indicated for
airwa" compromise due to tonsillar enlarement.

$$$$$$$$$$$$$$$$$$$$$$$$$$$$$$$$$$$$$$$
$$$$$$$$$$$$$$$$
:>) !reatment of 4"@ ( in scenario there atent with tonsiller eDudates5
lymhadenoathy5 slenomegaly) :
a. (ral acyclovir
b. (ral antibiotic
c. I& or I@ acyclovir
d. Suortive !!!
e. (bservation
-The correct answer is d.
$$$$$$$$$$$$$$$$$$$$$$$$$$$$$$$$$$$$$$$
$$$$$$$$$$$$$$$$
:C) 25 year old woman with weight loss5 heat intolerance5 irritable $.
a. Fyerthyroidism

$$$$$$$$$$$$$$$$$$$$$$$$$$$$$$$$$$$$$$$
$$$$$$$$$$$$$$$$
=3) -atient with coloured astules around his mouth5 organism show
heres simleD tye 15 what is the treatment:
a. (ral antiviral
b. Iv antiviral
c. Suortive
d. K
-The correct answer is a .
$$$$$$$$$$$$$$$$$$$$$$$$$$$$$$$$$$$$$$$
$$$$$$$$$$$$$$$$
=1) 'cute otitis media 5 initial !!!:
a. 'moDicilline
=2) &an with sudden onset of scrotal ain 5 also had FD of vomiting5 on
eDamination tender scrotom and there is tender 2 cm mass over right
groin5 what you will do:
a. +onsult surgeon
b. +onsult urologist
c. /o sonogram
d. 4lective surgery
-The correct answer is b * if e$pected tortion of tests * donCt wast time on
radioloical stud"
$$$$$$$$$$$$$$$$$$$$$$$$$$$$$$$$$$$$$$$
$$$$$$$$$$$$$$$$
=)) &ost common symtoms of renal cell carcinoma is
a. Fematuria
b. 'bdominal mass
c. 8lank ain
#The correct answer is a.
$$$$$$$$$$$$$$$$$$$$$$$$$$$$$$$$$$$$$$$
$$$$$$$$$$$$$$$$
=2) Bink the ttt with organism:
a. Shegella metronidaGole
b. Salmonella erythromycin
c. +ombylobacter amoDacilline
d. Eardia I foregut ( but I remember it is antibiotic)
-'ll of the above answers are wrong.O
$$$$$$$$$$$$$$$$$$$$$$$$$$$$$$$$$$$$$$$
$$$$$$$$$$$$$$$$
=5) &edical student had %!' systolic ressure is =3 mmhg5 what you will
do neDt in management:
a. I@ fluid theray
b. 4+E
c. 'bdominal *0S
d. ###
-The correct answer is a.
$$$$$$$$$$$$$$$$$$$$$$$$$$$$$$$$$$$$$$$
$$$$$$$$$$$$$$$$
=:) 23 year old male had been stabbed on midtrices 5 - -- -- _' =--
_- = - ~' .
(n microscoic eDamination of this greenish fluid show gram ositive
cocci in chains:
a. Stretococcal gangrene
b. +hlostrideal gangrene
c. 8ournier?s gangrene
d. meningocemia
e. K
-The correct answer is a.
==) -atient around his nose there are astules5 aules and telangiectasia
lesions. !he diagnosis is:
a. %osacea

$$$$$$$$$$$$$$$$$$$$$$$$$$$$$$$$$$$$$$$
$$$$$$$$$$$$$$$$
=>) +hild with radial head dislocation5 what is the neDt in management:
a. %eduction
b. D ray
c. &%I
d. K
-The correct answer is a.
$$$$$$$$$$$$$$$$$$$$$$$$$$$$$$$$$$$$$$$
$$$$$$$$$$$$$$$$
=C) In cervical B,s there are well differentiated thyroid cells5 during
oeration you find no lesion on thyroid what will you do neDt
a. !otal thyroidectomy
b. !otal thyoidectomy . radical cervical B,s dissection
c. !otal thyroidectomy . secific B,s dissection
d. !hyoid lobectomy with #####
-The correct answer is b.
$$$$$$$$$$$$$$$$$$$$$$$$$$$$$$$$$$$$$$$
$$$$$$$$$$$$$$$$
>3) Irritable bowel syndrome 5 ( they mention a secific mechanism< I do
not remember< that roduce which symtom
a. @omiting
b. /iarrhea
c. +onstiation
d. 'bdominal ain
OOOOOOOOOOOO A IS ,(! +B4'%
$$$$$$$$$$$$$$$$$$$$$$$$$$$$$$$$$$$$$$$
$$$$$$$$$$$$$$$$
>1) 8ree fluid accumulate in abdominal cavity cause:
a. Fyovolemic shock
b. +ardigenic shock
c. Sesis
d. 4mesis
e. ###
-The correct answer is c.
>2) 7oman with cyclic bilateral nodularity in her breast since : month5 on
eDamination there is ) cm tender mobile mass in her breast : what you
will do neDt
a. 8,' with cytology
b. &ammogram
c. "iosy
d. 8ollow u for neDt cycle
e. (bservation
-The correct answer is d. ( ICm not sure )
$$$$$$$$$$$$$$$$$$$$$$$$$$$$$$$$$$$$$$$
$$$$$$$$$$$$$$$$
>)) &ost common symtoms of soft tissue sarcoma :
a. -aralysis
b. (ngrowing mass
c. -ain
d. ###
-The correct answer is b.
(ngrowing mass: painless and slow-rowin.

$$$$$$$$$$$$$$$$$$$$$$$$$$$$$$$$$$$$$$$
$$$$$$$$$$$$$$$$
>2) the most common symtom in lacenta abrution is
a. @aginal bleeding
b. *terine tenderness
c. *terine contractions
d. 8etal distress
The correct answer is a
-lacental abrution is mainl" a clinical dianosis with all the above findins.
the most common s"mptom is dar# red vainal bleedin with pain durin the
third trimester of prenanc" (:;<) and abdominal or uterine tenderness
(D;<). 6leedin ma" occur at various times in prenanc" 6leedin in the first
trimester of prenanc" is ,uite common and ma" be due to the followin!
miscarriae (prenanc" loss) ectopic prenanc" (prenanc" in the fallopian
tube) . 6leedin in late prenanc" (after about 1; wee#s) ma" be due to the
followin! placenta previa or placental abruption.
>5) 8emale resented with vaginal discharge5 itching5 and on microscoe
showed mycoleous cells and sores. !his medical condition is most
likely to be associated with:
a. !"
b. /iabetes
c. %heumatoid 'rthritis
The correct answer is b
Vainal thrush is a common infection caused b" a "east called Candida
albicans. Vulvovainal candidiasis is usuall" secondar" to overrowth of normal
flora Candida species in the vaina. +onditions that interrupt the balance of
normal vainal flora include! antibiotic use* oral contraceptives* contraceptive
devices* hih estroen levels* and immunocompromised states such as diabetes
mellitus and /IV. Eomen are prone to vainal thrush between pubert" and the
menopause because* under the influence of the hormone estroen* the cells
linin the vaina produce a suar and "easts which Candida albicans are
attracted to. That is wh" thrush is rare before pubert".
$$$$$$$$$$$$$$$$$$$$$$$$$$$$$$$$$$$$$$$$
$$$$$$$$$$$$$$$
>:) -rimigravida in her >
th
week of gestation5 resented to your clinic
wanting to do genetic screening5 she declined invasive rocedure . the
best in this situation is
a. 'mniocentesis
b. 1
st
trimester screening
c. 2
nd
trimester screening
d. *ltrasound
The correct answer is b
$$$$$$$$$$$$$$$$$$$$$$$$$$$$$$$$$$$$$$$$
$$$$$$$$$$$$$$$
>=) !richomoniasis is classically have:
a. +lue cells
b. Ereenish frothy discharge
The correct answer is b
!richomoniasis is caused b" the flaellated proto7oan Trichomonas
vainalis9 itCs the most common curable se$uall" transmitted disease in the
world. It usuall" presents with froth" "ellow-reen vainal dischare* stron-
unpleasant odor* pain durin urination and se$ual intercourse.
$$$$$$$$$$$$$$$$$$$$$$$$$$$$$$$$$$$$$$$$
$$$$$$$$$$$$$$$
>=) (bsessive neurosis:
a. !reatment is east
b. +lomiramine doesn?t not work
c. &ostly associated with severe deression
d. +an be cured sontaneously
the correct aswer is c
>>) -atient came to you comlaining of hearing voices5 later he started to
comlain of thought get into his mind and can be taken out
a. S+I
b. &ood
c. &ania
d. 'gorahobia
The correct answer a
$$$$$$$$$$$$$$$$$$$$$$$$$$$$$$$$$$$$$$$$
$$$$$$$$$$$$$$$
>C) 8emale had history of severe deression5 many eisodes5 she got her
remission for three months with -aroDitine ( SS%Is) .. now she is
regnant .. your advise
a. Sto SS%iSs because it cause fetal malformation
b. Sto SS%iSs because it cause remature labor
c. +ontinue and monitor her deression
d. Sto SS%Is
The correct answer is c or a
In eneral* SSRI have the least side effects than other classes of
antidepressants. Some S5! Sleep disturbance* di77iness* se$ual
d"sfunction* Nervousness* and diarrhea
$$$$$$$$$$$$$$$$$$$$$$$$$$$$$$$$$$$$$$$$
$$$$$$$$$$$$$
C3) Fallucinations and -aranoia:
a. S+I
b. &ood
c. &ania
d. -hobia
The correct answer is a

$$$$$$$$$$$$$$$$$$$$$$$$$$$$$$$$$$$$$$$
$$$$$$$$$$$$$$$$
C1) 8emale resented with thirst and olyurea.. all medical history is
negative and she is not know to have medical issues.. .she gave history
of being diagnosed as "iolar and on Bithium but her +r and "*, is
normal. 7hat is the cause of her resentation
a. 'dverse affect of lithium
b. ,ehrogenic /I
c. +entral /I
The correct answer is a
,ehrogenic diabetes insiidus is characteri7ed b" a decrease in the
abilit" to concentrate urine due to a resistance to %./ action in the #idne".
Nephroenic diabetes insipidus can be observed in chronic renal
insufficienc"* lithium to$icit"* h"percalcemia* h"po#alemia.
C2) &ost common medical roblems encountered in -rimary care is:
a. +oryGa
b. *!I
c. F!,
The correct answer is a
$$$$$$$$$$$$$$$$$$$$$$$$$$$$$$$$$$$$$$$$
$$$$$$$$$$$$$$$
C)) %egarding diabetes mellitus in regnancy :
a. -revelance of diabetes mellitus in regnancy is 139
b. Elucose screeing is best in 22#2> week
c. /iabetic and non diabetic have same erinatal mortality
d. Eestational diabeted can be diagnosed by abnormal 8ES test
The correct answer is b

$$$$$$$$$$$$$$$$$$$$$$$$$$$$$$$$$$$$$$$
$$$$$$$$$$$$$$$$
C2) Beft red eye5 watery discharge5 hoto hobia5 eri#auricular non#tender
lymh nodes .. /D
a. "acterial con6ctvitis
b. @iral con6ctvitis
The correct answer is b

$$$$$$$$$$$$$$$$$$$$$$$$$$$$$$$$$$$$$$$
$$$$$$$$$$$$$$$$
C5) (ne of the 'nti#sychotics causes 4+E changes 5 Beukoenia5 drooling
:
a. %esiredone
b. +olGaine
c. 'misulride
The correct answer is b
$$$$$$$$$$$$$$$$$$$$$$$$$$$$$$$$$$$$$$$$
$$$$$$$$$$$$$$$
C:) (ne of the following decrease chance of colon cancer :
a. Iinc
b. @it. 4
c. @it +
d. 8olic acid
????????
6oth +* 5* 6eta carotene all are %ntio$idants* the" are correct but + is
more li#el". % bi dose of vitamin + fihts the bi F+.F some others!
8iber *Vitamin +* +alcium* Vitamin 5* Selenium. Ginc also .
C=) "est sentence to describe secifity of screening test5is the oulation
of eole who :
a. 're negative of disease5 and test is negative
b. 're ositive of disease5 and test is negative
c. 're ositive comaring to total other eole
d. ,egative disease 5 ositive test
e. -ositive disease 5 negative test
The correct answer is a
Secificity measures the proportion of neatives which are correctl"
identified (e.. the percentae of health" people who are correctl"
identified as not havin the condition).
Sensitivity (also recall rate) measures the proportion of actual positives
which are correctl" identified (e.. the percentae of sic# people correctl"
identified as havin the condition).
$$$$$$$$$$$$$$$$$$$$$$$$$$$$$$$$$$$$$$$$
$$$$$$$$$$$$$$$
C>) Feavy smoker came to you asking about other cancer5 not Bung cancer5
that smoking increase its risk:
a. +olon
b. "ladder
c. Biver
The correct answer is b
smo#in-associated cancers are lun* head >nec# (li#e esophaus and
lar"n$)* bladder and #idne"* pancreatic* cervical and stomach.

$$$$$$$$$$$$$$$$$$$$$$$$$$$$$$$$$$$$$$$
$$$$$$$$$$$$$$$$
CC) &id clavicle fracture :
a. Surgery is always indicated if fracture is dislaced
b. 8igure#>#dressing has better outcomes than simle sling
c. 8igure#>#dressing is strongly indicated in atient with un#union
risk
d. "oth figure#> and simle sling has similar outcomes
the correct answer is d
- simple slin has been to ive the same result as a fiure-: (more
comfort and fewer s#in problem).
$$$$$$$$$$$$$$$$$$$$$$$$$$$$$$$$$$$$$$$
$$$$$$$$$$$$$$$$
133) atient comlains of UcanSt breathe air in one nostril UV on
eDamination showed edematoud mucosa structure5 best to give initially
:
a. +orticosteroids
b. /econgestants
c. 'lfa#adrenergic blockers
The correct answer is a
131) -ediatric came to you in 4% with wheeGing5 dysnea5 muscle
contraction ( most robably asthma)5 best to give initially is :
a. theohillin
b. 'lbuterol neubeliGors
c. oral steroids
the correct answer is b
$$$$$$$$$$$$$$$$$$$$$$$$$$$$$$$$$$$$$$$$
$$$$$$$$$$$$$$
132) 8emale resented with comlain of neck ain and occiital
headache 5 no other symtoms 5 on H#ray has cervical sine
osteohyes and narrow disks :
a. cervical sonylosis O
OOOOOO

$$$$$$$$$$$$$$$$$$$$$$$$$$$$$$$$$$$$$$$
$$$$$$$$$$$$$$$$
13)) -atient comlaining of ain at night when he elevated his arm5
tingeling on lateral arm side and lateral three fingers 5 /D
a. brachial leDus neuroathy
b. shoulder imengment syndrom
c. brachial artery thrombohebitis
d. thoracic outlet roblem
the correct answer is d
"ranchial leDus neuroathy is characteri7ed b" acute onset of intense
pain in the shoulder or arm followed shortl" b" focal muscle wea#ness.
Imingement syndrome* swimmerSs shoulder* and throwerSs shoulder*
is a clinical s"ndrome which occurs when the tendons of the rotator cuff
muscles become irritated and inflamed. This can result in pain, weakness
and loss of movement at the shoulder. The pain is often worsened b"
shoulder overhead movement and ma" occur at niht* especiall" if the
patient is l"in on the affected shoulder.
!horacic outlet syndrome: numbness and tingling in the fingers9 pain in the
nec#* shoulder* and arm9 wea#ness of the arm and droppin thins from the
hand9 worsenin of the s"mptoms when elevatin the arm to do such thins
as comb or blow dr" oneCs hair or drive a car9 and coldness and color
chanes in the hand. The s"mptoms are often worse at niht or when usin
the arm for wor# or other activities.
132) Moung adult resented with ain on lateral elbow5 tingeling of lateral
arm5 he lays STuash:
a. carbel tunnel
b. tennis elbow
the correct answer is b
#Bateral eicondylitis( inflammation of common e$tensor tendon ) also
#nown as (tennis elbow* shooterCs elbow and archerCs elbow is a condition
where the outer part of the elbow becomes sore and tender. It is
commonl" associated with pla"in tennis and rac,uet sports.
-&edial eicondylitis (inflammation of common fle$or elbow) also #now
(olfer elbow)
$$$$$$$$$$$$$$$$$$$$$$$$$$$$$$$$$$$$$$$
$$$$$$$$$$$$$$$$
135) male came to you comlaining of sudden rogressive decreasing in
vision of left eye over last two0three days5 also ain on the same eye5
on fundoscoy otic disk swelling was sees 5 /D :
a.central retinal artery occlusion
b.central retinal vein occlusion
c.otic neuritis
d.macular degeneration
the" said the correct answer is c
%s# about s"mptoms of temoral arteritis in the older population. Patients
complain of sudden* painless* nonproressive vision loss in one e"e. /istor" of
headaches* Haw claudication* scalp tenderness* pro$imal muscle and Hoint aches*
anore$ia* weiht loss* or fever ma" be elicited.
Some patients ma" reveal a histor" of amaurosis fugaD involvin transient loss
of vision lastin seconds to minutes but which ma" last up to 1 hours. The vision
usuall" returns to baseline after an episode of amaurosis fua$.
+entral retinal 'rtery occlusion! presentin complaint is an acute sudden
painless unilateral loss of vision. Shows a classic cherr" red spot
+entral retinal vein occlusion! usuall" sudden painless variable visual loss9 the
fundus ma" show retinal hemorrhaes* dilated tortuous retinal veins* cotton-wool
spots* macular edema* and optic disc edema.
(tic neuritis! 3aHor s"mptoms are sudden loss of vision (partial or complete)*
or sudden blurred or Ffo" or washed outF vision* and pain on movement of the
affected e"e. 3an" patients with optic neuritis ma" lose some of their color vision
in the affected e"e (especiall" red). The optic dis# becomes swollen
&acular degeneration is a leadin cause of permanent painless irreversible
vision loss in the elderl".
%etinal /etachment9 S"mptoms are decreased peripheral or central vision* often
described as a curtain or dar# cloud comin across the field of vision. %ssociated
s"mptoms can include painless vision disturbances* includin flashin lihts and
e$cessive floaters.
13:) unilateral headache5 eDaggerated by eDcersice and light 5 /D :
a. migraine
b. cluster headach
c. stress headache
the correct answer is a
$$$$$$$$$$$$$$$$$$$$$$$$$$$$$$$$$$$$$$$
$$$$$$$$$$$$$$$$
13=) =3 years old with rogressive demntia 5 no ersonality changes 5
neurological eDamination was normal but there is visuodeficit 5 on
brain +! shower corteD atrohy and ventricular dilatations :
a. multi micro infract dementia
b. alGehimer demenita
c. arkinsonism dementia
the correct answer is b
alGehimer dementia :
most common cause of dementia. ae and famil" histor" are ris# factors for
%.. 5tiolo" un#nown but to$ic b-am"loid deposit in brain. Present with
amnesia for newl" ac,uired information is usuall" the first presentation*
followed b" lanuae deficit * acaluia* depression* aitation and finall"
apra$ia(inabilit" to perform s#illed movement). .ianosis b" e$clusion that
can be definitive dianosis onl" on autops"! suested b" clinical feature
and b" proressive conitive course without substantial motor impairment.
3RI > +T ma" show atroph" * venticule enlarement and can rule out other
causes. &n brain microscop" am"loid pla,ues and neurofibriall" tanle.
.eath usuall" occurrin secondar" to aspiration pneumonia . treatment b"
supportive therap" for Pt. and famil" * and cholinesterase inhibitor .
multi mico infarct dementia ( vascular demensia )
dementia associated with histor" of stro#. +riteria for vascular dementia
include presence of dementia and 1 or more of the followin!
0- focal neuroloical sins
1- s"mptoms onset that was abrupt * stepwise* or related to stro#
I- brain imain showin evidence of fold infarction or e$tensive deep white
matter chanes secondar" to chronic ischemia.

$$$$$$$$$$$$$$$$$$$$$$$$$$$$$$$$$$$$$$$
$$$$$$$$$$$$$$$$
13>) =3 years old with rogressive dementia 5 on brain microscoy
amyloid laTues and neurofibrillary tangles are clearly visible also
-laTues are seen : /D
a. lewy dementia
b. -arkisonism
c. 'lGehimer
The correct answer c
13C) after bite5 ediatric atient resented with abdominal ain and
vomiting 5 stool occult blood 5 rash over buttock and lower limbs 5
edema of hands and soles 5 urine function was normal but microscoic
hematurea was seen:
a. Byme
b. Fenoch#Schonlein -urura
The correct answer is b
/enoch-SchJnlein purpura (/SP) is a small-vessel vasculitis
characteri7ed b" purpura* arthritis* abdominal pain* and hematuria.
$$$$$$$$$$$$$$$$$$$$$$$$$$$$$$$$$$$$$$$
$$$$$$$$$$$$$$$$
113) for the above disorder (A 13C )5 which one is considered
athological
a. gross hematurea
b. microscoic hematurea
c. rashes
d. ..

The correct answer is b

$$$$$$$$$$$$$$$$$$$$$$$$$$$$$$$$$$$$$$$
$$$$$$$$$$$$$$$$
111) Moung adult resented with ainless enile ulcer rolled edges ..
what neDt to do :
a. +"+
b. /arkfeild microscoy
c. +ulturing
The correct answer is b
Syhilis also #nown as Freat imitatorF is a se$uall" transmitted disease
caused b" the spirochete bacteria Treponema pallidum. +lassicall"
presents as sinle painless non-itch" s#in ulceration with sharp borders. T
pallidum is too small to be seen under the liht microscope. So use
dar#field microscop" when sores are present. 6lood tests can confirm the
presence of antibodies. The antibodies remain in "our bod" for "ears* so
the test can be used to determine a current or past infection.
$$$$$$$$$$$$$$$$$$$$$$$$$$$$$$$$$$$$$$$
$$$$$$$$$$$$$$$$
112) /iabetic female ger 22h#urine roteinn is 153mg 55
a. start on '+4Is
b. refer to nehrologist
c . do nothing 5 this is normal range
the correct answer is a
11)) which revent or decrease incidence of getting ost heretic
neuralgia
a. 'mitritylin
b. 'cyclovir
c.varicella vaccination
d. valacyclovir
The correct answer is c
choose the best it will be varicella vaccination* if the rash start ive
valac"clovir it better than ac"clovir

$$$$$$$$$$$$$$$$$$$$$$$$$$$$$$$$$$$$$$$
$$$$$$$$$$$$$$$$
112) 'dult -olycystic kidney mode of inheritance:
a. 'utosomal dominant
$$$$$$$$$$$$$$$$$$$$$$$$$$$$$$$$$$$$$$$
$$$$$$$$$$$$$$$$
115) 8emale came with comlain of diahrrea in the last : months5 she lost
some weight5 she reorted that mostly was bloody 5 when you
reformed sigmoidoscoy you found fragile mucosa with bleeding 5/D
a. colon cancer
b. +hronSs
c. *lcerative colitis
d. Eastroenteritis
e. Femrrohids
The correct answer is c

$$$$$$$$$$$$$$$$$$$$$$$$$$$$$$$$$$$$$$$
$$$$$$$$$$$$$$$$
11:) 'nal fissure commonest site
a. -osteriorly
$$$$$$$$$$$$$$$$$$$$$$$$$$$$$$$$$$$$$$$
$$$$$$$$$$$$$$$$
11=) mother gave bitrh of baby with cleft li and alate5 she want to get
ergnant again what is the ercentage of recurrence
a. 19
b. 29
c. 159
The correct answer is b
11>) 2 years old girl resented with her arents to 4% with sore throat
and seroangious vaginal discharge with no ain what is most roable
cause5
a. +andida
b. 8oreign body
c. +hlamedia
d. Eonococci
e. stretococcus
The correcr answer is e
-ediatric vaginal discharge:
1 - infectious vulvovaginitis! present with malodorous * "ellow reen* most common
caused b" roup % streptococcus. ( ma" be present with se$ual abus FST.sF )
2#foreign body
)#candidal infection! ma" associated with diabetes K measure lucose.
2#Sarcoma botryoids (rhabdomyosarcoma ): malinant lesion appearance of
Fbunches of rapsF within vaina
.......................................
...............
11C) atient colaining of ain along median nerve ditribution 5 'nd
ositive tinel sign treatment include casting of both hand in what
osition
a. /orsifleDion
b. lantar fleDion
c. eDtention
d. /duction

The correct answer is c
$$$$$$$$$$$$$$$$$$$$$$$$$$$$$$$$$$$$$$$
$$$$$$$$$$$$$$$$
123) dermatomyosistis what is true
a. distal muscle weakness
b. *nderlying malignancy
c. EeneraliGed OOO Skin rash
The correct answer is c
-olymositis J dermatomyositis :
-olymyositis ! proressive * s"stemic tissue disease characteri7ed b" immune-
mediate striated muscle inflammation* present with s"mmetric proressive
pro$imal muscle wea#nes and pain .
/4%&'!(&M(SI!IS : present pol"m"ositis plus cutaneous involvement*
heiotro rash (violaceous periorbital rash) * shawl sign ( rash involvin the
shoulder* upper chest and bac# ) * EottronUs aule ( popular rash with
scale ).
-both ! increase serum += and anti-Lo antibodies . muscle biops" uscle fiber
and inflammation.
121) t with hyertrohic subaortic stenosis referred from dentist before
doing dental rocedure what is true
a. 53 9 risk of endocarditis
b. 12 9 risk of endo carditis
c. ,o need for rohylaDis
d. ost rocedure antibiotic is enough
the correct answer is c
$$$$$$$$$$$$$$$$$$$$$$$$$$$$$$$$$$$$$$$
$$$$$$$$$$$$$$$$
122) t want to Tuit smoking you till him that symtoms of nicotine
withdrawal aear after
a. 1#2 days
b. 2#2 days
c. 5#= days
d. ># 13 days
The correct answer is a
$$$$$$$$$$$$$$$$$$$$$$$$$$$$$$$$$$$$$$$
$$$$$$$$$$$$$$$$
12)) t taking buroion to Tuit smoking what is S4
a. 'rrythmia
b. SeiGure
c. Derostomia
d. Feadache
The correct answer is b
$$$$$$$$$$$$$$$$$$$$$$$$$$$$$$$$$$$$$$$
$$$$$$$$$$$$$$$$
122) 12 years old girl comlaining of ainless vaginal bleeding for 2#2
days every )7eeks to 2 months ranging from sotting to 2 acks er
dayV she had 2ry seDual ccc 1 year ago and had her menstruation since
: months on clinical eDamination she is normal seDual ccc5 normal
elvic eDam aroriate action
a. (+- can be used
b. Mou should ask for 8SF and rolactin level
OOOOOOOOOOOOOO
125) t want to do dental rocedure 5 he was dx to have mitral valve
prolapse clinically by cardiologist5 he had never done echo before what
is aroriate action
a. /o echo
b. ,o need for rohelaDis
c. give amicillin
d. Eive amoDicillin calvulinic
I thin# I will do 5cho* but this are some of the information I found so I cant decide.
This is from 3M& clinic
.octors used to recommend that some people with mitral valve prolapse ta#e
antibiotics before certain dental or medical procedures to prevent endocarditis*
but not an"more. %ccordin to the %merican /eart %ssociation* antibiotics are no
loner necessar" in most cases for someone with mitral valve reuritation or
mitral valve prolapse.
Still* if "ouCve been told to ta#e antibiotics before an" procedures in the past*
chec# with "our doctor to see how these new recommendations appl" to "ou.
This is from 3edicinNet
The vast maHorit" of patients with mitral valve prolapse have an e$cellent
pronosis and need no treatment. 8or these individuals* routine e$aminations
includin echocardiorams ever" few "ears ma" suffice. 3itral reuritation in
patients with mitral valve prolapse can lead to heart failure* heart enlarement*
and abnormal rh"thms. Therefore* mitral valve prolapse patients with mitral
reuritation are often evaluated annuall". Since valve infection* endocarditis* is
a rare* but potentiall" serious complication of mitral valve prolapse* patients with
mitral valve prolapse are usuall" iven antibiotics prior to an" procedure which
can introduce bacteria into the bloodstream. These procedures include routine
dental wor#* minor surer"* and procedures that can traumati7e bod" tissues
such as colonoscop"* "necoloic* or uroloic e$aminations. 5$amples of
antibiotics used include oral amo$icillin and er"throm"cin as well as
intramuscular or intravenous ampicillin* entam"cin* and vancom"cin.
$$$$$$$$$$$$$$$$$$$$$$$$$$$$$$$$$$$$$$$
$$$$$$$$$$$$$$$$
12:) ttt of cyclical mastalgia
a.(+-5 analgisc5 ,S'I/5 8at reduction5 and magnisuem
&astalagia : painful breast tissue that can be c"clic and usuall"
associated with hormonal chane* often bilateral . manaement ! stop
current hormonal therap" * reassurance* stop smo#in* fat reduction*
analesic* NS%I.* &+P.
12=) 2years old child what can he do
a. +oy sTuare and triangle
b. Seak in sentences
b. ..
The correct answer is a
$$$$$$$$$$$$$$$$$$$$$$$$$$$$$$$$$$$$$$$
$$$$$$$$$$$$$$$$
12>) baby can sit without suort5 walk by holding fourniture. -incer
gras5 ull to stand how old is he
a. > months
b. 13 months
c. 12 month
d. 1> month
The correct answer is b
$$$$$$$$$$$$$$$$$$$$$$$$$$$$$$$$$$$$$$$
$$$$$$$$$$$$$$$$
12C) reeated Tuestion about 'lGheimer disease
alGehimer dementia :
most common cause of dementia. ae and famil" histor" are ris# factors for
%.. 5tiolo" un#nown but to$ic b-am"loid deposit in brain. Present with
amnesia for newl" ac,uired information is usuall" the first presentation*
followed b" lanuae deficit * acaluia* depression* aitation and finall"
apra$ia(inabilit" to perform s#illed movement). .ianosis b" e$clusion that
can be definitive dianosis onl" on autops"! suested b" clinical feature
and b" proressive conitive course without substantial motor impairment.
3RI > +T ma" show atroph" * venticule enlarement and can rule out other
causes. &n brain microscop" am"loid pla,ues and neurofibriall" tanle.
.eath usuall" occurrin secondar" to aspiration pneumonia . treatment b"
supportive therap" for Pt. and famil" * and cholinesterase inhibitor .
$$$$$$$$$$$$$$$$$$$$$$$$$$$$$$$$$$$$$$$
$$$$$$$$$$$$$$$$
1)3) reeated A also about multiinfarct dementia
multi mico infarct dementia ( vascular demensia )
dementia associated with histor" of stro#. +riteria for vascular dementia
include presence of dementia and 1 or more of the followin!
0- focal neuroloical sins
1- s"mptoms onset that was abrupt * stepwise* or related to stro#
I- brain imain showin evidence of fold infarction or e$tensive deep white
matter chanes secondar" to chronic ischemia.
1)1) =) year atient comlain of rogressive loses of memory with
decrees in cognition function . +.! reveal enlarge ventricle and cortical
atrohy 5 diagnosis is
a# 'lGheimer
b#multi infarct dementia
c# multile sclerosis
d#OOOOOOOO
!he correct answer is a
'lGheimer (Dx by exclusion. Its associated with progressive memory loss,
decreased cognition function , & enlarged ventricles with cortical atrophy)
multi infarct dementia (N&T proressive > it has focal neuroloical
abnormalit")
multile sclerosis (recurrent relapsed > complete remission. Its
associated with dem"lenation of ra"-matter)
$$$$$$$$$$$$$$$$$$$$$$$$$$$$$$$$$$$$$$$
$$$$$$$$$$$$$$$$
1)2) :2 female with Kve a smear you should advice to reeat a
smear every:
'# :m
b# 12m
c# 1>m
d# no reeat
The correct answer is d
Screening a smear:
0- startin at ae 10 "ears or no more than I "ears after becomin se$uall"
active. 1- women N I; "ears who have
three consecutive normal test screenin ( 0 O I"easr). I-
screenin should be discontinue for women N P;-D; "ears who have had I or
more normal pap smear.
$$$$$$$$$$$$$$$$$$$$$$$$$$$$$$$$$$$$$$$
$$$$$$$$$$$$$$$$
1))) all following are criteria of chronic fatigue syndrome eDcet
b-???????
chronic fatigue syndrome:
characteri7es b" profound mental and ph"sical e$haustion. In associated
with multiple s"stem and neurosps"chiatric s"mptoms that last at least P
mounth. 3ust be new ( not life lon ) * must not be relived b" rest* and must
result in reater than Q;< reduction in previous activit". Presentation with R
or more of the followin ! poor memor" O concentration* m"alia* arthalaia*
sore throat* tender l"mph node* recent onset headach* unrefreshin sleep*
e$cessive tiredness with e$ercise. Treatment b" ! cognitive and excercise
therapy .also, diet, physiotherapy, dietary supplements, antidepressants
1)2) 12 year boy with sinsoneural hearing loose 5 +! scan show mass
(site in brainOOO) so diagnosis is
a# aTustic neuroma
b#meningioma
c# barotra
!he correct answer is a (ISm not sure )
# aTustic neroma (benin tumor of cranial nerve VIII. 3ostl" occur between ae of I;-
P; "ears. * 3ostl" unilateral e$cept if itCs associated with neurofibromatosis in which its
bilateral)
#meningioma (mostl" benin* with ae* more with female. Its occur in the
cerebellopontin)
# barotraumas (mostl" in the divers. The damae occur due to pressure)
..$$$$$$$$$$$$$$$$$$$$$$$$$$$$$$$$$$$$.
$$$$$$$$$$$$$$$$$$
1)5) 53 y with uncontrolled diabetes 5comlain of black to brown nasal
discharge. So diagnoses is
a# mycomyosis
b# asirglosis
c#foreign bo
The correct answer is a
# mycomyosis (funal infection caused b" 3"corales* affect nasal sinus >
luns* . KKKKKKK. characteri7ed b" blac# nasal dischare* .$ b"
biops").
$$$$$$$$$$$$$$$$$$$$$$.
$$$$$$$$$$$$$$$$$$$$$$$$$$$$$$$$
1):) 55 y comlain of dysnea5 -,/ with ast history of mitral valve
disease diagnosis is
a#B! side F8
b# %! side F8
c# nemothraD
d#-.4
The correct answer is a
$$$$$$$$$$$$$$$$$$$$$$$$$$$$$$$$$$$
$$$$$$$$$$$$$$$
1)=) clonidine is decrease effect of
a# benGotroin (anticholinirgic for -arkinson. ,ot affected by +lonidin)
b#levo doa (for -arkinson. +hanged in the brain to /oamen. +lonidin
the effect of Bevodoa through *nknown mechanism)
c#rubstinOOOOO

the correct answer is b
+lonodin is S1 aonist used to TTT h"pertension. S1 receptor in the brain
cause of both +&P > peripheral resistant .
1)>) 2 y baby with gray to green atch in lower back5 no redness or
hotness5 diagnosis is
a# child abuse
b#no ttt need
c# bleeding tendency
dOOOO
The correct answer is b
I thin# it is F 3onolian spot F * visible in P month and normall" disappear to
I-Q "ears. No need treatment.
$$$$$$$$$$$$$$$$$$$$$$$$$$$$$$$$$$$$$$$
$$$$$$$$$$$$$$$$
1)C) 15y boy with unilateral gyncomastia your advice is
a# my resolve sontiniously
b#there is variation from erson to erson
c#decrease use of soda oil or fish oil
d#OOOOOOOOOOO
the correct answer is a
- uni- or bilateral "necomastia occur normall" in newborn > at pubert"

$$$$$$$$$$$$$$$$$$$$$$$$$$$$$$$$$$$$$$$
$$$$$$$$$$$$$$$$
123) :m baby with mild viral diarrhea 5 ttt by (%S as
a-0;;mlO# for R hour then Q; mlO# Oda" after
b-Q;NNNNNNNNNNNNNNNN.Q;NNNNNNNNNN
c-0;;NNNNNNNNNNNNNNNN0;;NNNNNNNN
d-Q;NNNNNNNNNNNNNNNNN0;;NNNNNN
121) ): y female with breast mass mobile and change with menstrual
cycle 5 no skin dimle or fathering. Mour advice is
a#reeat eDam after 2 cycle
b#make biosy
c#fine needle asiration (there is ,( singe of breast cancer. It is
8ibroadenoma. Wust do 8,' to eDclude cancer J relive the t)
d#oral contracetion
The correct answer is a (ICm not sure )
$$$$$$$$$$$$$$$$$$$$$$$$$$$$$$$$$$$$$$$
$$$$$$$$$$$$$$$$
122) 53y female with breast cancer and +'125 elevate. So elevation due
to
a#breast cancer
b#associate with ovarian cancer
c#due to old age
d#normal variation
The correct answer is a * beause this pt. hih ris# of breast cancer.
# +'125 is a tumor mar#er mostl" used for ovarian +a* but itCs also used with
endometrial* fallopian* breast* > @IT +a. use manl" in ovarian cancer
$$$$$$$$$$$$$$$$$$$$$$$$$$$$$$$$$$$$$$$
$$$$$$$$$$$$$$$$
12)) 25y female with tachycardia and alitation. 4+E normal eDcet
F%1)3 and aical ulse is 213 .ast history of full ttt ovarian teratoma5
so your advice is
a# struma ovari should be consider
b#vagal stimulate should be done
c# referred to cardiology
!he correct answer is a
$$$$$$$$$$$$$$$$$$$$$$$$$$$$$$$$$$$$$$$
$$$$$$$$$$$$$$$$
122) t with alcohol drinked comlain of headache 5 dilated uil
hyeractivity5 agitation .he had history of alcohol withdrawal last week
so ttt is
a#diaGeam
b#naDtrol
c#haloeridol
the correct answer is a
#diaGeam (for sei7ure iven 0st occur in the 0st 01h).
#haloeridol (for hallucination iven 1nd occur in the 0st 1 da"s).
125) 53y man with chronic sychosis and not comlains for ttt .your
advice
a# deot haloeridol or floDtin
b#oral loraseam
c#oral busiron
d#OOOOOOO
The correct answer is a
$$$$$$$$$$$$$$$$$$$$$$$$$$$$$$$$$$$$$$$
$$$$$$$$$$$$$$$$
12:) 15y boy aear atch in rt lower leg these atch is clear center 5 red
in eriheral5 no fever no other comlain so diagnosis
a#contact dermatitis
b#tinea corora
c# lyme disease
d#OOOOOOO
The correct answer is b
-tinea corora: worm funal infection which transmitted b" contact s#in >
has clinical picture as in the ,uestion .
$$$$$$$$$$$$$$$$$$$$$$$$$$$$$$$$$$$$$$$
$$$$$$$$$$$$$$$$
12=) old t with of IF/ comlain for 2 mon of redness in lower leg and
ulse dim# inched in dorsalis edis these redness increase in
deendant osition and lim is cold and no swelling 5diagnosis is:
a#arterial inssuficncy
b#thrmbohibitis
c#cellulite
the correct answer is a
#thrombohilibitis! present with pain* swellin and redness .
#celluitis! present with redness* hotness and tender.
$$$$$$$$$$$$$$$$$$$$$$$$$$$$$$$$$$$$$$$
$$$$$$$$$$$$$$$$
12>) t with heart disease comlain of BB ischemia your advice
a#referred to cardiology
b#UUUUUUvascular surgery
c# start hearin

12C) female after seDual attack on eDam hymen tear in
a#2 oSclock
b#2UUUUUUU
c#:UUUUUU
d#>UUUUUUUU
the correct answer is c
-tear appear between Q and D oCcloc#
$$$$$$$$$$$$$$$$$$$$$$$$$$$$$$$$$$$$$$$
$$$$$$$$$$$$$$$$
153) =y boy comlain of limbing. +! show avascular necrosis in
eihysis of femur your advice
a# surgical ttt
b#slint for :m
c# hysiotheray
A not comlete ! but with these 3+2s * the correct answer is b
!his case is UerthSs diseasU ! vascular necrosis of femoral head. Q- 0; "ears .
usuall" self limitin with s"mptoms lastinT 0: months. Present with painless
limb * limited abduction and internal rotation.
Treatment! 0- observation if there is limited femoral head involvement or full
R&3 . 1- if e$tensive or if decrease R&3* consider bracin* hip abduction
with cast.
$$$$$$$$$$$$$$$$$$$$$$$$$$$$$$$$$$$$$$$
$$$$$$$$$$$$$$$$
151) t with trachoma in eye . for revention you should
a# water
b# UUUUUUUUUUUUUUUUU.eradication of organism
c# mass ttt
dOOOOOOOOOOO
the correct answer is a
#trachoma in the e"e is a bacterial infection caused b" +hlam"dia
trachomatis which is transmitted b" poor ha"en > contaminated /1&. TTT
b" antibiotic as er"throm"cin > .o$"c"cline. Surer" to prevent scar
$$$$$$$$$$$$$$$$$$$$$$$$$$$$$$$$$$$$$$$
$$$$$$$$$$$$$$$$
152) your advice to revent laTue disease is
a#hand washing
b#rodent eradication
c#sry insect side
d#OOOOOOOOOO
15)) t with severe headache and decrease in visual acuity 5uil is
dilated5 so ttt
a#ilocarin dro and ohthalmology referred
b#ergotamine
c#,SI/
dOOOOOOO
the correct answer is a
- this is closure angle glaucoma which characteri7ed b" sudden severe
headache* red e"e* visual acuit"* > dilated pupil. Pilocarpin is
paras"mpathomimmic which help in relive the pain
$$$$$$$$$$$$$$$$$$$$$$$$$$$$$$$$$$$$$$$
$$$$$$$$$$$$$$$$
152) main ttt of non inflammatory acne is
a#ritonic acid
b#clindmycinV
c#aGalic acid
d#erythromycin
the correct answer is a
#clindamycine J erythromycin are treatment of inflammator" acne.
. -aGalic acid ! treatment of non-inflammator" > inflammator" acne.
. K ritonic acid ! treatment of sever acne > non-inflammator"
$$$$$$$$$$$$$$$$$$$$$$$$$$$$$$$$$$$$$$$
$$$$$$$$$$$$$$$$
155) regnant with insulin deendant with good control5 so to decrease
risk of congenital disease
a#good metabolic control before regnancy
b#UUUUUUUUUUUUUUUUUUUUUUU1st trimester
c#UUUUUUUUUUUUUUUUUUUUUUUU2nd UUUUU
d#UUUUUUUUUUUUUUUUUUUUUUUU)rd UUUUUUU
the correct answer is a
$$$$$$$$$$$$$$$$$$$$$$$$$$$$$$$$$$$$$$$
$$$$$$$$$$$$$$$$
15:) female not married with normal investigation eDcet 8"SX122.
%"S1C:. so ttt
a#give insulin subcutaneous
b#advice not become married
c#barrier contracetive is good
d# "&I control
the correct answer is d
15=) diabetic t come to you with disturbance in conscious %"S : :5. so
main drug that cause hyoglycemia:
a#sulhnylurea
b#bugunid
c#acabos Y#glucosidase inhibitor
d#heniform
the correct answer a
#sulhnylurea ( insulin secretion* so* it can cause h"pol"cemia)
#bugunid ( insulin sensiti7ation* so* can not cause h"pol"cemia)
#acabos Y#glucosidase inhibitor ( used for postprandial
h"perl"cemia* so* can . . not cause h"pol"cemia)
#heniform ( insulin sensiti7ation* so* can not cause h"pol"cemia)
$$$$$$$$$$$$$$$$$$$$$$$$$$$$$$$$$$$$$$$
$$$$$$$$$$$$$$$$
15>) :m boy with fever you should give antiyretic to decrease risk of
a#febrile convulsion
b# eilesy
c# disseminate bacteri
The correct answer is a
$$$$$$$$$$$$$$$$$$$$$$$$$$$$$$$$$$$$$$$
$$$$$$$$$$$$$$$$
15C) icture in comuter aear vesicle 5 bulla and erythama in chest
skin so ttt
a# acyclovir cream
b#betamethGone cream
c#oralfamciclovir
d# erythromycin
the correct answeris b.

This case is Fheres GosterF . treatment of herpes 7oser are antiviral*
analesic. %ntiviral are ( s"stemic ) and include! ac"clovire* famciclovire*
ac"clovire. In multiple choice there is FfloclvirF and there is not dru have this
name. I thin# the writer 2 written in wron spellin. I thin# the choice c is
famciclovire and it is correct answer. 8amciclovir! Q;; m T.I.. for D da"s
1:3) t with scale in hair margin and nasal fold and behind ear with
aule and irregular erythema so ttt is
a-ni7oral cream
b- atovit
c- ac"clovir
d-antibiotic
the correct answer is a ( this is seborrheic dermatitis )
$$$$$$$$$$$$$$$$$$$$$$$$$$$$$$$$$$$$$$$
$$$$$$$$$$$$$$$$
1:1) 12y girl with athralgia and hotosensitivity and malar flush. 'nd
rotinurea so diagnosis is
a#%'
b#luus nehritis
c#*!I
d#OOOOOOOOOO
The correct answer is b
$$$$$$$$$$$$$$$$$$$$$$$$$$$$$$$$$$$$$$$
$$$$$$$$$$$$$$$$
1:2) Tuestion about diarrhea and Mesinia bacteria
Mersiniosis : it is infectious disease caused b" Mesinia bacteria. There are
I t"pes of Mesinia bacteria ! 1- M.entercolitis * 2- M.pseudotuberculosis* )-M.
pestis. M.enetercolitis cause blood" diarrhea* terminal ileitis and mesenteric
adenitis. .ianosis b" seroloical ! rise in antibod" titer . Treatment ! usuall"
self limitin and tetrac"cline for sever infection.
$$$$$$$$$$$$$$$$$$$$$$$$$$$$$$$$$$$$$$$
$$$$$$$$$$$$$$$$
1:)) aralegia t with ulcer in lower back 2.2 cm and lose of dermis
and eidermis these ulcer in stage
a# I
b#II
c#III
d#I@
the correct answer is b
-stage I ! non-blanchable redness that N&T subside after relive of the
pressure
#stage II ! damae to epidermis > dermis but N&T deeper
#stageIII ! subcutaneous tissue involvement
#stageI@ ! deeper than subcutaneous tissue as muscles > bones
1:2) -%4E,',! B'/M rim at labor ain 5 on eDa cervical in stage I of
labor so ain management is
a#morhine I&
b#eidural anesthesia
c#generalUUUUUUUUU
d#localUUUUUUU
The correct answer is b
$$$$$$$$$$$$$$$$$$$$$$$$$$$$$$$$$$$$$$$
$$$$$$$$$$$$$$$$
1:5) -sycatric t on antisychotic drug most drug that lead to imotence
with antisychotic is
a# rornlol
b#,S'I
c#'+4I
d#thaiGide
the correct answer is a
$$$$$$$$$$$$$$$$$$$$$$$$$$$$$$$$$$$$$$$
$$$$$$$$$$$$$$$$
1::) man resent with ainless ulcer in his enis with indurate base and
everted edge so diagnosis is
a# syhilis
b# gonorrhea
c# choncroid
d# FS@
The correct answer is a
# syhilis : painless ulcer in the penis
# gonorrhea: ST. b" bacterial infection cause pain > dischare but N&
ulcer
# choncroid: ST. b" bacterial infection cause pain > dischare >cause
painful ulcer
# FS@ : ST. b" virus that cause painful ulcer with dischare.
$$$$$$$$$$$$$$$$$$$$$$$$$$$$$$$$$$$$$$$
$$$$$$$$$$$$$$$$
1:=) man have long history of urethral stricture resent with tender right
testis and 7"+ in urine so diagnosis is
a#eddimorchitis
b# testicular torchin
c# varicosel
d#OOOOOO
the correct answer is a
-eddimorchitis : occur with BTI > urinar" retension)
# testicular torchin : sudden acute severe testicular pain with
elevated
. transversel" lin testes)
# varicosel : due to intra-abdomenal pressure.
1:>) man use saldinafil (@iagra)5 to revent hyotension you should not
use
a#nitrate
b#" blocker
c#'+I4
d#++"
the correct answer is a
1:C) female comlain of ainless odorless and colorless vaginal
discharge that aear after intercourse so ttt
a#give antibiotic
b#douche after intercourse
c# cervical cancer should be consider
d#may be due to chronic salingitis
the correct answer is b
+olor Smell
-ain (%
Itching
(ther
,ormal
vaginal
discharge
+lear &R mil#" &dorless N&
durin
ovulation* se$*
breastfeedin
"acterial
vaginosis
Ehite-ra" &R
"ellow
8ish" Mes durin se$
!richomoniasi
s
Eater" &r
"ellow
Mes especiall"
durin urination
+andida
Ehite-chees"
thic# stic#"
Mes especiall"
durin se$
$$$$$$$$$$$$$$$$$$$$$$$$$$$$$$$$$$$$$$$
$$$$$$$$$$$$$$$$
1=3) *!IQ12 day5 most robably cause ylonhritis
a-*;Q<
b-*Q<
c-Q<
d-Q;<
/ifficult A 5 !his A referral to urologist.
$$$$$$$$$$$$$$$$$$$$$$$$$$$$$$$$$$$$$$$
$$$$$$$$$$$$$$$$
1=1) atient comlain of right iliac fossa mass so diagnosis
a#diverticulitis
b#aendicitis
c#ancrtitis
d#chrons disease
- the correct answer is d
1=2) t with long history of *+ on endoscoes see oly and cancer
lesion on left colon so ttt
a#ttt of anemia
b#left hemicolctomy
c#total colctomy
d# remove oly
the correct answer is c
$$$$$$$$$$$$$$$$$$$$$$$$$$$$$$$$$$$$$$$
$$$$$$$$$$$$$$$$
1=)) female with hair on different site of body and refuse intake of food
and "&IP1> and feel as body is fat so diagnosis
a#anoreDia nervosa
b#bulimia nervosa
c#body dismorhic syndrome
d# anDiety
the correct answer is a
#anoreDia nervosa! decrease bod" wt. amenorrhea and lanuo (hair ).
#bulimia nervosa: normal or increase bod" wt. restrict eatin followin b"
overeatin then uilt.
$$$$$$$$$$$$$$$$$$$$$$$$$$$$$$$$$$$$$$$
$$$$$$$$$$$$$$$$
1=2) obese female5 insulin resistance and hairstisim so diagnosis
a#oly cystic ovary
b#hyerrolctinmia
c#familial
d#OUOOOOOOOO
the correct answer is a
$$$$$$$$$$$$$$$$$$$$$$$$$$$$$$$$$$$$$$$
$$$$$$$$$$$$$$$$
1=5) boy ) day after flue symtom develo con6unctivitis with occiital
and neck B., enlarged so diagnosis is
a#adenoviruses
b#stretococcus
c#FS@
/OOOOOOOOOO
the correct answer is a
$$$$$$$$$$$$$$$$$$$$$$$$$$$$$$$$$$$$$$$
$$$$$$$$$$$$$$$$
1=:) child with asthma use betamethaGone5 most common side effect is
a#increase intraocular ressure
b#eilesy
c#growth retardation
d#
. the correct answer is c
1==) curve of F"@ marker
answered in A )>
$$$$$$$$$$$$$$$$$$$$$$$$$$$$$$$$$$$$$$$
$$$$$$$$$$$$$$$$
1=>) sickling t after acute attack 5 discharge on
a# enicillin
b#iron
c#vitamin
dOOOOOOOOOOO
the correct answer is a
proph"lactic of pneumococcal infection b" vaccination and oral
penicillin
$$$$$$$$$$$$$$$$$$$$$$$$$$$$$$$$$$$$$$$
$$$$$$$$$$$$$$$$
1=C) :m with cough and wheeGy chest .diagnosis is
a# asthma
b# broncholitis
c#neumonia
d#8." asiration
The correct answer is b
# asthma : after 1 "ears old)
# broncholitis : before 1 "ears old)
#neumonia : associated with cr"pitation
#8." asiration : sudden whee7in
$$$$$$$$$$$$$$$$$$$$$$$$$$$$$$$$$$$$$$$
$$$$$$$$$$$$$$$$
1>3) 15y old with ilonidal sinuse so ttt
a#incision surgery
b# local antibiotic
c#daily clean
d#OOOOOOOOO
The correct answer is c
. ilonidal sinus : sinus tract witch commonl" contain hairs. Treatment *
firstl" conservative ttt in mild case remove all hair* washin cleanin . if not
relive! surer"

1>1) 8emale t > wks ostartum5not smoker diagnosed to have
asthma5her asthma was not controlled she attended 4% ) times last
month5on "2 agonist and oral steroid5she came c0o wheeGing and s.o.b
mildly cyanosed using her intercostal muscles5wheeGy
chest5"-:1:30133 -:123 -(2:=2 -48:):5there is oedema in her foot u
to the knee5the most likely diagnosis:
a. +(-/
b. ulmonary embolism
c. 'cute asthma attack
d. 'ngioedema
The correct answer is a or c
KKKKKKKKKKKKKKKKKKKKKKKKKKKKKKKKKKKKKKK
KKKKKKKKKKKKKKKK$
1>2) 8emale t develoed sudden loss of vision(both eyes) while she
was walking down the street5also c0o numbness and tingling in her
feet 5there is discreancy b0w the comlaint and the finding5
(04 refleDes and ankle 6erks reserved5there is decrease in the
sensation and weakness in the lower muscles not going with the
anatomy5what is your action:
a. +all ohthalmologist
b. +all neurologist
c. call sychiatrist
d. reassure her and ask her about the stressors
the correct answer is ! d
KKKKKKKKKKKKKKKKKKKKKKKKKKKKKKKKKKKK...........
..............................................
1>)) same scenario in A.1>2 what is the diagnosis:
a. +onversion disorder
b. Somatoform disorder
The correct answer is a
1 K somatiGation disorder! female before ae I; "ears . s"mptoms
include! . . two @IT * four site of pain * one se$ual d"sfunction * one
pseudoneuron . . 2# conversion disorder!
s"mptoms include voluntar" or sensor" . . )#
hyochondriasis: fear from life threatenin disease.
. 2# body dysmorhic disorder! aware from his imain.
. 5# somatoform ain disorder: intensit" pain is main s"mptoms

1>2) male t develoed corneal ulcer in his %t eye after trauma what is
the &D:
a. 'ntibiotic and cyclolagia is mydrasis and refer to ohthalmology
b. toical steroid
the correct answer is a
6ecause infection is a common occurrence in corneal ulcers* "our
ophthalmoloist will prescribe antibiotic e"edrops. If the infection appears
ver" lare* "ou ma" need to use these drops as often as one drop an hour.
-&ral pain medications will be prescribed to control the pain. Pain can also
be controlled with special e"edrops that #eep "our pupil dilated
(%nticholinerics such as atropine* h"osc"amine* and scopolamine)
.............................................................................................................................................
..............
1>5) female t with %t eye ain and redness with watery discharge5no
h.o trauma5itching5(04 there is diffuse congestion in the con6unctiva
and watery discharge what youSll do:
a. give 'b
b. give antihistamine
c. toical steroid
d. refer her to the ohthalmologist
???????????????
KKKKKKKKKKKKKKKKKKKKKKKKKKKKKKKKKKKKKKK
KKKKKKKKKKKKKKKKK..
1>:) 4idemic disease in oor sanitation areas affecting children and
young adults:
a. he '
b. "
c. +
d. /
The correct answer is a
1>=) mths baby with crying eisodes.current 6elly stool5looks slightly
ale5signs of obstruction wht is your &D:
a. barium enema
b. immediate surgery
c. I.v fluid J wait for resolution
The correct answer is a
Intussuscetions :
+ondition in which one portion of bowel invainates into an adHacent
sement Fusuall" pro$imal to ileocecal valveF . most common in first two
"ears of life F usuall" between I months and I "ears of lifeF . %brupt -onest *
colic#" abdominal pain * often accompanied b" fle$ed #nee and vomitin * (
one-off pain ) child ma" appear well between episodes. +lassic triad !
abdominal pain * vomitin * blood per rectumF onl" one third of pt.F . 4ate
sins ! blood" mucus in stool Fcurrant Hell" stoolF * abdominal tenderness *
palpable Fsausae- shape F . RB2 abdominal mas.
Investigation J treatment :
-correct an" volume and electrol"te abnormalit" and chec# ( cbc)
%bdominal film ma" be normal in earl" stae * and see obstruction *
perforation in late stae* BS see F taret sinF
In settin of hih clinical suspicion NNN air-contrast barium enema* should be
performed without dela". %s dianostic in UQ< of cases and curative in :;<
of cases perform surical reduction of anrenous bowel.
KKKKKKKKKKKKKKKKKKKKKKKKKKKKKKKKKKKKKKK
KKKKKK.KKKKKKKKK......
1>>) 1= y.o adolescent5 athletic 5with h0o %t foot ain lanter
surface5diagnosis is:
a. lanter fasciaitis
b. valuD$$
c. valuD$..
The correct answer is a.
-lanter fasciitis( heel sur syndrome ) :
Repetitive strain inHur" causin micro tears and inflammation of planter
fascia. +ommon in athletes * also associated with ..3 * obesit" *
seroneative and seropositive arthritis.
+linical feature: mornin pain and stiffiness * intense pain when wal#in
from rest and that subsides as pt. continuous to wal#. Swellin and
tenderness over sole* reatest at medical calcaneal tubercle and 0-1 cm
distal to alon planter fascia . pain with toe dorsifle$ion
1>C) regnant lady 1: wks resented with vaginal bleeding 5enlarged
abdomen5vomiting 5her uterus is smaller than eDected for the
gestational age5"h+E >35*0S snowstorm aearance5diagnosis:
a. comlete hydatiform mole
b. artial hydatiform mole
The correct answer is a
KKKKKKKKKKKKKKKKKKKKKKKKKKKKKKKKKKKKKKK
KKKKKKKKKKKKKKKK.....
1C3) 13. 12 y.o boy c0o abdominal ain after laying football5 he denied
any h0o trauma5 the ain is in the Bt araumbilical region what inD you
want to do:
a. +H%
b. ultrasound kidney
A not comlete
KKKKKKKKKKKKKKKKKKKKKKKKKKKKKKKKKKKKKKK
KKKKKKKKKKKKKKKK.....
1C1) 5 y.o child with h.o fever and swelling of the face ant to the both
ears (arotid gland enlargement) what is the most common
comlication:
a. Babrynthitis
b. meningitis
c. encehalitis
d. orchitis
The correct answer is b
# most common complication of mumps after pubert" . but it is rare occur
pre-pubert".
KKKKKKKKKKKKKKKKKKKKKKKKKKKKKKKKKKKKKKK
KKKKKKKKKKKKKKKK
1C2) . what is the meaning of difficulty breathing:
a. dysnia
b. tachycardia
The correct answer is a

1C)) ' female atient on the )
rd
week ostartum. She says to the
hysician that the freTuently visualiGes snakes crawling to her baby?s
bed. She knows that it is imossible but she cannot remove the idea
from her head. She says she wakes u around 53 times at night to
check her baby. !his roblem revents her from getting good slee and
it started to affect her marriage. 7hat is this roblem she is
eDeriencingO
a. an obsession
b. ' hallucination
c. ' ostartum sychosis
d. ' /elusion
The correct answer is a ( 0;; < correct )
#obsession : persistent* unwanted* and intrusive ideas * thouhts* impulses or
imaes
#hallucination! perception of obHective or event without e$istin e$ternal
stimulus #illusion: false perception of actual e$ternal stimulus.
# delusion: fi$ed false idios"ncratic belief.
KKKKKKKKKKKKKKKKKKKKKKKKKKKKKKKKKKKKKKK
KKKKKKK.KKKKKKKKKK.
1C2) female t c0o sever migraine that affecting her work5she mentioned
that she was imroved in her last regnancy5to revent that:
a. biofeedback
b. roranolol
The correct answer is b
# migraine rohylactic :
0- b-bloc#er (propranolol) ! first line treatment (note! contindication in
prenanc") 1- T+%
(amitript"line )
I- anticovulsant
R- calcium channel bloc#er
KKKKKKKKKKKKKKKKKKKKKKKKKKKKKKKKKKKKKKK
KKKKKKKKKKKKKKKKK..
1C5) 'bout /& in NS':
a.. most of ,I//& are obese ( correct )
KKKKKKKKKKKKKKKKKKKKKKKKKKKKKKKKKKKKKKK
KKKKKKKKKKKKKKKKK.
1C:) 8lu vaccine do not given to the baby who is allergic to:
a. egg
contraindication of flu vaccine are egg allergic and hyersensitivity to
thimerosal
1C=) 1=. -t with asymtomatic !richomniasis:
a. treat her anyway regardless
b. treat her if she is symtomatic only
The correct answer is a
#treatment of trichomnias :
Sinle dose of metradina7ole for s"mptomatic and as"mptomatic pt. and
treatment partner ( because it is se$ual transmitted disease )
6ut not treatment of as"mptomatic trichomniasis in first trimester of
prenanc". (contraindication in prenanc" )
KKKKKKKKKKKKKKKKKKKKKKKKKKKKKKKKKKKKKKK
KKKKKKK..KKKKKKKKKK
1C>) = y.o5she missed her second dose of varecila vaccine5the first one
about 1 y ago what youSll do:
a. give her double dose vaccine
b. give her the second dose only
c. see if she has antibody and act accordingly
The correct answer is b
KKKKKKKKKKKKKKKKKKKKKKKKKKKKKKKKKKKKKKK
KKKKKKKKKKKKKKKKK.
1CC) t with gonorrhea infection what else you want to check for
a. +lamydia trachomatis

KKKKKKKKKKKKKKKKKKKKKKKKKKKKKKKKKKKKKKK
KKKKKKKKKKKKKKKKK.
233) female t with 'ortic stenosis5she develoed syncoe while she
was in the class and she recovered immediately5what is the cause of
syncoe:
a. valvular ruture
b. systemic hyotension
The correct answer is b
KKKKKKKKKKKKKKKKKKKKKKKKKKKKKKKKKKKKKKK
KKKKKKKKKKKKKKKKK..
231) diabetic t well controlled5she came with h.o diGGiness and
sweating after taking a medication "S::3 what drug that cause her
rob:
-Sulfan"urea can cause this side effects.
232) male t with acute urine retention what is your action:
a. insert follySs cath and ask him to come back to the clinic
KKKKKKKKKKKKKKKKKKKKKKKKKKKKKKKKKKKKKKK
KKKKKKKKKKKKKKKKK.
23)) In battered women which is true:
a. mostly they come from oor socioeconomic area
b. usually they marry a second violent man
c. mostly they come to the 40% c0o$$$..
d. mostly they think that the husband resond like this because they
still have . strong feeling for them
%nswer ( d )
"attered women is Eomen who are ph"sicall" and mentall" abused over an
e$tended period* usuall" b" a husband or other dominant male fiure.
+haracteristics of the battered woman s"ndrome are helplessness* constant
fear* and a perceived inabilit" to escape. So I think choice d is correct
answer
232) smoking withdrawal symtoms eak at:
a. 1#2 days
b. 2#2 days
c. 5.= days
d. 13#12 days
The correct answer is a
KKKKKKKKKKKKKKKKKKKKKKKKKKKKKKKKKKKKKKK
KKKKKK.KKKKKKKKKKK.
235) &other who is breast feeding and she want to take &&% vaccine
what is your advice:
a. can be given safely during lactation
b. contain live bacteria that will be transmitted to the baby
c. sto breast feeding for =2 hrs after taking the vaccine
The correct answer is a
&&% ! contraindication durin prenanc" * and women should be avoid
prenant in R wee#s followed 33R vaccine . 33R is safe durin
lactation.

23:) male t c0o ain in his %t elbow5he said that he is using the hummer
a lot in his work diagnosis:
a. lateral eichondylitis
#Bateral eicondylitis( inflammation of common e$tensor tendon ) also #nown
as (tennis elbow* shooterCs elbow and archerCs elbow is a condition where the
outer part of the elbow becomes sore and tender. It is commonl" associated with
pla"in tennis and rac,uet sports.
-&edial eicondylitis (inflammation of common fle$or elbow) also #now (olfer
elbow)
KKKKKKKKKKKKKKKKKKKKKKKKKKKKKKKKKKKKKKK
KKKKKKKKKKKKKKKKK
23=) 53 y.o male with difficulty swallowing food with wt loss:
1. (esohageal cancer
,uestion not complete . but most li#el" is cancer.
causes of d"sphaia food than li,uid are !
0- carcinoma K 1- stricture K I- plummer vision s"ndrome V web ( iron
deficien" anemia A olssitis )
KKKKKKKKKKKKKKKKKKKKKKKKKKKKKKKKKKKKKKK
KKKKKKKKKKKKKKKKK
23>) . young female with ain in her elbow(lateral eichondylitis) best
treatment is :
a. $$$$..,S'I/
b. electric $$$$.
,uestion not complete .
# treatment of lateral eichondylitis:
0- first line ! NS%I. Arest A ice
1- second line ! corticosteroid inHection
I- third line ! surer" .percutaneous release of common tendon

KKKKKKKKKKKKKKKKKKKKKKKKKKKKKKKKKKKKKKK
KKKKKKKKK..KKKKKKK..
23C) what drug that imrove the survival in +F8
1. digoDin
2. FydralaGin
,uestion is not complete .
Improve mortailit" in +/8 ! 6-bloc#er A %+5 A %R6 ( most imortant
is '+4 )
213) old man with bilateral knee ain and tenderness that increase with
walking and relieved by restV
a. %'
b. ('
The correct answer is b
('! pain with activit" and weiht bearin . and improve with rest .
%'! mornin stiffness N 0 hour . painful and warm swellin of multiple
s"mmetric Hoint .
KKKKKKKKKKKKKKKKKKKKKKKKKKKKKKKKKKKKKKK
KKKKKKKKKKKKKKKKK
211) %egarding eritonitis:
a.+omlicated aendectomy the cause is anerobe organism
b. rigidity and the cause is aralytic ileus
c. can be caused by chemical erosions
d. $$$$..
The correct answer is c
KKKKKKKKKKKKKKKKKKKKKKKKKKKKKKKKKKKKKKK
KKKKKKKKKKKKKKKKK..
212) regarding &I all of the following true eDcet:
a. unstable angina5longer duration of ain and can occur even at rest.
b. stable angina5shorter duration and occur with eDcertion
c. there should be T wave in &I
d. even if there is very ainful unstable angina the cardiac enGymes will
be normal
e. $$$$$$
all of the above are correct OOO
KKKKKKKKKKKKKKKKKKKKKKKKKKKKKKKKKKKKKKK
KKKKKKKKKKKKKKKKK
21)) -t with scoliosis5 you need to refer him to the ortho when the degree
is:
a. 5
b. 13
c. 15
d. 23
The correct answer is d
212) : mths baby with undescending testis which is true:
a. till the mother that he need syrgery
b. in most of the cases sontaneous descent after 1 year
c. surgery indicated when he is 2 years
d. unlikely to become malignant
The correct answer is a
-after 0 "ear testes not desendin. %nd surer"(orchiope$") do from P -
0:months
KKKKKKKKKKKKKKKKKKKKKKKKKKKKKKKKKKKKKKK
KKKKKKKKKKKKKKKKK
215) 22 y.o -t with asymtomatic congenital inguinal hernia:
a. immediate surgery
b. surgery indicated when he is Q)5 y
c. elective surgery if it is reducible
d. $$$$..
The correct answer is c
KKKKKKKKKKKKKKKKKKKKKKKKKKKKKKKKKKKKKKK
KKKKKKKKKKKKKKKK..
21:) the most effective thing regarding counseling:
a. family raort
b. well ad6usted aointment before counseling
c. $$$$$$.
OOOOO
KKKKKKKKKKKKKKKKKKKKKKKKKKKKKKKKKKKKKKK
KKKKKKKKKKKKKKKK..
21=) 5 years old +0( liming in +! there is a venous necrosis ttt is:
a. surgery
b. slent
c. hysiothery
A not comlete!
!his case is UerthSs diseasU ! vascular necrosis of femoral head.
Q- 0; "ears . usuall" self limitin with s"mptoms lastinT 0:
months. Present with painless limb * limited abduction and internal
rotation.
Treatment! 0- observation if there is limited femoral head
involvement or full R&3 . 1- if e$tensive or if decrease R&3*
consider bracin* hip abduction with cast.
21>) -t. has /& and renal imairment when he had diabetic
nehroathy:there is curve for albumin
a. 5y
b. 13y
c. 23y
d. 25y
!he correct is b
$$$$$$$$$$$$$$$$$$$$$$$$$$$$$$$$$$$$$$$
$$$$$$$$$$$
21C) -t has alGahimar disease and halusination and delusion ttt:
a. Faloridole
Ps"chotic s"mptoms (e.. hallucinations and delusions)* aitation and
aressive behavior are common in patients with %l7heimerWs .isease. %
stud" suests that haloperidol at a dose of 1-I msOda" is effective and
well tolerated b" most patients.
223) EeneraliGe aneDity disorder best ttt:
a. SS%Is
b. tricyclic ' /
The correct answer is a
$$$$$$$$$$$$$$$$$$$$$$$$$$$$$$$$$$$
$$$$$$$$$$$$$$$
221) &a6or deression management:
a. Intial theray even sever
&anagement of ma6or deression disorder:
1#harmacotheray! effective in Q; - D;< .allow for 1-P wee#s to ta#e
effect * treat more than P months ( SSRI* T+%s* 3%&Is).
2#sycotheray! ps"chotherap" combined with antidepressant is more
effect than either treatment alone.
)# 4lectroconvulsion ( 4++! ).
2# hototheray: effective for pt. who has a seasonal pattern.
$$$$$$$$$$$$$$$$$$$$$$$$$$$$$$$$$$$
$$$$$$$$$$$$$$$
222) -sychiatric t with un comliance of drugs ttt:
a. dero haloredol
$$$$$$$$$$$$$$$$$$$$$$$$$$$$$$$$$$$
$$$$$$$$$$$$$$$
22)) Strongest risk factor for strok:
a. Fyertension
b. 'trial fibrillation
The correct answer is a
222) -t. with salingitis and there is swelling in elvis in osterior forneD
and it is fluctuant m:
a. +olotomy
b. Baroscoic
The correct answer is a (ICm not sure )
colotomy5 also #nown as a vainotom"* is a procedure b" which an
incision is made in the vaina.
Purpose! % colpotom" is performed either to visuali7e pelvic structures or
to perform surer" on the fallopian tubes or ovaries.

$$$$$$$$$$$$$$$$$$$$$$$$$$$$$$$$$$$
$$$$$$$$$$$$$$$
225) +hild swallowing battery in the oesohegus5 managementO
a. broncoscoby
b.endoscoy
22:) In the aendisitis the histology is:
a. leukocyte in muscle
b. layer of lymhoid
c. tumor
d. lasma cell
the correct answer is a
in aendicitis ! neutrophil e$udation throuhout mucus* submucus* and
muscularis

$$$$$$$$$$$$$$$$$$$$$$$$$$$$$$$$$$$
$$$$$$$$$$$$$$$
22=) water in the body:
a. 239
b. differ deend on age and seD
the correct answer is b
@u"tonCs Textbook of Medical Physiology states that Fthe total amount of
water in a man of averae weiht (D; #ilorams) is appro$imatel" R; liters*
averain QD< of his total bod" weiht. In a newborn infant* this ma" be as
hih as DQ< of the bod" weiht* but it proressivel" decreases from birth to
old ae* most of the decrease occurrin durin the first 0; "ears of life. %lso*
obesit" decreases the percentae of water in the bod"* sometimes to as low
as RQ<F.
$$$$$$$$$$$$$$$$$$$$$$$$$$$$$$$$$$$
$$$$$$$$$$$$$$$
22>) +orneal ulcer ttt:
a. 'ntibiotic and cyclolagia is mydrasis and refer to ohthalmology
-6ecause infection is a common occurrence in corneal ulcers* "our
ophthalmoloist will prescribe antibiotic e"edrops. If the infection appears
ver" lare* "ou ma" need to use these drops as often as one drop an hour.
-&ral pain medications will be prescribed to control the pain. Pain can also
be controlled with special e"edrops that #eep "our pupil dilated
(%nticholinerics such as atropine* h"osc"amine* and scopolamine)
22C) regarding drainage of the abscess one of the following is true:
a. +arbuncle and frunculosis need drainage
b. *sually give cihtriaDon and enicillin ost draing
the correct answer is a
8runculosis and carbuncles are pus-filled infected lumps on the s#in. The"
usuall" occur as a one-off in a health" person. Treatment commonl" involves
drainin the pus and ta#in a course of antibiotics. If "ou have recurrin boils
"ou ma" be advised to have tests to chec# for an underl"in cause.

$$$$$$$$$$$$$$$$$$$$$$$$$$$$$$$
$$$$$$$$$$$$$$$$$$$
2)3) Salingitis and -I/ on enicillin but not imrove the most likely
organism is :
a. +hlamydia trachomatis
b. ,eisseria gonorrhoeae
The correct answer is b
5mpiric antibiotic reimens should be aimed at treatin li#el" causative
aents* that is* N. gonorrhoeae, C. trachomatis, enital m"coplasmas* and
bacterial vainosis-associated endoenous microflora. The latter include
anaerobic (acteroides and Pre!otella species and anaerobic streptococci)
as well as aerobic oranisms (". !aginalis, #. coli, and facultative
streptococci). 5$cept for N. gonorrhoeae and some anaerobes* resistance is
not "et a clinical problem.

$$$$$$$$$$$$$$$$$$$$$$$$$$$$$$$$$$$
$$$$$$$$$$$$$$$
2)1) 7ound at end inflammatory hase when:
a. 4ithelial tissue formation
b. 'ngiogenisis
c. when the wound clean
d. Scar formation
The correct answer is b
2)2) Wuvenile %' ttt:
a. 'sirin
b. Steroid
c. -enicillamine
d. dydrocloroTuin
The correct answer is a
The treatment of LR% focuses on suppressin inflammation* preservin and
ma$imi7in function* preventin deformit"* and preventin blindness.
NS%I.s are the first choice in the treatment of LR%.
$$$$$$$$$$$$$$$$$$$$$$$$$$$$$$$$$$$
$$$$$$$$$$$$$$$
2))) -t. has hemorrhoid with 2 degree ttt:
a. Femoridectomy
b. band ligation
c. sclerotheray
d. fiber diet
-The correct answer is a.
# +lassification of internal hemorrhoid :
8irst degree : hemorrhoid do not prolapsed
Second degree ! hemorrhoid prolapsed upon defecation but spontaneousl"
reduce
!hird degree ! hemorrhoid prolapsed upon defecation * but must be manuall"
reduce
8ourth degree ! hemorrhoids are prolapsed and cannot be manuall" reduce
-treatment:
8irst J Second : life st"le modification ( dite)
!hird : life st"le modification with (band liation* sclerotherap" or
cr"otherap" ) * if
8ailed o to surer".
8ourth : surer" ( hemorrhiodectom" )
2)2) -t. come with history of tinia caitis ttt:
a. tar shamoo
b. 8luconaGol
The correct answer is b
Newer antifunal medications* such as #etocona7ole* itracona7ole*
terbinafine* and flucona7ole* have been reported as effective
alternative therapeutic aents for tinea capitis. &f these aents*
itracona7ole and terbinafine are used most commonl".
$$$$$$$$$$$$$$$$$$$$$$$$$$$$$$$$$$$
$$$$$$$$$$$$$$$
2)5) -t. with history of +(-/ the most action to revent comlication is:
a. -nemoccoccal vaccine
b. Smoking sto
c. (ral steroid
d. "2 agonist
The correct answer is b
$$$$$$$$$$$$$$$$$$$$$$$$$$$$$$$$$$$
$$$$$$$$$$$$$$$
2):) -t. with congenital hi dislocation :
a. abducting at fleDed hi can causes click or tali
+onenital hip dislocation (+/.) occurs * more commonl" in irls than in bo"s. The left
hip is twice as often involved as the riht and bilateral dislocation occurs in more than 1Q
percent of affected children. The criteria for the dianosis of conenital dislocation of the
hip include both ph"sical and radioraphic findins. +ertain clinical sins have been
identified that are helpful in the evaluation of newborns and infants for possible +/.*
which include the followin!
limited abduction of the fle$ed hip* due to shortenin and contraction of the hip
adductors9
increase in depth or as"mmetr" of the inuinal or thih s#infolds9
shortenin of one le9
%llisC or @alea77iCs sin -- lower position of #nee of the affected side when #nees
and hips are fle$ed* due to location of femoral head posterior to acetabulum in
this position9
&rtolaniCs FHer#F sin (Fclun# of entr"F or reduction sin)9
6arlowCs test (Fclun# of e$itF or dislocation sin)9
telescopin or pistonin action of thihs* due to lac# of containment of femoral
head with acetabulum9
TrendelenburCs test -- drop of normal hip when child* standin on both feet*
elevates unaffected limb and bears weiht on affected side* due to wea#ness of
hip abductors9
waddlin t"pe of ait.
There are characteristic radioraphic presentations which are present in each of the
staes of +/.. The term conenital hip d"splasia enerall" refers to dela"ed or
defective development of the hip Hoint leadin to a deraned articular relationship
between an abnormal acetabulum and a deformed pro$imal end of the femur. This
condition is considered a precursor of the clinical entities of sublu$ation and dislocation
of the hip. In conenital sublu$ation of the hip* there is an abnormal relationship
between the femoral head and acetabulum* but the two are in contact. +onenital
dislocation of the hip* on the other hand* is associated with a complete loss of contact of
the femoral head with the acetabular cartilae. Bnfortunatel"* the femoral head and
acetabulum in the newborn cannot be assessed b" direct visuali7ation* since the femoral
head is not ossified and is a cartilainous bod" which is not visuali7ed on plain films. The
ossification center for the femoral head enerall" appears between three and si$
months9 a dela" in its appearance is an indication of conenital hip d"splasia. The nec#
of the femur is used for ascertainin the relationship between the acetabulum and the
femoral head until the ossification center appears. The measurement used to evaluate
the relationship of the femoral head and acetabulum include /ilenreiner line* acetabular
inde$* Per#in-&mbredanne line* Shenton-3enard line* and %ndren-von Rosen line.
The principal treatment for +/. is conservative* especiall" if dianosed earl". The most
common techni,ue is to reduce the dislocation of the femoral head b" means of a
fle$ionOabduction maneuver* for a sufficient period of time to permit proper rowth of the
head and acetabulum* which in turn assures a conruent and stable hip Hoint. This
techni,ue is usuall" performed on patients in the ver" earl" staes of +/. and in infants
under two "ears of ae9 which include splintin* with a 8reH#a splint or Pavli# harness.
+olonna or 6uc#Cs s#in traction is used in children 1 to 01 "ears of ae* with a well-
padded spica cast applied simultaneousl" to the unaffected side
2)=) +olon cancer with stage ) give the chemotheray:
a. 's soon as ossible
b. 1 month
The correct answer is a (ICm not sure because 3+2s not complete )
Treatment for Stae I +olon +ancer
Treatment for stae I colon cancer enerall" consists of a surical resection
followed b" chemotherap". In a surical resection* a sureon removes the
part of the colon affected b" the tumor and Hoins the remainin health"
sections toether to form one lon* health" piece.
The standard chemotherap" reimen used to be si$ months of treatment
with Q-8B and leucovorin* but that Fcoc#tailF was developed decades ao
and is seldom used an"more. there are man" new chemotherap" reimens
available for stae I colon cancer.
$$$$$$$$$$$$$$$$$$$$$$$$$$$$$$$$$$$
$$$$$$$$$$$$$$$
2)>) 5: y old resent with vasomotor rhinitis
a. Bocal anti histamine
b. Bocal decongestion
c. Bocal steroid
d. Systemic antibiotic
The correct answer is b
Vasomotor rhinitis is a nonalleric condition that causes a constant
runn" nose* snee7in* and nasal conestion.

$$$$$$$$$$$$$$$$$$$$$$$$$$$$$$$$$$$
$$$$$$$$$$$$$$$
2)C) Sickle +ell 'nemia give rohlaDis:
a. -enicillin
b. Iron
The correct answer is a
TREATMENT
X 'cute crisis: %nalesia and h"dration.
X /"dro$"urea to Ythe amount of fetal hemolobin.
X $. influen%ae and pneumococcal vaccines9 proph"lactic penicillin for
+hildren Q "ears of ae.
X 'cute chest syndrome: Respirator" support and e$chane transfusion.
223) diagnosis of thalasimia minor:
a. Fb '2 and Fb f
b. &icrocytosis
the correct answer is a
$$$$$$$$$$$$$$$$$$$$$$$$$$$$$$$$$$$
$$$$$$$$$$$$$$$
221) -t. with &+@ decrease and reticulocyt decrease iron deficiency
anaemia investigation:
a. 8erritin level and !I"+ and serum iron
$$$$$$$$$$$$$$$$$$$$$$$$$$$$$$$$$$$
$$$$$$$$$$$$$$$
222) "orn "+E
a. 1month heatitis b oral olio dt
b. 2month s
c. )month s
d. C to 12 month mmr
OOOOO I canSt understand th A ( may the writer A missed some
information )
22)) 4.histolytica cyst is destroyed by:
a. 8reeGing
b. "oiling
c. Iodine treatment
d. +hlorine
The correct answer is b
%mebiasis (or amoebiasis) is the name of the infection caused b" 5. histol"tica.
To help prevent infection!
Z %void raw veetables when in endemic areas as the" ma" have been fertili7ed
usin human feces.
Z 6oil water or treat with iodine tablets
KKKKKKKKKKKKKKKKKKKKKKKKKKKKKKKKKKKKKKK
KKKKKKKKKKKKKKKKK
222) -atient after accident5 the left ribcage move inward during
insiration and outward during eDiration:
a. 8lial chest
8lail chest is a clinical anatomic dianosis noted in blunt trauma patients
with parado$ical or reverse motion of a chest wall sement while
spontaneousl" breathin. This clinical findin disappears after intubation with
positive pressure ventilation* which occasionall" results in a dela"ed
dianosis of the condition.
KKKKKKKKKKKKKKKKKKKKKKKKKKKKKKKKKKKKKKK
KKKKKKKKKKKKKKKKK
225) Ereatest risk of stroke:
a. /&
b. 4levated blood ressure
c. 8amily history of stroke
d. Fyerliedemia
e. Smooking
the correct answer is b
22:) +hild has allor 5 eats little meat 5 by investigation :microcytic
hyochromic anemia . what will you do:
a. !rial of iron therary
b. &ultivitamin with iron daily
the correct answer is a ( ICm not sure 6ecause 2 is not complete )
KKKKKKKKKKKKKKKKKKKKKKKKKKKKKKKKKKKKKKK
KKKKKKKKKKKKKKKKK
22=) !reatment of mania that does not cause heatotoDicity
a. Bithium
b. carbamaGeine
c. valoric acid
d. lamotrigine
The correct anser is not a
Treatment of mood disorder !
Bithium : hepatoto$icit"
carbameGaine : arnuloc"tosis
valoric asid : neural tube defect
KKKKKKKKKKKKKKKKKKKKKKKKKKKKKKKKKKKKKKK
KKKKKKKKKKKKKKKKK
22>) Sickle cell anemia atient 5 the macula is cherry red 5 and absence
of afferent aillary light refleD
a. %etinal artery occlusion.
. b. veine occlusion
the correct answer is a * cherr" red spot is sin of retinal ater"
occlusion.

22C) Inflammatory bowel disease is idioathic but one of following is
ossible underlying cause
a. Immunological
Inflammatory "owel /isease +auses
Researchers do not "et #now what causes inflammator" bowel disease.
Therefore* I6. is called an idiopathic disease (disease with an un#nown
cause).
%n un#nown factorOaent (or a combination of factors) triers the bod"Ws
immune s"stem to produce an inflammator" reaction in the intestinal tract
that continues without control. %s a result of the inflammator" reaction* the
intestinal wall is damaed leadin to blood" diarrhea and abdominal pain.
KKKKKKKKKKKKKKKKKKKKKKKKKKKKKKKKKKKKKKK
KKKKKKKKKKKKKKKKK
253) -atient resent with high blood ressure (systolic 233) 5 tachycardia
5 mydriasis 5 sweating . what is the toDicity:
a. 'nticholenergic
b. Symathomimetic
c. !ricyclic antideressant
d. (rganohoshorous comounds
the correct answer b
ses o! "ympathomimetics# To treat and pre!ent re!ersible
bronchospasm associated with bronchial and nocturnal &nighttime'
asthma, chronic bronchitis,emphysema, exercise-included bronchospasm
and other obstructi!e airway disease of the lungs.
To treat serious allegoric reactions &epinephrine in(ection only'
251) !reatment of +hlamydia with regnancy:
a. 'Githromycin
b. 4rythromycin base
the correct answer b
$or treatment o! chlamydia d%rin& pre&nancy, the CDC recommends:
-er"throm"cin base* Q;; m orall"* four times a da" for seven da"s9 or amo$icillin
(%mo$il)* Q;; m* three times a da" for seven da"s.
'lternatives include:
-5r"throm"cin base 1Q; m* four times a da" for 0R da"s9
-er"throm"cin eth"lsuccinate :;; m orall"* four times a da" for seven da"s9 or
-er"throm"cin eth"lsuccinate R;; m orall"* four times a da" for 0R da"s.
KKKKKKKKKKKKKKKKKKKKKKKKKKKKKKKKKKKKKKK
KKKKKKKKKKKKKKKKK
252) the maDimum body lenght will be reached after menarch by
a #: months
b. 1 year
c. 2year
' child 'ill have also reached her final adult height about two years a!ter
menarche.
KKKKKKKKKKKKKKKKKKKKKKKKKKKKKKKKKKKKKKK
KKKKKKKKKKKKKKKKK
25)) -atient develoed sudden loss of vision bilaterally while she was
walking in the street 5 followed by numbness 5 the sub6ective symtoms
are different from ob6ective 5 and does not match anatomical athology5
what is your diagnosis:
a. +onversion syndrome
+onversion disorder!
S"mptoms or deficits of voluntar" motor or sensor" function (e..* blindness*
sei7ure) suest a condition incompatible with medical processes. +lose
temporal relationship to stress or intense emotion. 3ore common in "oun
females and in lower socioeconomic and less educated roups.
KKKKKKKKKKKKKKKKKKKKKKKKKKKKKKKKKKKKKKK
KKKKKKKKKKKKKKKKK
252) 'fter inflammatory hase of wound 5 there will be wound healing by:
a. If the wound is clean
b. 'ngiogenesis
c. 4ithelial tissue
. the correct answer is b
255) (ld male with tender knee 5 ain 5 creitus . the diagnosis:
a. (steoarhritis
b. 'nkylosin sondylitis
c. %heumatoid
the correct answer a
(steoarhritis
('%!F%I!IS ((')
% chronic* noninflammator" arthritis of movable Hoints (e..* .IP Hoints).
+haracteri7ed b" deterioration of the articular cartilae and osteoph"te
formation at Hoint surfaces.
Ris# factors include a [ famil" histor"* obesit"* and a histor" of Hoint
trauma.
/$OP5! +repitus9 decrease rane of motion (R&3)9 pain that worsens with
activit"
and weiht bearin but improves with rest.
KKKKKKKKKKKKKKKKKKKKKKKKKKKKKKKKKKKKKKK
KKKKKKKKKKKKKKKKK
25:) &other has baby with cleft alate and asks you what is the chance
of having a second baby with cleft alate or cleft li 5
a. 259
b. 539
c. 91
d. 29
the correct answer is d
KKKKKKKKKKKKKKKKKKKKKKKKKKKKKKKKKKKKKKK
KKKKKKKKKKKKKKKKK-
25=) 1 liter fluid deficit eTuals :
. a.1 kg
. 4iter of fluid deficit e,ual 0 # of fluid as h"dration protocol
KKKKKKKKKKK.
KKKKKKKKKKKKKKKKKKKKKKKKKKKKKKKKKKKKKK....
................
25>) 'fter accident atient with tachycardia5 hyotension5 what will be
your initial ste:
a. %aid I@8 crystalloid
b. +!s
!he correct answer is a
A not comlete . management '"+ then raid I@8 crystalloid.
25C) : years child was born to F"S ositive mother is F"S ositive 5 he
was only vaccinated by "+E after birth 5 what you will give him now :
a. F"@ . oral olio . /!- . hib
b. F"@ . oral olio . dt . &&% .hib
c. F"@ . oral olio . /t . &&%
OOOOOOOOOO all the above are wrong
KKKKKKKKKKKKKKKKKKKKKKKKKKKKKKKKKKKKKKK
KKKKKKKKKKKKKKKKK
2:3) !reatment of non inflammatory acne
a. %etinoic acid
%etinoids! 3edicines structurall" similar to vitamin % are useful in
preventin several t"pes of acne lesions. Topical retinoids are effective in
treatin the noninflammator" t"pes of acne (blac#heads and whiteheads).
KKKKKKKKKKKKKKKKKKKKKKKKKKKKKKKKKKKKKKK
KKKKKKKKKKKKKKKKK
2:1) !reatment of comedones:
a. !oical retinoids.
+omedones: The plural of comedo* the primar" sin of acne* consistin of
a dilated (widened) hair follicle filled with #eratin s,uamae (s#in debris)*
bacteria* and sebum (oil). +omedones ma" be closed or open.
KKKKKKKKKKKKKKKKKKKKKKKKKKKKKKKKKKKKKKK
KKKKKKKKKKKKKKKKK
2:2) !reatment of aules or ustules:
a.!oical benGoyl
b.-eroDide lus toical antibiotics5 mainly clindamycin or
erythromycin. c. In severe cases5 intralesional
steroid in6ection or oral antibiotics5 such as tetracycline or
erythromycin may be added.
The correct answer b
TREATMENT
&ild acne: Topical clindam"cin or er"throm"cin9 ben7o"l pero$ide9 topical
retinoids.
&oderate acne: The above reimen plus oral antibiotics such as
tetrac"cline.
Severe nodulocystic acne: &ral isotretinoin (%ccutane).
2:)) 7hich is not true In emergency management of stroke
a.Eive I@8 to avoid /5 539
b.Eive diaGeam in convulsions
. c.'nticonvulsants not needed in if seiGures
d.#&ust correct electrolytes
e. !reat elevated blood ressure
the correct answer is c
KKKKKKKKKKKKKKKKKKKKKKKKKKKKKKKKKKKKKKK
KKKKKKKKKKKKKKKKK
2:2) S +ell 'nemia comlications in adults
a. +erebral infarction
b. +erebral hemorrhage
Ne%rolo&ical complications occur in 1Q< of patients* with transient ischaemic attac#s*
fits* cerebral infarction* cerebral haemorrhae and coma. Stro#es occur in about
00< of patients under 1; "ears of ae. The most common findin is obstruction of a
distal intracranial internal carotid arter" or a pro$imal middle cerebral arter". 0;< of
children
without neuroloical sins or s"mptoms have abnormal blood-flow velocit" indicative of
clinicall" sinificant arterial stenosis9 such patients have ver" hih ris# of stro#e. It has
now been demonstrated that if children with stenotic cranial arter" lesions* as
demonstrated on transcranial .oppler ultrasonoraph"* are maintained on a reular
proramme of transfusion that is desined to suppress er"thropoiesis so that no more
than I;< of the circulatin red cells are their own* about U;< of stro#es in such children
could be prevented.
KKKKKKKKKKKKKKKKKKKKKKKKKKKKKKKKKKKKKKK
KKKKKKKKKKKKKKKKK
2:5) !he most common risk for intracerebral stroke
a. Fyertension
2::) !he antideressant used for deression that cause seDual
dysfunction
a.Sertatline (SS%Is)
b. Imairamine
c. BevofluDine
the correct answer a
KKKKKKKKKKKKKKKKKKKKKKKKKKKKKKKKKKKKKKK
KKKKKKKKKKKKKKKKK
2:=) -reviously healthy female atient resented to 4% with dysnea 5
aneDiety 5 tremor 5 and she breath heavily 5 the symtoms began 23
minutes before she came to 4% 5 in the hosital she develoed
numbness eriorbital and in her fingers 5 what you will do
a. 'sk her to breath into a bag
. b. !ake blood samle to look for alcohol toDicity
the correct answer is a
KKKKKKKKKKKKKKKKKKKKKKKKKKKKKKKKKKKKKKK
KKKKKKKKKKKKKKKKK
2:>) 7hat is the most imortant in councling
a. 4Dclude hysical illness
b. 4stablishing rabbot
c. 8amily
d. Schedule aointement
OOOOO
2:C) In breaking bad news
a. 8ind out how much the atient know
b. 8ind out how much the atient wants to know
the correct answer a
%obert "uckmanSs SiD Ste -rotocol for "reaking "ad ,ews
). "etting started .
1. *inding out how much the patient knows.
+. *inding out how much the patient wants to know .
,. -haring the information .
.. /esponding to the patients feelings .
0. Planning and follow-through .
$$$$$$$$$$$$$$$$$$$$$$$$$$$$$$$$$$$$$$$
$$$$$$$$$$$$$$$$
2=3) -atient with chest ain that aggrevated by coughing5 there is added
sound on left sternal border .in ecg you will find
a. S! changes
b. -% rolongation
c. Fyervoltage
the correct answer a . (percarditis)
$$$$$$$$$$$$$$$$$$$$$$$$$$$$$$$$$$$$$$$
$$$$$$$$$$$$$$$$
2=1) !he most common site for visceral hemangioma is
a. Biver ( most common site for visceral hemangioma )
% hepatic hemanioma is the most common noncancerous tumor of the
liver. It is believed to be a birth defect. /epatic hemaniomas can occur at
an" time* but are most common in people in their I;s - Q;s. Eomen are
affected more often than men* and usuall" have bier tumors than men.
6abies ma" develop a t"pe of hepatic hemanioma called benin infantile
hemanioendothelioma (also called multinodular hepatic hemaniomatosis).
This rare* noncancerous tumor has been lin#ed to hih rates of heart failure
and death in infants. Infants are usuall" dianosed b" the time the" are P
months old.
2=2) +hild with large eriorbital hemangioma 5 if this hemangioma cause
obstruction to vision 5 when will be ermenant decrease in visual acuity
a. 'fter obstruction by one day
b. "y 1 week
c. "y ) months
d. "y : months
OOOOOOOOO
KKKKKKKKKKKKKKKKKKKKKKKKKKKKKKKKKKKKKKK
KKKKKKKKKKKKKKKK
2=)) !he symtoms of soft tissue sarcoma
a. -rogressive enlarging mass ( on growing )
In their early stages5 soft tissue sarcomas usually do not cause
symtoms. "ecause soft tissue is relatively elastic5 tumors can grow
rather large5 ushing aside normal tissue5 before they are felt or cause
any roblems. !he first noticeable symtom is usually a ainless lum
or swelling. 's the tumor grows5 it may cause other symtoms5 such as
ain or soreness5 as it resses against nearby nerves and muscles. If
in the abdomen it can cause abdominal ains commonly mistaken for
menstrual crams5 indigestion5 or cause constiation
$$$$$$$$$$$$$$$$$$$$$$$$$$$$$$$$$$$$$$$
$$$$$$$$$$$$$$$$
2=2) )5 year female with bilateral breast ain 5 that decrease after
menstruation 5 the breast is nodular with rominent ) cm mass
subareolar 5 aDillary lymh nodes are not enlarged 5 what you will do
a. &ammograhy followed by *S
b. See her neDt cycle
c. 8ine needle biosy followed by tissue studies
the correct answer is a or c
2=5) -regnant with bleeding for 12 hours and tissue 5 the cerviD is 1 cm
a. +omlete abortion
. b. Incomlete abortion
. c. &olar regnancy
the correct answer is b
2=:) 2 month infant with vomiting after each meal 5 he is in 53 centile 5 Fe
assed meconium early and stool 5 diagnosis is
a. &idgut volvulus
b. &econium ileus (not assing early meconium and stool)
c. Fischsrung disease (not assing early meconium and stool)
I thin# this case is p"loric stenosis. If not mention choose a
$$$$$$$$$$$$$$$$$$$$$$$$$$$$$$$$$$$$$$$
$$$$$$$$$$$$$$$$
2==) -atient with dyshagia to solid and liTuid 5 and regurg 5 by barium
there is non eristalsis dilatation of osohagus and air#fluid level and
taering end.diagnosis is
a. (sohageal sasm
b. 'chalasia
c. (sohageal cancer
the correct answer is b
$$$$$$$$$$$$$$$$$$$$$$$$$$$$$$$$$$$$$$$
$$$$$$$$$$$$$$$$
2=>) !he most common cause of cough in adults is
a. 'sthma
b. E4%/
c. -ostnasal dri
The correct answer c
The most common causes of acute couh is respirator" viral infection. The
most common cause of chronic couh are postnasal drip* asthma* and acid
reflu$ from the stomach. These three causes are responsible for up to U;
percent of all cases of chronic couh.
2=C) Eirl with amenorrhea for many months . "&I is 23 and is stable over
last 5 years . diagnosis
a.4ating disorder
b.-ituitary adenoma
$$$$$$$$$$$$$$$$$$$$$$$$$$$$$$$$$$$$$$$
$$$$$$$$$$$$$$$$
2>3) (ld female with itching of vulva 5 by eDamination there is ale and
thin vagina 5 no discharge . what is management
a. 4strogen cream
b. +orticosteroid cream
c. 8luconaGole
the correct answer is a
Some women will opt for prescription medications as the" o throuh
menopause. The most common prescriptions include!
1 -/ormone replacement therap" or anti-depressants to minimi7e hot
flashes. 2 -8osama$ or %ctonel (non-hormonal medications) to
reduce bone loss and reduce the ris# of fractures .
) -Selective estroen receptor modulators (S5R3s)* which mimic estroenCs
beneficial effects on bone densit".
2#Vainal estroen* administered locall"* to relieve vainal dr"ness and
discomfort durin intercourse
$$$$$$$$$$$$$$$$$$$$$$$$$$$$$$$$$$$$$$$
$$$$$$$$$$$$$$$$
2>1) -atient with dysuria 5 freTuency 5 urgency 5 but no flank ain 5 what
is the treatment
a. +irofloDacin o od for )#5 days
b. ,orfloDacin er os for = K 12 days
the correct answer is b
2>2) -atient with flank ain 5 fever 5 vomiting 5 treatment is
a. FositaliGation and intravenous antibiotics and fluid
This is most li#el" a case of p"elonephritis which need urent
hospitali7ation
$$$$$$$$$$$$$$$$$$$$$$$$$$$$$$$$$$$$$$$
$$$$$$$$$$$$$$$$
2>)) -atient resent with mid face ain 5 erethematous lesions and
vesicles on eriorbital and forehead 5 the ain is at nose 5 nose is
erythematous . what is diagnosis
a. %oseola
. b. FS@
. c. Feres Goster
the correct answer is c

$$$$$$$$$$$$$$$$$$$$$$$$$$$$$$$$$$$$$$$
$$$$$$$$$$$$$$$$
2>2) &ale with itching in groin erythematous lesions and some have
clear centers 5 what is diagnosis :
a. -soriasis
b. !inea curis
c. 4rythrasma
the correct answer is b
Patients with tinea cruris report pruritus and rash in the roin. % histor" of
previous episodes of a similar problem usuall" is elicited. %dditional historical
information in patients with tinea cruris ma" include recentl" visitin a tropical
climate* wearin tiht-fittin clothes (includin bathin suits) for e$tended
periods* sharin clothin with others* participatin in sports* or coe$istin
diabetes mellitus or obesit". Prison inmates* members of the armed forces*
members of athletic teams* and people who wear tiht clothin ma" be
subHect to independent or additional ris# for dermatoph"tosis.4are patches
of er"thema with central clearin are centered on the inuinal creases and
e$tend distall" down the medial aspects of the thihs and pro$imall" to the
lower abdomen and pubic area.-
2>5) @asoconstrictive nasal dros comlication
a. %ebound henomenon

$$$$$$$$$$$$$$$$$$$$$$$$$$$$$$$$$$$$$$$
$$$$$$$$$$$$$$$$
2>:) !he useful eDcurcise for osteoarthritis in old age to maintain
muscle and bone Bow resistance and high reetion weight training:
a. +onditioning and low reetion weight training
b. 7alking and weight eDercise
the correct answer is b
5$ercise is one of the best treatments for osteoarthritis. The best
e$ercises for osteoarthritis suffers depend on what Hoints are affected.
Swimmin* wal#in* and c"clin are often the best e$ercises for people
with osteoarthritis. Tr" to et thirt" minutes of e$ercise five times per
wee#. The #e" is to start slowl".
$$$$$$$$$$$$$$$$$$$$$$$$$$$$$$$$$$$$$$$
$$$$$$$$$$$$$$$$
2>=) *nilateral worsening headache 5 nausea 5 eDcacerbeted by
movement and aggrevated by light in 1= old girl.
a. &igraine (-hotohobia5 vomiting)
b. +luster
the correct answer is a
2>>) /iet sulement for osteoarthritis
a. Einger
% lare number of dietar" supplements are promoted to patients with
osteoarthritis and as man" as one third of those patients have used a
supplement to treat their condition. @lucosamine-containin supplements are
amon the most commonl" used products for osteoarthritis. %lthouh the
evidence is not entirel" consistent* most research suests that lucosamine
sulfate can improve s"mptoms of pain related to osteoarthritis* as well as
slow disease proression in patients with osteoarthritis of the #nee.
+hondroitin sulfate also appears to reduce osteoarthritis s"mptoms and is
often combined with lucosamine* but there is no reliable evidence that the
combination is more effective than either aent alone. S-adenos"lmethionine
ma" reduce pain but hih costs and product ,ualit" issues limit its use.
Several other supplements are promoted for treatin osteoarthritis* such as
meth"lsulfon"lmethane* /arpaoph"tum procumbens (devilCs claw)*
+urcuma lona (turmeric)* and Iingiber officinale (ginger) 5 but there is
insufficient reliable evidence reardin lon-term safet" or effectiveness.
$$$$$$$$$$$$$$$$$$$$$$$$$$$$$$$$$$$$$$$
$$$$$$$$$$$$$$$
2>C) (ld male with abdominal ain 5 nausea 5 7"+ =. 7hat is true about
aendicitis in elderlyO
a. +t not usefull for diagnosis.
b. 7"+ is often normal.
c. %uture is common
d. If there is no fever the diagnosis of aendicitis is unlikely
e. 'nemia is common
the correct answer is c
Appendicitis in eIderIy
Appendicitis is a less common cause of abdominal pain in elderly patients than in younger patients, but
the incidence among elderly patients appears to be rising. nly approximately !"# of cases of acute
appendicitis occur in patients older than $" years, whereas one half of all deaths from appendicitis
occur in this age group. %he rate of perforation in elderly patients is approximately &"#, & times higher
than in younger adults. %his is largely because '&# of elderly patients wait more than () hours to
see* medical attention. %he diagnosis can be difficult to ma*e, since more than one half of patients in
this age group do not present with fever or leu*ocytosis. +urther confusing the picture, approximately
one third do not locali,e pain to the right lower -uadrant, and one fourth do not have appreciable right
lower -uadrant tenderness. nly ("# of elderly patients present with anorexia, fever, right lower
-uadrant pain, and leu*ocytosis. %he initial diagnosis is incorrect in )".&"# of patients in this age
range . / perforation is the most common compIication of appendicitis )

2C3) (ld atient with bilateral enlarged knee 5 no history of trauma 5 no
tenderness 5 normal 4S% and +#reactive roteins . the diagnosis is
a. (steoarthritis
b. Eout
c. Infectous arthritis
the correct answer is a
$$$$$$$$$$$$$$$$$$$$$$$$$$$$$$$$$$$$$$$
$$$$$$$$$$$$$$$
2C1) -atient has decrease visual acuity bilateral 5 but more in %t side 5
visual field is not affected 5 in fundus there is irregular igmentations
and early cataract formation .
a. %efer to ohthalmologist for laser theray
b. %efer to ohthalmologist for cataract surgery ??????????
the correct answer is a
$$$$$$$$$$$$$$$$$$$$$$$$$$$$$$$$$$$$$$$
$$$$$$$$$$$$$$$
2C2) 7hat is the most common treatment for 6uvenile rheumatoid
arthritis
a. Intraarticular in6ection of steroid
b. (ral steroid
c. -aracetamol
d. enicillamine
e. 'srin
the correct answer is e
the most common important for LR% is ( NS%I. )

$$$$$$$$$$$$$$$$$$$$$$$$$$$$$$$$$$$$$$$
$$$$$$$$$$$$$$
2C)) 7hich of the following decrease mortality after &I
a. &etorolol
b. ,itroglycerine
c. !hiaGide
d. &orhine
the correct answer is a
onl" b-bloc#er ans %S% are mortalit" benefit for treatment of anina
2C2) !he cardiac arrest in children is uncommon but if occur it will be
due to
a. -rimary
b. %esiratory arrest
c. hyovolemic shock
d. neurogenic shock
the correct answer is b
Proressive respirator" insufficienc" accounts for P;< of all
paediatric arrests.
$$$$$$$$$$$$$$$$$$$$$$$$$$$$$$$$$$$$$$$
$$$$$$$$$$$$$$$
2C5) (ld female with recurrent fracture 5 @itamen / insufeciency and
smoker . which eDogenous factor has the gretest eDogenous side effect
on osteoorosis.
a. (ld age
b. Smoking
c. . @it / insufeciency
d. +ontinue smoking
e. %ecurrent fracture
the correct answer is b
$$$$$$$$$$$$$$$$$$$$$$$$$$$$$$$$$$$$$$$
$$$$$$$$$$$$$$$
2C:) atient resented with sudden chest ain and dysnea 5 tactile vocal
fremitus and chest movemebt is decreased 5 by D#ray there is
decreased ulmonary marking in left side 5 diagnosis
a. atelectasis of left lung
b. sontaneous neumothoraD
c. ulmonary embolism
the correct answer is b
2C=) boy after running for hours 5 has ain in knee and mass on uer
surface of tibia
a. (sgood scatter disease
b. Iliotibial band
the correct answer is a
$$$$$$$$$$$$$$$$$$$$$$$$$$$$$$$$$$$$$$$
$$$$$$$$$$$$$$$
2C>) ancreatitis
a. 'mylase is slowly rising but remain for days
b. 'mylase is more secific but less sensitive than liase
c. %anson criteria has severity (redictive) in acute ancreatitis
d. -ain is increased by sitting and relieved by lying down
e. +ontracetive ills is associated
the correct answer is c
The %anson and Elasgow scoring systems are based on such
parameters and have been shown to have an :;< sensitivit" for predictin a
severe attac#* althouh onl" after R: hours followin presentation.
risk mortality is 239 with )#2 signs5 239 with 5#: signs5 1339 = signs.
2CC) -atient has fever 5 night sweating 5 bloody sutum 5 weight loss 5
d test was ositive . D#ray show infiltrate in aeD of lung 5 d test is
now reactionary 5 diagnosis
a. 'ctivation of rimary !"
b. sarcoidosis
c. +ase control is
d. "ackward study
the correct answer is a
The tuberculosis s#in test (also #nown as the tuberculin test or PP. test) is
a test used to determine if someone has developed an immune response to
the bacterium that causes tuberculosis (T6).
KKKKKKKKKKKKKKKKKKKKKKKKKKKKKKKKKKKKKKK
KKKKKKKKKKKKKKKKK..
)33) atient with /& resented with limited or decreased range of
movement assive and active of all directions of shoulder
a. froGen shoulder
b. imingment syndrome
c. osteoarthritis
the correct answer is a
KKKKKKKKKKKKKKKKKKKKKKKKKKKKKKKKKKKKKKK
KKKKKKKKKKKKKKKKK..
)31) 2> years old with irregular menses resented with fatigue and no
menstruation for ) months with increased igmentation around the
vaginal area with no other symtoms. ur neDt ste would be
a. reassure the atient
b. do a regnancy test
c. do ultrasound
the correct answer b
)32) ) years old resented with shortness of breath and cough at night
which resolved by itself in 2 days. he has FD of rash on his hands and
allergic rhinitis. he most likely had
a. crou
b. bronchial asthama
c. eilotitis
????????

0roup 1piglottitis
nset Days 2ours
+lu.li*e symptoms 3es 4o
0ough 5ever Absent
Able to drin* 3es 4o
Drooling saliva 4o 3es
+ever 678 978
5tridor 2arsh 5oft
:oice 2oarse muffed
$$$$$$$$$$$$$$$$$$$$$$$$$$$$$$$$$$$$$$$
$$$$$$$$$$$$$$$$
)3)) a man went on vacation. he noticed a white atch in his chest which
became more clear after getting a sun tan which was sread on his
chest.ur /D is
a . ytriasis versicolor
b. vitilligo
c. ytriasis roscea
the correct answer a
KKKKKKKKKKKKKKKKKKKKKKKKKKKKKKKKKKKKKKK
KKKKKKKKKKKKKKKKK..)32) a 2 years old resented with 2
day history of shortness of breath a seal like cough with no sutum and
mild fever. on eDamination he did not look ISll or in distress
a. acute eilossitis
b. crou
c. angioedema
The correct answer b
)32):5 years old came with knee ain. and limited movement. on
eDamination had creition on knee. dD
a. rhuematiod arthritis
b. osteoorosis
c. osteoarthritis
the correct answer c
KKKKKKKKKKKKKKKKKKKKKKKKKKKKKKKKKKKKKKK
KKKKKKKKKKKKKKKKK..
)35) which vitamin is given to new born to sto bleeding
a. vit. '
b. vit. /
c. vit. N
d. vit 4
e. vit +
the correct answer c
KKKKKKKKKKKKKKKKKKKKKKKKKKKKKKKKKKKKKKK
KKKKKKKKKKKKKKKKK..
)3:) a child came with congested throught and mild bulging of tymanic
membrane your /D was *%!I. !he
OOOOOOOOO

KKKKKKKKKKKKKKKKKKKKKKKKKKKKKKKKKKKKKKK
KKKKKKKKKKKKKKKKK..
)3=) a child resented with erythematous harynD5 with cervical lymh
nodes and raid strlysin test negative and low grade fever with
ositive 4"@. it neDt ste
a. give antibiotics and anti yretic
b. give anti yretic and fluids
d. culture and sensitivity
the correct answer 6
KKKKKKKKKKKKKKKKKKKKKKKKKKKKKKKKKKKKKKK
KKKKKKKKKKKKKKKKK..
)3>) child with low grade fever and congested throat5 negative 'S( and
ositive 4"@. he has
a. infectous mononucleosis
b. *%!I
the correct answer is a
KKKKKKKKKKKKKKKKKKKKKKKKKKKKKKKKKKKKKKK
KKKKKKKKKKKKKKKKK..
)3C) the most common cause of intracerebral or intraarynchimal bleeding
is.
a. Fyertensive angioathy
b. annurysm
c. '@ malformation
the correct answer is a
)13) young adult Sickle cell atients are commonly affected with
a. dementia
b. multile cerebral infarcts
the correct answer b
KKKKKKKKKKKKKKKKKKKKKKKKKKKKKKKKKKKKKKK
KKKKKKKKKKKKKKKKK..
)11) >3 years old living in nursing home for the last ) months. his wife died
: months ago and he had a cornary artery disease in the last month. he
is now forgetful esecially of short term memory and decrease eye
contact with and loss of interest. dD
a. alGihiemer
b. deression
c. hyothyroidism
the correct answer b
KKKKKKKKKKKKKKKKKKKKKKKKKKKKKKKKKKKKKKK
KKKKKKKKKKKKKKKKK..
)12) thyroid cancer can be from
a. hyothyroidism
b. graves disease
c. toDic nodule
OOOOOOOOO
KKKKKKKKKKKKKKKKKKKKKKKKKKKKKKKKKKKKKKK
KKKKKKKKKKKKKKKKK..
)1)) a mother came with her son who is = years old very active never
sitting in class and with oor concentration. ur management would be.
a. olanGiine
b. amitalin
c. aloDane
the correct answer is b
this case is Fattention Kdeficit hyeractivity disorderF
treatment!
0- 3eth"lphenidate -ritalin
1- .e$troamphetamine
I- %derall
R- %tomo$etine
Q- Pemoline
P- %ntidepressant ( SSRI )
)12) from the grah above which of the following statement is true
a. osteorosis affecting ma6ority of women above >3
b. 239 of women under =3 are affected
OOOOOOOOO
KKKKKKKKKKKKKKKKKKKKKKKKKKKKKKKKKKKKKKK
KKKKKKKKKKKKKKKKK..
)15) a atient resented with rogressive weakness on swollowibg with
diloia and fatigability. the most likely underlying cause of her
disease is.
a. antibody against acetylcholine recetors
KKKKKKKKKKKKKKKKKKKKKKKKKKKKKKKKKKKKKKK
KKKKKKKKKKKKKKKKK..
)1:) an :C year old non diabetic. with mild hyertension and no hD of
+oronary heart desease. the best drug in treatment is.
a. thiaGides
b. '+4I
c. '%"
d. ++"
the correct answer a
KKKKKKKKKKKKKKKKKKKKKKKKKKKKKKKKKKKKKKK
KKKKKKKKKKKKKKKKK..
)1=) which of the following anti hyertensive is contraindicated for an
uncontrolled diabetic atient
a. hydrochlorothiaGide
b. Bosartan
c. hydralsGine
d. sironolactone
the correct answer is a
-side effect of thia7ide ! h"perl"cemia
KKKKKKKKKKKKKKKKKKKKKKKKKKKKKKKKKKKKKKK
KKKKKKKKKKKKKKKKK..
)1>) a wound stays in itSs rimary inflammation untill
a. 4scher formation
b. eitheliaGation
c. after 22 hours
d. wound cleaning
the correct answer d
KKKKKKKKKKKKKKKKKKKKKKKKKKKKKKKKKKKKKKK
KKKKKKKKKKKKKKKKK..
)1C) =3 years old male atient with mild urinary driing and hesitency ur
/D is mild "-F. ur neDt ste in management is
a. transurethral retrograde rostatectomy
b. start on medication
c. oen rostatectomy
the correct answer b
)23) most common cause of secondary F!, is
a. %enal disease # %enal arenchymal disease
KKKKKKKKKKKKKKKKKKKKKKKKKKKKKKKKKKKKKKK
KKKKKKKKKKKKKKKKK..
)21) one of the following food is known to reduce cancer
a. fibers
KKKKKKKKKKKKKKKKKKKKKKKKKKKKKKKKKKKKKKK
KKKKKKKKKKKKKKKKK..
)22) smoking is a definitive risk of
a. sT.c.ca of bladder
b. liver
c. breast
the correct answer is a

KKKKKKKKKKKKKKKKKKKKKKKKKKKKKKKKKKKKKKK
KKKKKKKKKKKKKKKKK..
)2)) eDercise recommended for atients with +'/. is
a. isometric
b. isotonic
c. yoga
a
KKKKKKKKKKKKKKKKKKKKKKKKKKKKKKKKKKKKKKK
KKKKKKKKKKKKKKKKK..
)22)what is the definitive treatment of frostbites
a5 %ewarming
KKKKKKKKKKKKKKKKKKKKKKKKKKKKKKKKKKKKKKK
KKKKKKKKKKKKKKKKK..
)25) the most common cause of failure to thrive in ediatric is
a. malnutrition
KKKKKKKKKKKKKKKKKKKKKKKKKKKKKKKKKKKKKKK
KKKKKKKKKKKKKKKKK..
)2:) which of the following causes the highest maternal mortality in
regnancy
a. toDolasma
b. hyerbilirubenia
c. OOOO

haemorrhae* h"pertensive disorders* sepsis* abortion
I1D) !tt of frostbite
a. Immersion in water 23#25 +
b. /ebridement
c. Beave it at room temrature
The correct answer is a
KKKKKKKKKKKKKKKKKKKKKKKKKKKKKKKKKKKKKKK
KKKKKKKKKKKKKKKKK..
)2>) +ase about a child both %"S5 8"S are elevated so he has /&1$
whatSs the tye of FB'
a./%)
b./%2
c./%5
d./%:
e./%=
the correct answer is a
!yes of FB' :
/%2 : @oodCs pasture s"ndrome > multiple m"eloma
/%) : ..3 * S45 * > raveCs disease
/%2= : an#"losin spond"laitis > reiterCs s"ndrome
"51 : behceatCs disease
/11 : hashimotCs disease
KK.
KKKKKKKKKKKKKKKKKKKKKKKKKKKKKKKKKKKKKKK
KKKKKKKKKKKKKK..
)2C) /efinition of --@ .
-redictive ositive value ( --@ ) : roortion of eole with a ositive test
who have s a disease : a0 (a.b)
KKKKKKKKKKKKKKKKKKKKKKKKKKKKKKKKKKKKKKK
KKKKKKKKKKKKKKKKK..
))3) 7hatSs the most common case in -F+ centers
a.*!I
b.F!,
c. +oryGa(*%!I)
the correct answer is c
))1) all of the following will decrease t comliance eDcet:
a. involve t in the lan
b. make simlified regimen
c. give easy written instructions
d. make aointments fleDible
e. warn the atient about the danger of missing a ill
the correct answer is e
KKKKKKKKKKKKKKKKKKKKKKKKKKKKKKKKKKKKKKK
KKKKKKKKKKKKKKKKK..
))2) +ase about old diabetic atient who still have hyerglycemia desite
increase insulin dose$the roblem with insulin in obese atients is
a. -ost recetor resistance
KKKKKKKKKKKKKKKKKKKKKKKKKKKKKKKKKKKKKKK
KKKKKKKKKKKKKKKKK.
)))) +ase about a child with drooling5 fever5 barking cough in sitting
osition5 dD:
a. +rou
b. "roncholities
c. -neumonia
The correct answer is a
KKKKKKKKKKKKKKKKKKKKKKKKKKKKKKKKKKKKKKK
KKKKKKKKKKKKKKKKK..
))2) &other came to you after her son had hematoma under the nail dt
in6ury:
a. Send home with a ad on the head
b. Send home with acetaminohen
c. /o wedge resection
d. 4vacuate the hematoma
The correct answer is d
This case is ( subunual hematoma )
))5) 7hat vaccine uSll give to a S+/ ( sickle cell disease ) child
a.F"@
b. F.influenGa
c. neumococcal
d. both ' and "
e. all of the above
the correct answer is c
KKKKKKKKKKKKKKKKKKKKKKKKKKKKKKKKKKKKKKK
KKKKKKKKKKKKKKKKK..
IIP) a mother with F"s'g ositive came with her child : yrs old who
has F"s'g .ve what will you give him:
a. oral olio5 /!-5 &&%
the correct answer is a
other choices all had /6V vaccine which uCll not ive
KKKKKKKKKKKKKKKKKKKKKKKKKKKKKKKKKKKKKKK
KKKKKKKKKKKKKKKKK..
))=) -atient wil BBA ain5 vomiting5 fever5 high 7"+ (1=.333)5 tenderness
and rebound tenderness
a. /iverticulitis
b. Sigmoid volvulus
c. 'endicitis
d. !oDic enteritis
The correct answer is a

KKKKKKKKKKKKKKKKKKKKKKKKKKKKKKKKKKKKKKK
KKKKKKKKKKKKKKKKK..
))>) 7hatSs the organism resonsible for suedomembranous colitis:
a. -seudomonas
b. +olisteridum
c. 4.coli
d. 4nterococcus fecalis
The correct answer is b
))C) &other came with her child who had botillism5 what you will advice
her:
a. ,ever eat canned food again
b. Store canned food at home
c. "oil canned food for 23#53 min
d. +heck eDiry date of canned food
The correct answer is d
KKKKKKKKKKKKKKKKKKKKKKKKKKKKKKKKKKKKKKK
KKKKKKKKKKKKKKKKK..
)23) (ld t resented with abdominal ain5 back ain5 ulsatile abdomen
whatSs the ste to confirm dD: this is a case of aortic aneurysm
a. 'bdominal *S
b. 'bdominal +!
c. 'bdominal &%I
The correct answer is a b
-initial investiation BS - +&N8IR3 b" +T

KKKKKKKKKKKKKKKKKKKKKKKKKKKKKKKKKKKKKKK
KKKKKKKKKKKKKKKKK..
)21) 1> months old came with bite by her brother5 what you will do:
a. Eive augminten
b. Eive titunus toDoid
c. Suture
T/5 +&RR5+T %NSE5R IS %
KKKKKKKKKKKKKKKKKKKKKKKKKKKKKKKKKKKKKKK
KKKKKKKKKKKKKKKKK..
)22) 1C yrs old after bike accident5 he cant bring the soon infront of
himself to eat5 lesion is in:
a. !emoral lobe
b. +erebellum
c. -arietal lobe
d. (cciital lobe
The correct answer is b
)2)) Fow to dD /@!:
a. +ontrast venograhy
b. /uleD *S
The correct answer is b
KKKKKKKKKKKKKKKKKKKKKKKKKKKKKKKKKKKKKKK
KKKKKKKKKKKKKKKKK..
)22) -t came after %!'5 E+S 125 near comlete amutation of the arm5 1
st
ste:
a. Secure air way
b. !ourniTuet on the arm
The correct answer is a
KKKKKKKKKKKKKKKKKKKKKKKKKKKKKKKKKKKKKKK
KKKKKKKKKKKKKKKKK..
)25) -t with hD of rolonged heavey bleeding 2 hrs ost artum5 you will
give:
a. %ingers lactate
b. ,S
c. ,S. acked erythrocytes
The correct answer is c
KKKKKKKKKKKKKKKKKKKKKKKKKKKKKKKKKKKKKKK
KKKKKKKKKKKKKKKKK..
)2:) 7hat is secial about lacenta abrution:
a. 'bnormal uterine contractions
b. -@ bleeding
c. 8etal distress
The correct answer is b * P% ! dar# * painful vainal bleedin and complication
is fetal h"po$ia
KKKKKKKKKKKKKKKKKKKKKKKKKKKKKKKKKKKKKKK
KKKKKKKKKKKKKKKKK..
)2=) -t )2 wks5 hD of -@ bleeding for many hrs5 dark blood5 abdominal
tenderness5 8F% 1235 uterine contractions every ) min5 /D:
a. 'brution lacenta
)2>) (ne of the stes in managing eistaDis:
a. -acking the nose
b. -ress the fleshy arts of nostrils
c. -ut atient of lateral lying osition
The correct answer is b
KKKKKKKKKKKKKKKKKKKKKKKKKKKKKKKKKKKKKKK
KKKKKKKKKKKKKKKKK..
)2C) -t with hD of diarrhea5 abdominal ain5 agitation5 headache5
diGGiness5 weakness5 ulstile thyroid5 unsteady gate. 4Damination was
normal. /D:
a. Fyochondriasis
b. SomatiGation disorder
c. !hyroid +a
d. 'nDiety
The correct answer is b
KKKKKKKKKKKKKKKKKKKKKKKKKKKKKKKKKKKKKKK
KKKKKKKKKKKKKKKKK..
)53) +hild with headache that increase by changing head ostion5
unilateral5 hotohobia
a. Sinus headache
b. &igraine
c. +luster headache
d. !ension headache
The correct answer is b
KKKKKKKKKKKKKKKKKKKKKKKKKKKKKKKKKKKKKKK
KKKKKKKKKKKKKKKKK..
)51) !tt of isolated fracture of femur (reeated) OOO
KKKKKKKKKKKKKKKKKKKKKKKKKKKKKKKKKKKKKKK
KKKKKKKKKKKKKKKKKK
)52) Mounge t with ain in BB after running ) kms5 more at night5
swelling5 H% was ,B
a. Stress fracture
)5)) F!, t5 with decrease vision5 fundal eDam showed increase cuing
of otic disc dD:
a. (en angle glaucoma
b. +losed angle glaucoma
c. +ataract
d. F!, changes
The correct answer is a
KKKKKKKKKKKKKKKKKKKKKKKKKKKKKKKKKKKKKKK
KKKKKKKKKKKKKKKKK..
)52) -t with unilateral red eye5 tearing5 clear tears no swelling or
discharge:
a. Eive toical antihistamin
b. Eive toical '"
c. Eive systemic '"
d. %eassurance
The correct answer is d 6e7 this is the viral conHectivitis
KKKKKKKKKKKKKKKKKKKKKKKKKKKKKKKKKKKKKKK
KKKKKKKKKKKKKKKKK..
)55) "usiness man went to -akistan5 came with bloody diarrhea5 stool
eDamination showed trohoGoite with %"+ inclusion5 /D:
a. 'mebic desyntry (entamoeba histlolytica )
KKKKKKKKKKKKKKKKKKKKKKKKKKKKKKKKKKKKKKK
KKKKKKKKKKKKKKKKK..
)5:) 5 yrs old child with abdominal ain after 2 wks of *%!I5 F" >5 retics
129 7"+ ,B eriheral blood smear showed target cells5 %"+ inclusions
dD:
a. S+' (the only hemolytic anemia in the answers)
OOOOO
KKKKKKKKKKKKKKKKKKKKKKKKKKKKKKKKKKKKKKK
KKKKKKKKKKKKKKKKK..
)5=) +hild had hb electrohoresis showed hb 29
a. F"' 23 F"S )5 F"8 5 F"'2
b. Sickle cell trait
c. !halacemia ma6or
d. &inor
OOOOO
)5>) Mounge t with hD of cough5 chest ain5 fever +H% showed %! lower
lobe infiltrate:
a. 'moDicillin
b. +eferuDim
c. 4mienim
d. +irofloDacin
OOOOOO
The correct answer is d
)5C) "est thing to reduce mortality rate in +(-/:
a. Fome (2 theray
b. 4naliril
c. Sto smoking
The correct answer is c
KKKKKKKKKKKKKKKKKKKKKKKKKKKKKKKKKKKKKKK
KKKKKKKKKKKKKKKKK..
):3) /rug that will delay need of surgery in '%:
a. digoDin
b. veraamil
c. nefidiin
d. enalril
the correct answer is d
KKKKKKKKKKKKKKKKKKKKKKKKKKKKKKKKKKKKKKK
KKKKKKKKKKKKKKKKK..
):1) +hild with skin rash5 ericarditis5 arthritis dD:
Nawasaki
KKKKKKKKKKKKKKKKKKKKKKKKKKKKKKKKKKKKKKK
KKKKKKKKKKKKKKKKK..
):2) -ic of skin with urle flat toed olygonal aules5 dD:
a. Bichen lanus
KKKKKKKKKKKKKKKKKKKKKKKKKKKKKKKKKKKKKKK
KKKKKKKKKKKKKKKKK..
IPI) +hild with atoic dermatitis5 what you will give other than cortisone
a. !here were many drugs5 steroids and 'ntibiotics and only one strange
name maybe moisturiGer (4mollients ). !hatSs the answer
):2) ' man had increase shoe siGe and 6aw5 the resonsible is:
a. '+!F
b. Somatostatin
c. !SF
d. +ortisone
The correct answer is b
KKKKKKKKKKKKKKKKKKKKKKKKKKKKKKKKKKKKKKK
KKKKKKKKKKKKKKKKK..
):5) !he best advice to atient travelling is:
a. "oiled water
b. Ice
c. 7ater
d. Salad and under cooked sea shells
The correct answer is a
KKKKKKKKKKKKKKKKKKKKKKKKKKKKKKKKKKKKKKK
KKKKKKKKKKKKKKKKK..
)::) Seldinfil is contraindicated with:
a. ,itrate
b. &ethyldoa
c. Eabaentine
The correct answer is a
KKKKKKKKKKKKKKKKKKKKKKKKKKKKKKKKKKKKKKK
KKKKKKKKKKKKKKKKK..
):=) -t with !"5 had ocular toDicity symtoms5 the drug resonsible is:
a. I,F
b. 4thambutol
c. %ifamicin
d. Stretomycin
The correct answer is b
I,F ! peripheral neuritis and hepatitis . so add ( 6P )
4thambutol ! optic neuritis
%ifamicine ! bod" fluid orane
):>) 'nother t treated for !" started to develo numbness5 the vit
deficient is:
a. !hiamin
b. ,iacin
c. -yridoDine
d. @it +
T/5 correct answer is c

KKKKKKKKKKKKKKKKKKKKKKKKKKKKKKKKKKKKKKK
KKKKKKKKKKKKKKKKK..
):C) -t with bilateral breast disease5 dD:
a. -aget
b. -ailloma
c. &eduallary
d. Bobular
KKKKKKKKKKKKKKKKKKKKKKKKKKKKKKKKKKKKKKK
KKKKKKKKKKKKKKKKK..
)=3) +hild fell on her elbow and had abrasion5 now swelling is more5
tenderness5 redness5 swelling is demarcated (they gave dimensions) child
has fever. /D:
a. Eonoccal arthritis
b. Synovitis
c. +ellulitis of elbow
The correct answer is c
KKKKKKKKKKKKKKKKKKKKKKKKKKKKKKKKKKKKKKK
KKKKKKKKKKKKKKKKK..
)=1) 'bout burn
a. Mou will give 1R2 fluid in the 1
st
> hrs
b. 1R2 in the 1
st
> hrs
The correct answer is a
-arkland formula ! fluid in first 1R hours V R \ wt. in # \ <
6S%A1;;;de$terose for dial" mentinance
Q;< in first : hours and anotherQ;< in ne$t 0P hours
)=2) Mou r suosed to kee a child ,-( heSs 25 kgs5 how much you will
give:
a. 1)33
b. 1233
c. 1533
d. 1:33
The correct answer is d
8or the first 13 kg of weight* a child needs 0;; m4 per # of weiht.
8or the neDt 13 kg of weight (11#23 kg)* a child onl" needs Q; m4 per # of
weiht.
for anything over 23 kg (21 kg of weight and higher)* the child onl" needs 1;
m4 per # of weiht.
Ta#e a IQ # child* for e$ample. /e needs 0;;; m4 for his first 0; # of weiht
(0; # $ 0;; m4)* Q;; m4 for his second 0; # of weiht (0; # $ Q; m4)* and 1;
m4O# for an" weiht above 1; # (0Q # $ 1; m4). % IQ # child* therefore*
needs appro$imatel" 0:;;m4 of water or free li,uids.
KKKKKKKKKKKKKKKKKKKKKKKKKKKKKKKKKKKKKKK
KKKKKKKKKKKKKKKKK..
)=)) Moung atient with haryngitis5 inflammation of oral mucosa and
lis that has whitish cover and erythmatous base5 febrile5 slenomegaly. /D:
(this is infectious mono)
a. Scarlet fever
b. 4"@
c. FI@
The correct answer is b
KKKKKKKKKKKKKKKKKKKKKKKKKKKKKKKKKKKKKKK
KKKKKKKKKKKKKKKKK..
)=2) 8emale with greenish vaginal discharge5 red cerviD. /D:
!rchimoniosis
KKKKKKKKKKKKKKKKKKKKKKKKKKKKKKKKKKKKKKK
KKKKKKKKKKKKKKKKK..
)=5) 'nother female with malodorous discharge and ain maybe. /D:
"acterial vaginosis
KKKKKKKKKKKKKKKKKKKKKKKKKKKKKKKKKKKKKKK
KKKKKKKKKKKKKKKKK..
)=:) !he best way to reduce the weight in children is:
a. sto fat intake
b. /ecrease calories intake
c. /rink a lot of water
The correct answer is b
)==) Mou have to advice a teenager that mainly eats fast food to take:
a. 8olic acid and +a
b. @it + and +a
c. +a alone
The correct answer is a
KKKKKKKKKKKKKKKKKKKKKKKKKKKKKKKKKKKKKKK
KKKKKKKKKKKKKKKKK..
)=>) -atient has symtoms of infection5 desTuamation of hands and feet5
"- 1=3R113 dD:
a. Syhilis
b. !oDic shock syndrome
c. Scarlet fever
The correct answer is b
!oDic shock syndrome :
+aused b" S.aureus * often with Q da"s of onset of menestral period in
women who have used tampons. 8eature: abrupt fever ( IU c or more )*
vomitin* diffuse macular er"thematous rash* des,uamation especiall" in
palms and soles* nonpurulent conHunctivitis. /iagnosis ! blood culture are -ve
.so* dianosis b" clinical. !reatment ! 0
st
step reh"dration and antibiotic
KKKKKKKKKKKKKKKKKKKKKKKKKKKKKKKKKKKKKKK
KKKKKKKKKKKKKKKKK..
)=C) Snellin chart5 if atient can read u to line )5 how much is his vision
loss :
2 not complete!
but this pt. see letters at 1; feet * where normal person see it at D; feet.

)>3) I cant remember the case but gram .ve cocci were isolated5 dD is
stert bcG it was the only gm .ve cocci.
KKKKKKKKKKKKKKKKKKKKKKKKKKKKKKKKKKKKKKK
KKKKKKKKKKKKKKKKK..
)>1) /iabetic mother asking about risk of diabetes related congenital
defect. It will be in:
a. 1
st
trimester
b. 2
nd
trimester
c. )
rd
The correct answer is a
KKKKKKKKKKKKKKKKKKKKKKKKKKKKKKKKKKKKKKK
KKKKKKKKKKKKKKKKK..
)>2) If diabetic mother blood sugar is always high desite of insulin5
neonate comlication will mostly be:
a. &aternal hyerglycemia
b. &aternal hyoglycemia
c. ,eonatal hyoglycemia
d. ,eonatal hyerglycemia
The correct answer is c
KKKKKKKKKKKKKKKKKKKKKKKKKKKKKKKKKKKKKKK
KKKKKKKKKKKKKKKKK..
)>)) (bssive neurosis atients will have:
a. &a6or deression
b. Bake of insight
c. SchiGohrenia
The correct answer is a
KKKKKKKKKKKKKKKKKKKKKKKKKKKKKKKKKKKKKKK
KKKKKKKKKKKKKKKKK..
)>2) "efore giving biolar atient lithium you will do all of the following
eDcet:
a. !8!
b. B8!
c. %8!
d. -regnancy test
The correct answer is b
)>5) -atient came with -@ small bleeding (she fell down and felt decrease
in fetal movement but on eDamination baby was ok) all her ) revious
regnancies were normal5 you will do:
a. Immediate +S
b. +areful observation of the bleeding
c. &edication (strange name)
d. &g sulhate
2 not complete * depend on date of estation.
KKKKKKKKKKKKKKKKKKKKKKKKKKKKKKKKKKKKKKK
KKKKKKKKKKKKKKKKK..
)>:) -t came to you missing her eriod for = wks5 she had minimal
bleeding and abdominal ain5 .ve home regnancy test5 1
st
thing to order is:
a. "F+E
b. *S
c. /rugs maybe
The correct answer is a
KKKKKKKKKKKKKKKKKKKKKKKKKKKKKKKKKKKKKKK
KKKKKKKKKKKKKKKKK..
)>=) -atient came to you and you susect re eclamisa5 which of the
following will make it most likely:
a. 4levated blood ressure
b. /ecrease fetal movement
c. OO
The correct answer is a
KKKKKKKKKKKKKKKKKKKKKKKKKKKKKKKKKKKKKKK
KKKKKKKKKKKKKKKKK..
)>>) (ld atient male5 hematuria5 assing red clots and %! testicular
ain:
a. !esticular +a
b. %++ ( renal cell carcinoma )
c. +ystitis
The correct answer is b
)>C) to get more information from the t
a. oen end Tuestion
( avoid leadin ,uestion to et more information )
KKKKKKKKKKKKKKKKKKKKKKKKKKKKKKKKKKKKKKK
KKKKKKKKKKKKKKKKK..
)C3) gingivitis most likely cause
a.FS@
( , not complete but ! The most common cause of inivitis is poor oral
h"iene that encouraes pla,ue to form.)
KKKKKKKKKKKKKKKKKKKKKKKKKKKKKKKKKKKKKKK
KKKKKKKKKKKKKKKKK..
)C1) watery discharge from eye 5 con6unctivitis treated by
a. toical corticosteroid
(Its s"mptoms include water" dischare and variable itch. The infection
usuall" beins with one e"e* but ma" spread easil" to the other
Ttt! cold compresses
]
and artificial tears)
KKKKKKKKKKKKKKKKKKKKKKKKKKKKKKKKKKKKKKK
KKKKKKKKKKKKKKKKK..
)C2) blow out fracture eyelid swelling 5 redness other symtoms
a. resent air fluid level
b. enothalmos
The correct answer is b
its enophaloms * other name of blow out orbital floor fracture
KKKKKKKKKKKKKKKKKKKKKKKKKKKKKKKKKKKKKKK
KKKKKKKKKKKKKKKKK..
)C)) drinking of dirty water causes
a. hetitis '
b. "
c. +
d. /
The correct answer is a
KKKKKKKKKKKKKKKKKKKKKKKKKKKKKKKKKKKKKKK
KKKKKKKKKKKKKKKKK..
)C2) giemsa stained blood film
a. malaria

)C5) regarding eritonitis
a.+omlicated aendectomy the cause is anerobe organism
b. rigidity and the cause is aralytic ileus
c. can be caused by chemical erosions.
The correct answer is c
KKKKKKKKKKKKKKKKKKKKKKKKKKKKKKKKKKKKKKK
KKKKKKKKKKKKKKKKK..
)C:) fraucture of humerus associated with
a. radial , in6ury
b. median nerve in6ury
the correct answer is a ( radial roove )
KKKKKKKKKKKKKKKKKKKKKKKKKKKKKKKKKKKKKKK
KKKKKKKKKKKKKKKKK..
)C=) there is case i cant remember they asked about diagnosis of acute
lymhocytic leukemia 'BB
The total number of white blood cells ma" be decreased* normal* or
increased* but the number of red blood cells and platelets is almost alwa"s
decreased. In addition* ver" immature white blood cells (blasts) are present in
blood samples e$amined under a microscope.
% bone marrow biops"is almost alwa"s done to confirm the dianosis and to
distinuish %44 from other t"pes of leu#emia
KKKKKKKKKKKKKKKKKKKKKKKKKKKKKKKKKKKKKKK
KKKKKKKKKKKKKKKKK..
)C>) diagnosis of hemochromatosis
a. serum ferritinOZ
/emochromatosis is suested b" a persistentl" elevated transferrin
saturation in the absence of other causes of iron overload. It is the initial test
of choice .
8erritin concentration can be hih in other conditions* such as infections*
inflammations* and liver disease .
8erritin levels are less sensitive than transferrin saturation in screenin tests
for hemochromatosis .
)CC) kawasaki disease associated with
a.strawberry tongue
Nawasaki disease
3ultis"stem acute vasculitis that primar" affected "oun children. 8ever
plus four or more of the followin criteria for dianosis!
1-fever N R; + for at least five da"s
2-bilateral* none$udative* painless conHunctivitis
)-pol"morphous rash ( primaril" truncal )
2-cervical l"mphadenopath" ( often painful and unilateral )
5- diffuse mucous membrane er"thema ( strawberry tongue ) * dr" red
:- er"thema of palm and sole
=- other maindfestation ! allbladder h"drops* hepatitis* arthritis
Bntreated =awasa#i diease can lead to coronar" aneur"sms and even 3I
!reatment :
1#hih dos %S% ( for fever and inflammation) > IVI@ ( to prevent
aneurr"smal )
2-referral to pediatric cardioloist
KKKKKKKKKKKKKKKKKKKKKKKKKKKKKKKKKKKKKKK
KKKKKKKKKKKKKKKKK..
233) most common tumor in children
a.'BB
b. rabdomyosarcoma
c. wilmSs tumor

the correct answer is a in case cancer but if tumor its brain tumor.
#'BB : most common childhood tumor
#rabdomyosarcoma ! most common soft tissue tumor
#wilmSs tumor: most common intra- abdominal childhood tuomr
KKKKKKKKKKKKKKKKKKKKKKKKKKKKKKKKKKKKKKK
KKKKKKKKKKKKKKKKK..
231) &ost common intra# abdominal tumor in children:
a.wilmSs tumor
b. lymhoma

the correct answer is a
the most common intra-abdominal tumor in children are neruoblastoma
and wilmCs tumor.
232) t c0o of hyoigmentful skin 5 nerve thicking diagnosis
a.lerosy
Berosy : chronic ranulomatous infection caused b" 3"cobacterium leprae*
acid-fast bacillus preferentiall" affectin cooler reions of bod" ( e. s#in *
mucousmembrane* peripheral nerve) . dianosis b" s#in lesion
( h"popimentation or h"perpimentation macule ) and enlare nerves eith
sensor" loss.
KKKKKKKKKKKKKKKKKKKKKKKKKKKKKKKKKKKKKKK
KKKKKKKKKKKKKKKKK..
23)) regrading +(-/ to reduce comlication we should give
a. theohyline
b. neumococcal vaccine
c. smoking cessation.
The correcr answer is c
KKKKKKKKKKKKKKKKKKKKKKKKKKKKKKKKKKKKKKK
KKKKKKKKKKKKKKKKK..
232) seud#gout its
a. +'+()
b. +'+B)
The correct answer is a
@out ! deposition of ( 3SB3 ) monosodium urate monoh"drate -ve of
birefrinent * needle shape*
Psudoout ! deposition of ( cppd ) calcium p"rophosphates deh"drate
cr"stal * Ave birefrinent * rhomboid shape* ( +%+&I)
.
235) man with history of alcohol assocation with
a. high &+@
b. folic acid deficiency
c. "12 deficieny
e. heatitis
The correct answer is b
KKKKKKKKKKKKKKKKKKKKKKKKKKKKKKKKKKKKKKK
KKKKKKKKKKKKKKKKK..
23:) neanate C days on breast feed develo 6aundice
a. breast feed 6aundice
b. athological 6aundice
c. hysiological 6aundice
the correct answer is a
1#athological 6aundice ! conHuated ( direct ) bilirubin* start in 0
st
1R hours*
bilirubine N 0Q m O dl * persistent 0 wee# in term infant and 1 wee#s in preterm
infant
2#hysilogical 6aundice: unconHuated ( indirect ) bilirubine * start in 1- I da"s*
bilirubine T 0Q mOdl * persistent 0 wee# in term infant and 1 wee#s in preterm
infant.
)#breast feeding 6aundice ! unconHucated ( indirect ) bilirubine * start in D da"s
of life * persistent to one or more months.
KKKKKKKKKKKKKKKKKKKKKKKKKKKKKKKKKKKKKKK
KKKKKKKKKKKKKKKKK..
23=) lady c0o headche bandlike ain
a. tension headache
KKKKKKKKKKKKKKKKKKKKKKKKKKKKKKKKKKKKKKK
KKKKKKKKKKKKKKKKK..
23>) regarding breast screening
a. self breast eDamination early detection of tumor
b. mammogram not advise before )5 y
the correct answer is b
KKKKKKKKKKKKKKKKKKKKKKKKKKKKKKKKKKKKKKK
KKKKKKKKKKKKKKKKK..
23C) 1C yrs old c0o abdo ain within menesturation for last : years
diagnosis
a. rimary dysmenorrhea
b. secondary dysmenorrheal
The correct answer is a
Primar" d"smenorrhea !
&nset within P months after menarche . 4ower abdominalOpelvic pain beins
with onset of menses and lasts :-D1 hours
213) bilateral breast mass diagnosis
a. ductal carcinoma
b. agets disease
it could be Intraductal papilloma ??????
KKKKKKKKKKKKKKKKKKKKKKKKKKKKKKKKKKKKKKK
KKKKKKKKKKKKKKKKK..
211) beriberi cause of deficiency
a. @I! "1
b. @I! "2
c. @I! ")
The correct answer is a
KKKKKKKKKKKKKKKKKKKKKKKKKKKKKKKKKKKKKKK
KKKKKKKKKKKKKKKKK..
212) chronic uses of estrogen association OOOOO
KKKKKKKKKKKKKKKKKKKKKKKKKKKKKKKKKKKKKKK
KKKKKKKKKKKKKKKKK..
21)) 'systole
a. adrenalin
b. atroine
The correct answer is a
as"stol has onl" I durs epinephrine-vasopressine-atropin
KKKKKKKKKKKKKKKKKKKKKKKKKKKKKKKKKKKKKKK
KKKKKKKKKKKKKKKKK..
212) t has diarrhea 5 dermatitis and dementia diagnosis
a. pellara due to niacin diffecienc" (vit 6I)
KKKKKKKKKKKKKKKKKKKKKKKKKKKKKKKKKKKKKKK
KKKKKKKKKKKKKKKKK..
215) regarding in6ectable rogestron
a . an cause skin rob OZZ
InHected proesterone ( medro$"-proestrone ) ! advantaes ! liht or no
periods* safe with breastfeedin* I3 inHection ever" I months. .isadvantaes!
irreular bleedin* wt. ain* decrease bone mineral densit" ( reversible ) *
dela" fertilit" discontinue .
KKKKKKKKKKKKKKKKKKKKKKKKKKKKKKKKKKKKKKK
KKKKKKKKKKKKKKKKK..
21:) sencodary revention is
a. coronary byass gaft
Secondar" prevention enerall" consists of the identification and interdiction
of diseases that are present in the bod"* but that have not proressed to the
point of causin sins* s"mptoms* and d"s-function
21=) 22 yrs old c0o insomina0 slee disturb treatment
a. SS%I(traGodone)
Short histor" *
6ut the initial manaement of insomnia ! ood sleep h"iene .
KKKKKKKKKKKKKKKKKKKKKKKKKKKKKKKKKKKKKKK
KKKKKKKKKKKKKKKKK..
21>) celluitis in children most commin causes
a. grou ' stetoccus
In children* the most common cause of cellulitis is - aureus or roup %
streptococcus.

KKKKKKKKKKKKKKKKKKKKKKKKKKKKKKKKKKKKKKK
KKKKKKKKKKKKKKKKK..
21C) antiderssants associated with hyertensive crisis treatment
a. SS%I
b. &('Is
c. !+'s
the correct answer is b
esp h"dra7ines ( phenel7ine )
KKKKKKKKKKKKKKKKKKKKKKKKKKKKKKKKKKKKKKK
KKKKKKKKKKKKKKKKK..
223) t his &"I X 22 kg he is
a. noramal weight
b. over weight
c. morbid weight
d. mild weight
The correct answer is a
KKKKKKKKKKKKKKKKKKKKKKKKKKKKKKKKKKKKKKK
KKKKKKKKKKKKKKKKK..
221) baby c0o fever 5 chills 5 rigors and head rigidity .ve kurnings sign rash
on his lower limb diagnosis :
# meningoccal meninigits
OOOOOOO
KKKKKKKKKKKKKKKKKKKKKKKKKKKKKKKKKKKKKKK
KKKKKKKKKKKKKKKKK..
222) 2> yrs t with abdo ain 5 neusea5 vominting tenderness in right
hyochondrial
# acute cholecysitis
8ever OZ
22)) 2C t c0o dysurea his microscoic showed E #ve organism is
a. legonealla
b. 4. coli
the correct answer is b
KKKKKKKKKKKKKKKKKKKKKKKKKKKKKKKKKKKKKKK
KKKKKKKKKKKKKKKKK..
222) )3 yrs t c0o feeling heaviness in the lower abdomen having ulge
aable at the to scrotum that was reducible and icreasing in
valsalva maneuver diagnosis
a. hydrocele
b. variocele
c. indirect inguinal hernia
d. direct inginal hernia
The correct answer is c
KKKKKKKKKKKKKKKKKKKKKKKKKKKKKKKKKKKKKKK
KKKKKKKKKKKKKKKKK..
225) anticoagulation rescrib for
a. one month
b. : months
c. : weeks
d. one year
The correct amswer is b
KKKKKKKKKKKKKKKKKKKKKKKKKKKKKKKKKKKKKKK
KKKKKKKKKKKKKKKKK..
22:) cushing syndrome best single test to confirm
a. alsma cortison
b. '!+F
c. /eDamethasone Suression !est
The correct answer is c
KKKKKKKKKKKKKKKKKKKKKKKKKKKKKKKKKKKKKKK
KKKKKKKKKKKKKKKKK..
22=) 2) yrs old history of *%!I then he develoed ecchomysis best treated
a. local '"
b. local antiviral
c. steroid
The correct answer is c dianosis is ITP
22>) chronic sycotic disorder manged by
a. haloperidol
KKKKKKKKKKKKKKKKKKKKKKKKKKKKKKKKKKKKKKK
KKKKKKKKKKKKKKKKK..
22C) 2C yrs old lady "#FSE 1:3 c0o vomiting 5 abdomenal ain which is
more accurate to diagnosis
a. "F+E serial
b. elvic *S
c. larascoy
this case is molar and chooce is a .
KKKKKKKKKKKKKKKKKKKKKKKKKKKKKKKKKKKKKKK
KKKKKKKKKKKKKKKKK..
2)3) =3 yrs old man c0o fever 5 vesicular rash over forehead mangement
a. I@ '"
b. I@ antiviral
c. oral acyclovir
the correct answer is c
KKKKKKKKKKKKKKKKKKKKKKKKKKKKKKKKKKKKKKK
KKKKKKKKKKKKKKKKK..
2)1) celiac disease involves :
a. roDimal art of small intestin
b. distal art of small intestin
c. roDimal art of large intesin
d. distal art of large intensin
The correct answer is a
KKKKKKKKKKKKKKKKKKKKKKKKKKKKKKKKKKKKKKK
KKKKKKKKKKKKKKKKK..
2)2) : yes old t cyanosis ast history of similar attack : month ago u will
do for him
a. +D%
b. -8!
c. secure airway
d. +"+
2 is not clear or not complete * if pt. came with life threatenin c"anosis donCt
waste time with investiation and start with %6+ ( scure airwa" ) * but if pt.
came with past histor" c"anosis choose b .

2))) side affect of diaGeam
Sedation * depence* respirator" supression * anterorade amnesia *
confusion (especiall" pronounced in hiher doses) and sedation
KKKKKKKKKKKKKKKKKKKKKKKKKKKKKKKKKKKKKKK
KKKKKKKKKKKKKKKKK..
2)2) endemic means:
4ndemic !is the constant presence of a disease or infectious aent in a
certain eoraphic area or population roup. ( usuall" rate of disease )
e.malaria
4idemic : is the rapid spread of a disease in a specific area or amon a
certain population roup. ( e$cessive rate of disease ) e.menitis
-andemic : is a worldwide epidemic; an epidemic occurring over a wide
geographic area and affecting a large number of people.eg.influnza
KKKKKKKKKKKKKKKKKKKKKKKKKKKKKKKKKKKKKKK
KKKKKKKKKKKKKKKKK..
2)5) 4idemic curve :
a raph in which the number of new cases of a disease is plotted aainst an
interval of time to describe a specific epidemic or outbrea# .
2):) -t with hodgkinSs lymhoma 5 and red strunberg cell in athology and
there is esinohil lymhocyte in blood so athological classification
is:
a. &iDed#cellularity subtye
b. nodular sclerosis subtye of FodgkinSs lymhoma
the correct answer is b
+lassical /od#inCs l"mphoma can be subclassified into R patholoic
subt"pes based upon Reed-Sternber cell morpholo" and the composition
of the reactive cell infiltrate seen in the l"mph node biops" specimen (the
cell composition around the Reed-Sternber cell(s))
.
,ame descretion
Nodular sclerosin +/4 Is the most common subt"pe and is composed of lare tumor nod%les showin
scattered lacunar classical RS cells set in a bac#round of reactive l"mphoc"tes*
eosinophils and plasma cells with var"in derees of collaen fibrosisOsclerosis.
3i$ed-cellularit" subt"pe Is a common subt"pe and is composed of numerous classic RS cells admi$ed
with numerous inflammator" cells includin l"mphoc"tes* histioc"tes* eosinophils*
and plasma cells. without sclerosis. This t"pe is most often associated with 56V
infection and ma" be confused with the earl"* so-called CcellularC phase of nodular
sclerosin +/4
4"mphoc"te-rich or
4"mphoc"tic
predominance
Is a rare subt"pe* show man" features which ma" cause dianostic confusion
with nodular l"mphoc"te predominant 6-cell Non-/od#inCs 4"mphoma (6-N/4).
This form also has the most favorable pronosis
4"mphoc"te depleted Is a rare subt"pe* composed of lare numbers of often pleomorphic RS cells with
onl" few reactive l"mphoc"tes which ma" easil" be confused with diffuse lare
cell l"mphoma. 3an" cases previousl" classified within this cateor" would now
be reclassified under anaplastic lare cell l"mphoma

2)=) :2 y male with /@! and I@+ obstruction due to thrombosis so most
like dd is
a. nehrotic syndrome
b. SB4
+. +hirstm disease
/. OZ
KKKKKKKKKKKKKKKKKKKKKKKKKKKKKKKKKKKKKKK
KKKKKKKKKKKKKKKKK..
2)>) -t with bdominal ain heamatutea 5 F!,5 and have abnormalty in
chromosm 1: 5 diagnosis is
a. -(BM +M+!I+ NI/,4M
KKKKKKKKKKKKKKKKKKKKKKKKKKKKKKKKKKKKKKK
KKKKKKKKKKKKKKKKK..
2)C) 1= year t with dysnea -o2 5 -+(2 5Hray normal -F increase so dd is:
a. acute attack of asthma
b. - 4
c. neumonia
d. nemothraD
the correct answer is a
KKKKKKKKKKKKKKKKKKKKKKKKKKKKKKKKKKKKKKK
KKKKKKKKKKKKKKKKK..
223) ' long scenario about atient with olydisia ad olyuria. Serum
osmolrity high . desmorsin inductin no change urine omolarity and
lasma osmolrity so dd is
a. nhrognic tye
b. central tue
The correct answer is a
KKKKKKKKKKKKKKKKKKKKKKKKKKKKKKKKKKKKKKK
KKKKKKKKKKKKKKKKK..
221) 53 year old &an resented to 4% with sudden headach5 blurred of
vision and eye ain. !he diagnosis is:
a. 'cute closure glaucoma
b. 'cute con6unctivitis
c. +orneal ulcer
d. #####
The correct answer is a
222) %!' with hi dislocation and shock so causes of shock is
a. blood lose
b. urtheral in6ery
c. neurogenic
The correct answer is c
KKKKKKKKKKKKKKKKKKKKKKKKKKKKKKKKKKKKKKK
KKKKKKKKKKKKKKKKK..
22)) most common causes of hand infection
a. truma
b. imunocrombromise
c. $$$
the correct answer is a
KKKKKKKKKKKKKKKKKKKKKKKKKKKKKKKKKKKKKKK
KKKKKKKKKKKKKKKKK..
222) ttt of cholestatoma is
a. antibiotic
b. steroid
c. surgery
d. Erommet tube
the correct answer is c
-+holesteatoma is a destructive and e$pandin rowth consistin of
#eratini7in s,uamous epithelium in the middle ear andOor mastoid .
treatment ! Surer" is performed to remove the sac of s,uamous debris and
a mastoidectom" is performed.

225) twins one male and other female . his father notice that femle become
uberty before male so what you say to father
a. female enter uberty 1#2 year before male
b. female enter uberty 2#) year before male
c. female enter uberty at the same age male
the correct answer is b
KKKKKKKKKKKKKKKKKKKKKKKKKKKKKKKKKKKKKKK
KKKKKKKKKKKKKKKKK..
22:) in devoling countery to revent dental carise 5 it add to water
a. florid
b. Gink
c. coer
d. iodide
the correct answer is a
KKKKKKKKKKKKKKKKKKKKKKKKKKKKKKKKKKKKKKK
KKKKKKKKKKKKKKKKK..
22=) 12 y.o boy c0o abdominal ain after laying football5he denied any h0o
trauma 5the ain is in the Bt araumbilical region what inD you want to
do:
a. +H%
b. ultrasound kidney
The correct answer is b
KKKKKKKKKKKKKKKKKKKKKKKKKKKKKKKKKKKKKKK
KKKKKKKKKKKKKKKKK..
22>) t child with back ain that wake t from slee
So diagnosis
a. lumber kyhosis
b. osteoarthritis
c. %'
d. Scoliosis(ainless)
the correct answer is c or a???
22C) child with aule vesical on oroharnD and rash in alm and hand so
dd:
a. +&@
b. 4"@
c. &4'SBS
d. %("4BB'
???? +o$sac#ie % virus. @esical in buccal mucusa is measls.
KKKKKKKKKKKKKKKKKKKKKKKKKKKKKKKKKKKKKKK
KKKKKKKKKKKKKKKKK..
253) chid with dental caries and history of bottle feading
So dd
a. nurse milk caris
KKKKKKKKKKKKKKKKKKKKKKKKKKKKKKKKKKKKKKK
KKKKKKKKKKKKKKKKK..
251) EeneraliGe antDity disorder best ttt: #1C
a. SS%Is
b. tricyclic a d
d. &'(I
The correct answer is a

KKKKKKKKKKKKKKKKKKKKKKKKKKKKKKKKKKKKKKK
KKKKKKKKKKKKKKKKK..
252) &a6or deression management:
a. Intial &(,(!F4%'"M even sever sever derisson
b. !tt should be change if no resonse during 2wk ('! B4'S! : .
744NS )
OOOO
KKKKKKKKKKKKKKKKKKKKKKKKKKKKKKKKKKKKKKK
KKKKKKKKKKKKKKKKK..
25)) sychotheray5 medication5 and electroconvulsive thera
-sychiatric t with un comliance of drugs ttt:
a. dero haloredol in6ection
b. oral colonaGeam
the correct answer is a
252) mild diarrhea mangement :
a. oral antibiotis
b. I@8
c. (%S
the correct answer is c
KKKKKKKKKKKKKKKKKKKKKKKKKKKKKKKKKKKKKKK
KKKKKKKKKKKKKKKKK..
255) salingitis and -I/ on enicillin but not imrove the most likely
organism is :
a. chlamydia
b. nessiria
c. SM-FBIS
d. FS@
the correct answer b
KKKKKKKKKKKKKKKKKKKKKKKKKKKKKKKKKKKKKKK
KKKKKKKKKKKKKKKKK..
25:) *sed for treatment of seudomembranous colitis:
P& metronida7ole is the empiric treatment of choice (Q;; m TI. for 0;-
0R da"s). 8or severe infections* oral Vancom"cin is the treatment of
choice (01Q m 2I. 0;-0R da"s).
KKKKKKKKKKKKKKKKKKKKKKKKKKKKKKKKKKKKKKK
KKKKKKKKKKKKKKKKK..
25=) Bady with of right hyochondrial ain5 fever and slight 6aundice. 7hat
is your diagnosisO
a. 'cute cholecystitis.
2 not complete.
3ost common presentin s"mptom of acute cholec"stitis is upper
abdominal pain often radiatin to riht scapula. Nausea and vomitin
enerall" present. Patients ma" report fever.
25>) /ee 6aundice with alable gallbladder O
i . a. +ancer head of ancreas .

Pancreatic cancer presents with abdominal pain radiatin toward the
bac#* as well as with
Haundice* loss of appetite* nausea* vomitin* weiht loss* wea#ness*
fatiue* and indiestion.
5$amination! ma" reveal a palpable* nontender allbladder
(+ourvoisier?s sign) or mirator" thrombophlebitis (!rousseau?s sign).
.ianose b" +T. treatment usuall" palliative (due to metastasis at .$).
KKKKKKKKKKKKKKKKKKKKKKKKKKKKKKKKKKKKKKK
KKKKKKKKKKKKKKKKK..
25C) -atient with erforated gallbladder underwent cholecystectomy.
%eturn to you with fever. (n abdominal H#ray5 there is elevation of
right hemidaiahragm. 7hat is the ossible diagnosisO
a. Subhrenic abcess
KKKKKKKKKKKKKKKKKKKKKKKKKKKKKKKKKKKKKK
KKKKKKKKKKKKKKKKKK..
2:3 ( 2:3 ( -atient with acute erianal ain since 2 days with black
mass 2L) ain 2increase with defecation %D :
a. 4vacuation under local anesthesia
A not comlete
KKKKKKKKKKKKKKKKKKKKKKKKKKKKKKKKKKKKKKK
KKKKKKKKKKKKKKKKK..
2:1) %humatoid arthritis distal interhalengial nodules
a. FebrerdenSs nodes
FeberdenSs nodes are at the .IP while "ouchardSs nodes are at PIP.

2:2) 4lderly atient with %BA fullness5 weight loss5 changed bowel habit5
anemic and ale. 7hat is the investigation of choiceO
a. +olonoscoy.
This is now the investiation of choice if colorectal cancer is suspected
provided the patient is fit enouh to undero the bowel preparation. It has the
advantae of not onl" pic#in up a primar" cancer but also havin the abilit"
to detect s"nchronous pol"ps or even multiple carcinomas* which occur in
Q< of cases.
KKKKKKKKKKKKKKKKKKKKKKKKKKKKKKKKKKKKKK
KKKKKKKKKKKKKKKKKK..
2:)) Investigation of choice in Iron deficiency anemiaO
a. serum iron
b. ferritin
c. !I"+
the correct answer is b
Serum ferritin is the most sensitive lab test for iron deficienc" anemia.
KKKKKKKKKKKKKKKKKKKKKKKKKKKKKKKKKKKKKK
KKKKKKKKKKKKKKKKKK..
2:2) In atients with hyertension and diabetes5 which antihyertensive
agent you want to add firstO
a. [#blockers
b. '+4 inhibitor
c. Y#blocker
d. +alcium channel blocker
the correct answer is b
.iuretics (ine$pensive and particularl" effective in %frican-%mericans) and
^-bloc#ers (beneficial for patients with +%.) have been shown to reduce
mortalit" in uncomplicated h"pertension. The" are first-line aents unless a
comorbid condition re,uires another medication. (see table)
Population Treatment
/iabetes with
roteinuria
%+5Is.
+F8 ^-bloc#ers* %+5Is* diuretics (includin
spironolactone).
Isolated systolic
hyertension
.iuretics preferred9 lon-actin dih"drop"ridine
calcium channel bloc#ers.
&I ^-bloc#ers without intrinsic s"mpathomimetic
activit"* %+5Is.
(steoorosis Thia7ide diuretics.
"-F S-antaonists.
2:5) -atient resented with retinal artery occlusion5 which of the following
is wrong:
a. ainful loss of vision
b. ainless loss of vision
the correct answer is a
The most common presentin complaint of retinal arter" occlusion is an
acute persistent painless loss of vision.
KKKKKKKKKKKKKKKKKKKKKKKKKKKKKKKKKKKKKK
KKKKKKKKKKKKKKKKKK..
2::) 4+E finding of acute ericarditisO
a. S! segment elevation in all leads
+lassic 5+@ findins in pericarditis! 4ow-voltae* diffuse ST-sement
elevation.
KKKKKKKKKKKKKKKKKKKKKKKKKKKKKKKKKKKKKK
KKKKKKKKKKKKKKKKKK..
2:=) 4idemiological study want to see the effect of smoking in the
bronchogenic carcinoma5 they saw that is C39 of smokers has
bronchogenic carcinoma.
)39 of nonKsmokers has the disease5 the secificity of the disease as
a risk factor is:
a. =3 9
b. )3 9
c. C3 9
&+As not comlete
Sensitivity: The probabilit" that a diseased patient will have a positive test result.
Secificity: The probabilit" that a nondiseased person will have a neative test
result.
/isease
-resent
,o /isease
-ositive
test
a b
,egative
test
c d
Sensitivity X a 0 (a . c) Secificity X d 0 (b . d)
,ote:
- % sensitive test is ood for rulin out a disease.
- /ih sensitivit" V ood screenin test ( false neatives).
- /ih specificit" V ood for rulin in a disease (ood confirmator" test).
2:>) "aby resent with ain in the ear5 by eDamination there is iece of a
glass dee in his ear canal5 the mother mention a history of a broken
glass in the kitchen but she clean that comletely. 7e treat that by:
a. "y alying a stream of solution to syringing the ear.
b. %emove it by forces.
c. %efer her to otolaryngology
L 8oreign bodies in the ear canal:
Techni,ues appropriate for the removal of ear forein bodies include
mechanical e$traction* irriation* and suction. Irriation is contraindicated for
oranic matter that ma" swell and enlare within the auditor" canal (e.
seeds). Insects* oranic matter* and obHects with the potential to become
friable and brea# into smaller evasive pieces are often better e$tracted with
suction than with forceps. 4ive insects in the ear canal should be immobili7ed
before removal is attempted.

+onsultations! +onsult an 5NT specialist if the obHect cannot be removed or if
t"mpanic membrane perforation is suspected.
KKKKKKKKKKKKKKKKKKKKKKKKKKKKKKKKKKKKKK
KKKKKKKKKKKKKKKKKK..
2:C) !reatment of oen tibial fracture:
a. cehaGolin
b. cehaGolin.gentamycin
c. gentamicin
d. cehaGolin.gentamicin.metronidaGole
treatment of oen fracture
0- 8irst-eneration cephalosporins (@ram-positive coverae) such as
cephale$in (0-1 ,P-:h) suffice for @ustilo t"pe I open fractures.
1- %n aminol"coside (@ram-neative coverae) such as entam"cin (01;
m ,01h9 1R; mOd) is added for t"pes II and III inHuries.
I- %dditionall"* metronida7ole (Q;; m ,01h) or penicillin (0.1 ,Ph) can be
added for coverae aainst anaerobes.
R- Tetanus proph"la$is should be instituted.
Q- %ntibiotics enerall" are continued for D1 hours followin wound closure.

(en fracture classification
@ustilo classification
I 4ow ener"* wound less than 0 cm
II Eound reater than 0 cm with moderate soft tissue damae
III /ih ener" wound reater than 0 cm with e$tensive soft tissue
damae
IIIa %de,uate soft tissue cover
IIIb Inade,uate soft tissue cover
IIIc %ssociated with arterial inHur"
2=3) 7hich of the following found to reduce the risk of ostheretic
neuralgia:
a) corticosteroids only
b) corticosteroids . valacyclovir
c) valacyclovir only
The correct answer is c
Patients with herpes 7oster should receive treatment to control acute
s"mptoms and prevent complications. Patients over the ae of Q;*
irrespective of other ris# factors* are at much reater ris# of developin
postherpetic neuralia and should be offered treatment. 6" inhibitin
replication of varicella 7oster virus* the antiviral aents ac"clovir* famciclovir*
and valaciclovir attenuate the severit" of 7oster_specificall"* the duration of
viral sheddin is decreased* rash healin is hastened* and the severit" and
duration of acute pain are reduced. %ttenuation of the severit" of the acute
infection and the neural damae it causes should reduce the li#elihood of
postherpetic neuralia.
KKKKKKKKKKKKKKKKKKKKKKKKKKKKKKKKKKKKKK
KKKKKKKKKKKKKKKKKK..
2=1) > months old infant with on J off recurrent crying eisodes J hD of
current 6elly stools:
a) intusscetion
b) intestinal obstruction
c) mickelSs diverticulitis
d) strangulated hernia
The correct answer is a
2=2) a man with oblong swelling on to of scrotum increase in siGe with
valsalva maneuver most likely /D:
a) direct inguinal hernia
b) indirect inguinal hernia
c) varicocele
d) femoral hernia
the correct answer is b
\ Indirect hernia: /erniation of abdominal contents throuh the internal and
then e$ternal inuinal rins and eventuall" into the scrotum (in males).
- The most common hernia in both enders.
- .ue to a conenital patent processus vainalis.
- Indirect inuinal hernia increases in si7e it becomes apparent when the
patient couhs* and persists until reduced.
\ /irect hernia: /erniation of abdominal contents throuh the floor of
/esselbachWs
Trianle. (/esselbachWs trianle is an area bounded b" the inuinal liament*
inferior epiastricarter"* and rectus abdominis)
- /ernial sac contents do not traverse the internal inuinal rin9 the" herniate
directl" throuh the abdominal wall and are contained within the aponeurosis
of the e$ternal obli,ue muscle.
- 3ost often due to an ac,uired defect in the transversalis fascia from
mechanical brea#down that Y with ae.
KKKKKKKKKKKKKKKKKKKKKKKKKKKKKKKKKKKKKK
KKKKKKKKKKKKKKKKKK..
2=)) ) weeks old male newborn with swelling of scrotum transarent to
light J irreducible:
a) eidydemitis
b) hydrocele
The correct answer is b
KKKKKKKKKKKKKKKKKKKKKKKKKKKKKKKKKKKKKKK
KKKKKKKKKKKKKKKKK..
2=2) a young girl eDerienced cramy abdominal ain J roDimal muscular
weakness but normal refleDes after receiving setra (trimethorim
sulfamethoDaGole) :
a) functional myositis
b) olymyositis
c) guillian barre syndrome
d) neuritis
the correct answer is d
2=5) 7hich drug causes SB4 like syndrome:
a) hydralaGine
b) roranolol
c) amoDicillin
The correct answer is a
- .ru-induced S45 causes! +hlorproma7ine* /"drala7ine* Isonia7id*
3eth"ldopa* Penicillamine* Procainamide* 2uinidine* Sulfasala7ine.
KKKKKKKKKKKKKKKKKKKKKKKKKKKKKKKKKKKKKK
KKKKKKKKKKKKKKKKKK..
2=:) 7hen to give asirin and cloidogrelO
a) t with a hD of revious &I
b) 'cute &I
c) hD of revious ischemic stroke
d) hD of eriheral artery disease
e) after cardiac cat
The correct answer is b

KKKKKKKKKKKKKKKKKKKKKKKKKKKKKKKKKKKKKK
KKKKKKKKKKKKKKKKKK..
2==) 1= years with history of right iliac fossa ain rebound tenderness .ve
guarding what is the investigation that you will doO
a. Baaroscoy
b. *S
c. +! scan
the correct answer is c
In appendicitis* +T scan with contrast has UQ-U:< sensitivit" which shows
periappendiceal strea#in.
2=>) +hild with history of *%!I before ) weeks now has both knees tender5
red and inflamed. 7hat is your diagnosisO
a. %heumatoid arthritis
A is not comlete
The dianosis of LR% is established b" the presence of arthritis* the duration
of the disease for at least P wee#s* and the e$clusion of other possible
dianoses. %lthouh a presumptive dianosis of s"stemic-onset LR% can be
established for a child durin the s"stemic phase* a definitive dianosis is not
possible until arthritis develops. +hildren must be "ouner than 0P "ears old
at time of onset of disease9 the dianosis of LR% does not chane when the
child becomes an adult. 6ecause there are so man" other causes of arthritis*
these disorders need to be e$cluded before providin a definitive dianosis
of LR% (Table :U-1). The acute arthritides can affect the same Hoints as LR%*
but have a shorter time course. In particular* LR% can be confused with the
spond"loarthropathies* which are associated with spinal involvement* and
enthesitis* which is inflammation of tendinous insertions. %ll of the pediatric
spond"loarthropathies can present with peripheral arthritis before other
manifestations and initiall" ma" be dianosed as LR% (Table :U-I).
\ Luvenile rheumatoid arthritis! mono- and pol"arthropath" with bon"
destruction that occurs in patients ` 0P "ears of ae and lasts N P wee#s.
%ppro$imatel" UQ< of cases resolve b" pubert". Subt"pes! -auciarticular:
\ 5 6oints involved5 associated with an ris# of iridocyclitis that lead to
blindness if left untreated. -olyarticular: aQ small Hoints. S"stemic features
are less prominent9 carries a ris#
of iridoc"clitis. 'cute febrile: The least common subt"pe9 manifests as
arthritis withdaily high5 siking fevers and an evanescent5 salmon#
colored rash. /epatosplenomeal" and serositis ma" also be seen. No
iridoc"clitis is present9 remission occurs within one "ear.
KKKKKKKKKKKKKKKKKKKKKKKKKKKKKKKKKKKKKK
KKKKKKKKKKKKKKKKKK..
2=C) (ld female came comlain from ain in her 6oint increase with
walking5 what is your diagnosisO
a. (steoarthritis
Ph"sical sins of &%! +repitus* rane of motion* pain that worsens with
activit"
and weiht bearin but improves with rest.

2>3) -atient came with history of neck discomfort5 alitation5 cold sweat5
!SF low5 !2 high5 tender neck. 7hat is your diagnosisO
a. Subacute thyroiditis .
# !hyroiditis: Inflammation of the th"roid land. +ommon t"pes are
subacute ranulomatous* radiation* l"mphoc"tic* postpartum* and dru-
induced (e..* amiodarone) th"roiditis.
# FD0-4: In subacute and radiation* presents with tender th"roid* malaise*
and BRI s"mptoms.
# /D: Th"roid d"sfunction (t"picall" h"perth"roidism followed b"
h"poth"roidism)*
all with b upta#e on R%IB.
# %D:
- ^-bloc#ers for h"perth"roidism9 levoth"ro$ine for h"poth"roidism.
- Subacute th"roiditis! %nti-inflammator" medication.
KKKKKKKKKKKKKKKKKKKKKKKKKKKKKKKKKKKKKK
KKKKKKKKKKKKKKKKKK..
2>1) In ulcerative colitis5 what you will start treatment withO
a. +orticosteroids ??
%minosalic"lates and steroid for mild s"mptoms
- Sulfasala7ine or Q-%S% (mesalamine).
- +rticosteroids and immunosuppressants indicated if no refractor" disease.
KKKKKKKKKKKKKKKKKKKKKKKKKKKKKKKKKKKKKK
KKKKKKKKKKKKKKKKKK..
2>2) !reatment of rostatitisO
a. +irofloDacin
treatment is for R-P wee#s with drus that penetrate into the prostate* such as
trimethoprim or ciproflo$acin.

KKKKKKKKKKKKKKKKKKKKKKKKKKKKKKKKKKKKKKK
KKKKKKKKKKKKKKKKK..
2>)) In I@ canula and fluid:
a.Site of entry of cannula is a common site of infection.
KKKKKKKKKKKKKKKKKKKKKKKKKKKKKKKKKKKKKK
KKKKKKKKKKKKKKKKKK..
2>2) -atient with neck rigidity5 rigor5 fever5 etechial rash over eDtremities.
!he causative mRo is:
a. &eningococcal meningitis
.iscuss meninitis c
KKKKKKKKKKKKKKKKKKKKKKKKKKKKKKKKKKKKKK
KKKKKKKKKKKKKKKKKK..
2>5) 8emale atient works in office came with vulva itching5 yellow vaginal
discharge:
a. !richomoniasis
2>:) another one old c0o bilateral knee ain with mild 6oint enlargement
4S% and +%- normal dD:
a. (steoarthritis
b. %heumatoid arthritis
c. EoutO
The correct answer is a
KKKKKKKKKKKKKKKKKKKKKKKKKKKKKKKKKKKKKKK
KKKKKKKKKKKKKKKKK..
2>=) .Scitica increased incidence of :
a. Bumbar lordosis
b. -arasthesis

correct answer is b
KKKKKKKKKKKKKKKKKKKKKKKKKKKKKKKKKKKKKKK
KKKKKKKKKKKKKKKKK..
2>>) old male c0o knee ain on walking with creitus H %ay show narrow
6oint sace and subchondoral sclerosis:
a. %heumatoid arthritis
b. (steoarthritis
c. Eout
The correct answer is b
KKKKKKKKKKKKKKKKKKKKKKKKKKKKKKKKKKKKKKK
KKKKKKKKKKKKKKKKK..
2>C) 12y f with "&I )2.: (associated big chart):
a. (verweight
b. (bese
c. ,ormal weight
The correct answer is b
63I T 0P ! severe under ET.
63I 0P - 1; ! under wt.
63I 1; - 1Q ! normal
63I 1Q - I; ! over wt.
63I I; - IQ ! obese classic 0
63I IQ - R; ! obese classic 1
63I N R; ! obese classic I
2C3) vaginal discharge odorless watry microscoy show clue cells :
a. "acterial vaginosis
b. +andidiasis
The correct answer is a
KKKKKKKKKKKKKKKKKKKKKKKKKKKKKKKKKKKKKKK
KKKKKKKKKKKKKKKKK..
2C1) Prenant women on I
rd
trimester infected with measels can I ive her
33R?????
KKKKKKKKKKKKKKKKKKKKKKKKKKKKKKKKKKKKKKK
KKKKKKKKKKKKKKKKK..
2C2) lactating women infected with ruella $$..management is :
a. &&%
b. Sto lactation
The correct answer is a
33R is safe durin lactatin
KKKKKKKKKKKKKKKKKKKKKKKKKKKKKKKKKKKKKKK
KKKKKKKKKKKKKKKKK..
2C) ) which of following not live vaccine:
a. "+E
b. Featitis "
c. (ral olio
d. &&%
The correct answer is b
KKKKKKKKKKKKKKKKKKKKKKKKKKKKKKKKKKKKKKK
KKKKKKKKKKKKKKKKK..
2C)) the most commone cause of community acTuired neumonia:
a. Femohilus influenGa
b. Stret.neumonia
c. &ycolasma
d. Nliebsella
The correct answer is b
2C2) female after vaginal hesterectomy she comlain of urine come from
vagina$$$dD:
a. @esicovaginal fistula
b. *rethrovaginal fistula
c. *reterovaginal fistula
The correct answer is a
KKKKKKKKKKKKKKKKKKKKKKKKKKKKKKKKKKKKKKK
KKKKKKKKKKKKKKKKK..
2C5) child %t ear ain and tenderness on ulling ear 5 no fever 5 (04
inflamed odemateous rt ear canal with yellow discharge
QQQQQQQQQQdD:
a. (titis media
b. (titis eDterna
c. +holesteatoma
The correct answer is b
KKKKKKKKKKKKKKKKKKKKKKKKKKKKKKKKKKKKKK
KKKKKKKKKKKKKKKKKK..
2C:) male with erianal ain 5tenderness 5fluctuant (erianal abscess)
a. Incision and drainage
b. 7arm bath
The correct answer is a
KKKKKKKKKKKKKKKKKKKKKKKKKKKKKKKKKKKKKK
KKKKKKKKKKKKKKKKKK..
2C=) . regnant )2w with vaginal bleeding$$$. what would you ask about
:
a. +igarette smoking
b. %ecent seDual intercourse
correct answer is a
KKKKKKKKKKKKKKKKKKKKKKKKKKKKKKKKKKKKKK
KKKKKKKKKKKKKKKKKK..
2C>) In duodenal obstruction of neoborn what is the sign that aer in
H%ay:
a. /ouble bubble
2CC) the following more common with tye2 /& than tye1 /&:
a. 7eight loss
b. Eradual onset
c. Fereditary factors
d. FB' /%).#/%2
The correct answer is c
KKKKKKKKKKKKKKKKKKKKKKKKKKKKKKKKKKKKKKK
KKKKKKKKKKKKKKKKK..
533) t with oen angle glaucoma and k0c of +(-/ and /& ttt:
a. !imelol
b. betaDolol
c. 'cetaGolamide
The correct answer is c
KKKKKKKKKKKKKKKKKKKKKKKKKKKKKKKKKKKKKKK
KKKKKKKKKKKKKKKKK..
531) t child with back ain that wake t from slee So diagnosis
a. lumber kyhosis
b. osteoarthritis
c. %'
d. Scoliosis
The correct answer is a or +
KKKKKKKKKKKKKKKKKKKKKKKKKKKKKKKKKKKKKKK
KKKKKKKKKKKKK
532) 5C y0o resented with new onset suraventicular tachycardia with
alitation5no FD of S( or chest ain 5chest eDamination normal 5
oDygen sat in room air X C>9 no eriheral edema(thers normal5 the
best initial investigation:
a. 4+E stress test
b. -ulmonary arteriograhy
c. +! scan
d. !hyroid stimulating hormone
the correct answer is d
53)) Bactating women resented with breast engorgment and tendress
Mour managements:
a. 7arm comressor and continoue breast feeding
b. /icloDacillin and continoue breast feeding
c. /icloDacillin and milk eDression
d. /iscontinoue breast feeding and cold comressor
the correct answer is a . this case is ( enorement of breast ) . but if there
are redness * swellin * fever this case is ( mastitis ) and choose b
532) !he 8astest route of antisychotic is:
a. I&
b. I@
c. (ral
d. Sublingual
the correct answer is b
535) 7hile you are in the clinic you find that many atients resents with
red follicular con6activitis (+hlamydia ) your management is:
a. Imrove water suly and sanitation
b. Imrove sanitation and destroying of the vector
c. 4radication of the reservoir and destroying the vector
d. /estroy the vector and imrove the sanitation

the correct answer is a
53:) !he most imortant eDogenous risk factor for osteoorosis is:
a. 'lcohol intake
b. 'ge
c. Smoking
d. Back of eDercise
the correct answer is c
53=) -atient with family history of coronary artery disease his "&IX 2>
came to you asking for the advice:
a. Start >33 calorie intake daily
b. /ecrease carbohydrate daytime
c. Increase fat and decrease rotein
d. Start with decrease . N calorie er kg er week
the correct answer is d
53>) Bactating mother newly diagnosed with eilesy 5 taking for it
henobarbital you advice is:
a. /iscontinue breastfeeding immediately
b. "reastfeed baby after > hours of the medication
c . +ontinoue breastfeeding as tolerated
the correct answer is c
-ver" vaue ,uestion * some boo#s avoid Phenobarbital durin breast
feedin if possible. %nd in %merican academ" of pediatric classified
Phenobarbital as adru that cause maHor advers effect in some nursin
infant* and should be iven to nursin women with cation .
53C) -regnant women has fibroid with of the following is !rue:
a. -resented with severe anemia
b. Bikely to regress after -regnancy
c. Surgery immediately
d. -resented with 'nteartum Fe
the correct answer is b
8ibroids ma" also be the result of hormones. Reproductive hormones
li#e estroen and proesterone can stimulate cell rowth* causin fibroids
to form. .urin prenanc"* "our influ$ of hormones ma" cause "our
fibroids to row in si7e. %fter prenanc" and durin menopause most
fibroids bein to shrin#* due to a lac# of hormones.
513) ' known case of chronic atrial fibrillation on the warfarin 5 mg came
for follow
u you find I,% = but no signs of bleeding you advice is:
a. /ecrease dose to 2.5 mg
b. Sto the dose and reeat I,% neDt day
c. Sto warfarin
d. +ontinoue same and reeat I,%
the correct answer is b
I,% '+!I(,
Q13 Stop warfarin. +ontact patient for e$amination. 3&NIT&R INR
=#13 Stop warfarin for 1 da"s9 decrease wee#l" dosae b" 1Q< or b" 0 mOd for ne$t
wee# (D m total)9 monitor INR
2.5#= .ecrease wee#l" dosae b" 0Q< or b" 0 mOd for Q da"s of ne$t wee# (Q m total)9
repeat monitor INR
)#2.5 .ecrease wee#l" dosae b" 0;< or b" 0 mOd for I da"s of ne$t wee# (I m total)9
repeat monitor INR.
2#) No chane.
1.5#2 Increase wee#l" dosae b" 0;< or b" 0 mOd for I da"s of ne$t wee# (I m total)9
P1.5 Increase wee#l" dose b" 0Q< or b" 0 mOd for Q da"s of ne$t wee# (Q m total)9
511) -atient is a known case of +'/ the best eDercise:
a. Isotonic eDercise
b. Isometric eDercise
c. 'nerobic eDe
d. Mogha
the correct answer is b
0-anerobic e$ercise ( endurance ) ! for improve cardiac function
1- weiht bearin e$cercise ( isotonic ) ! for build muscle strenth * bone
densit"
I- stretchin e$cercise ! for prevent cramp * stiffness and bac# pain
512) !he mechanism of action of 'sirin:
a. Inhibit cycloDgenase
b. Inhibit hosholiase '2
c. Inhibit hosholiid /
the correct answer is a
51)) !he absolute contraindication of breastfeeding is :
a. 'symtomatic FI@ atient
b. 'ctive heatitis +
c. -ulmonary !" on treatment ) months
The correct answer is a
'bsolute contraindication of breastfeeding :
1-Infants with galactosemia.
2-Mothers who use illegal drugs.
3-Mothers infected with I!" human #-cell l$mphotropic virus t$pe I
or t$pe II" or who have an active herpes lesion on the breast.
%-Mothers ta&ing an$ of the following medications: radioactive
isotopes" cancer chemotherap$ agents" such as antimetabolites
512) ' boy felt down on his elbow 5 the lateral D#ray shows:
a. 'nterior -ad sign
b. -osterior ad sign
c. 'nterior line of humerous intersecting the cubilium
d. %adial line forming C3 degree with cubilium

the correct answer is b
515) ' known case of treated hodgkin lymhoma(mediastinal mass) with
radiotheray ,ot on regular follow u resented with gradual ainless
difficulty in swallowing and S(" 5 !here is facial swelling and redness :
/H
a. S@+ obstruction
b. I@+ obstruction
c. !horacic aortic aneurysm
d. 'bdominal aortic aneursm
The correct answer is a
]]]]]]]]]]]]]]]]]]]]]]]]]]]]]]]]]]]]]]]]]]]]]]]]]]]]]]]]]]]]]]]]
]]]]]]]]
51:) -atient is resented with hand cellulitis and red streaks in the hand
and tender aDillary lymhadenoathy. !his condition is more likely to
be associated with:
a. &alignancy
b. -yoderma
c. ,euroathy
d. Bymhangitis
the correct answer is d

51:) Moung aged male resented to 4% after blunt trauma to 'bdomen5 +!
scan shows intramural hematoma: your management is
a. Baratomy with evacuation of the hematoma
b. /issection of duodenum
c. (bservation
the correct answer is c
51=) -atient resented with sore throat5 anoreDia5 loss of aetite 5 on
throat eDam showed enlarged tonsils with etechi on alate and uvula 5
mild tenderness of sleen and liver :/H
a. Erou ' stre
. b. 4"@
The correct answer is b
51>) -atient with E4%/ has barret esohagus 5 this metalasia increase
risk of :
a. 'denocarcinoma
b. STumaou cell carcinoma
the correct answer is a
51C) +omlication of Slee anea is :
a.+F8
b. $$
the correct answer is a
slee anea : FyoDic ulmonary vasoconstriction -'F +or
-ulmonale +F8
complication of sleep apnea : sleep apnea increases health risks such as
cardiovascular disease, high blood pressure, stroke, diabetes, clinical depression, weight
gain and obesity. The most serious consequence of untreated obstructive sleep apnea is to
the heart. In severe and prolonged cases, there are increases in pulmonary pressures that
are transmitted to the right side of the heart. This can result in a severe form of congestive
heart failure (cor pulmonale).
523) 7hich of the following medication can be used as rohylaDis in
aendectomy:
a. +ehaleDin
b. +eftriaDone
c. &etronidaGole
d. @ancomycin
e. 'micillin
The correct answer is b
0
st
line of antibiotic ! 0- cefo$itin 1- cefotetan
1
nd
line of treatment ! 0- metradina7ole 1- ampici##in-sulbactam
521) 7hich of the following rognostic factor for SB4:
a. ',' levels
b. SeD
c. 'ge
d. %enal involvement
The correct answer is d
522) !he most common site for osteomyelitis is:
a. 4ihysis
b. /iahysis
c. &etahysis
d. "lood flow
The correct answer is c
52)) In ;holding breath holding< which of the following !rue:
a. &ostly occurs between age of 5 and 13
b. Increase %isk of eilesy
c. ' known reciitant cause of generaliGed convulsion
d. /iaGeam may decrease the attack
the correct answer is c
"reath holding sells are the occurrence of episodic apnea in children* possibl"
associated with loss of consciousness* and chanes in postural tone. The" are most
common in children between P and 0: months and usuall" not present after Q "ears of
ae. The" are unusual before P months of ae. % positive famil" histor" can be elicited in
1Q< of cases. It ma" be confused with a sei7ure disorder.
!here are four tyes of breath holding sells.
0-The most common is termed simple (reath holdin& spell5 in which the manifestation
is the holdin of breath in end e$piration. There is no maHor alteration of circulation or
o$"enation and the recover" is spontaneous.
1-The second t"pe are the Cyanotic (reath)holdin& spells. The" are usuall"
precipitated b" aner or frustration althouh the" ma" occur after a painful e$perience.
The child cries and has forced e$piration sometimes leadin to c"anosis (blue in color) *
loss of muscle tone* and loss of consciousness. The maHorit" of children will reain
consciousness. The child usuall" recovers within a minute or two* but some fall asleep
for an hour or so. Ph"sioloicall"* there is often h"pocapnea (low levels of carbon
dio$ide) and usuall" h"po$ia (low levels of o$"en. There is no Fpost ictalF phase (as is
seen with sei7ures)* no incontinence* and the child is fine in between spells. 55@s are
normal in these children. There is no relationship to the subse,uent development of
sei7ures or cerebral inHur" as a conse,uence of breath holdin spells.
I-In the third t"pe* #nown as *allid (reath)holdin& spells* the most common stimulus
is a painful event. The child turns pale (as opposed to blue) and loses consciousness
with little if an" cr"in. The 55@ is also normal* and aain there is no post ictal phase*
nor incontinence. The child is usuall" alert within a minute or so. There ma" be some
relationship with adulthood s"ncope in children with this t"pe of spell.
R- % fourth t"pe* #nown as Complicated (reath)holdin& spells5 ma" simpl" be a more
severe form of the two most common t"pes. This t"pe enerall" beins as either a
c"anotic or pallid spell that then is associated with sei7ure li#e activit". %n 55@ ta#en
while the child is not havin a spell is still enerall" normal.
.d! clinical * ood histor" include se,uence of event * lac# of incontinence and no post
ictal phase. Treatment ! reassurance and iron.
522) Infant brought by the mother that noticed that the baby has decreasing
feeding 5 activity and lethargic (n eDamination febrile()C)5
tachycardic 5his b =50)35 with skin rash . /H:
a.Setic shock
A not comlete
525) (ld atient newly diagnosed with hyerthyroidism resented with
(hyerthrodism symtoms) !he best initial symtomatic treatment is:
a. "" ( beta#blocker )
b. -!*
The correct answer is a
]]]]]]]]]]]]]]]]]]]]]]]]]]]]]]]]]]]]]]]]]]]]]]]]]]]]]]]]]]]]]]]]
]]]]]]]
52:) Infant resented with hemangioma on the back . your management is:
a. Intralesional in6ection of corticosteroids
b. !oical corticosteroids
c. 4Dcise of the lesion
d.observation.
d then b then a. according to the case.
52=) -regnant lady 5 )2 wk E' 5 resented with vaginal bleeding more than
her menstruation. (n eDamination 5 cerviD is dilated ) cm with bulging
of the membrane5 fetal heart rate X 1=3 bm . !he fetus lies transverse
with back facing down . us done and shows that lacenta is attached to
osterior fundus and sonotranulence behind lacenta. Mour
management is :
a. +0S
b. (Dytocin
c.!ocolytics
d.'mniotomy
the correct answer is a
52>) Infant with congenital hi dislocation:
a. .ve click in fleDion 5abduction
b. !he only treatment is surgery
c. ,ot reduced with fleDion and abduction of the hi
the correct answer is a . this is ortolani test

52C) In irritable bowel S. the following mechanism Ocontraction and slow
wave myoelectricity seen in:
a.+onstiation
b. /iarrhea
the correct answer is a
52C) 7hich is not found in coarctation of the aorta:
a. *er limb hyertension
b. /iastolyic murmur heard all over recordium
c. Skeletal deformity on chest D#ray
the correct answer is b
5)3) 8emale atient resented with tender red swelling in the aDilla with
history of reeated black head and large ore skin in same area: ttt is
a. Immidate surgery
b. !oical antibiotic
c. +old comressor
d. (ral antibiotic
b
5)1) In indirect hernia the relation of the sac to the cord structure is:
a. 'nteromedial
b. 'nterolateral
c. -osteromedial
d.-ostrolateral
the correct answer is b
5)2) !he most common cause of crou is:
a. -arainfluenGa
b. InfluenGa
the correct answer is a
5))) kwashikor disease usually associated with :
a. decrease rotein intake5 decrease carbohydrate
b. increase rotein 5 increase carbo
c. decrease rotein 5 increase carbo
the correct answer is c. in the book5 written there is decrease rotein
and adeTuate amount of carbohydrate5 but with this only &+As the
answer is c
5)2) in cachectic atient5 the body utiliGe the roteins of the muscles :
a. to rovide 'mino acid and rotein synthesis
b. to maintain bloodflow to vital organ
c. to increase body fat
the correct anwser is a
5)5) atient is comlaining of memory loss. 'lGehimer disease is
diagnosed what is the cause of this:
a. brain death cell
A not comlete
5):) -arents brought their baby to you who is on bottle feeding. (n eDam
whitish lesion on either side of teeth seen with blackish lesion on
maDillary incisors and second molar teeth. !here is history of leaving
the baby with bottle in his mouth during sleeing. !he /D:
a. ,ursery dental caries
b.Eingvostomatis
the correct answer is a
5)=) 7hich of the following medication if taken need to take the atient
immidiatly to the hosital:
a. -enicillin
b. dihenhydramine
c. (+-s
d. Auinine or Auinidine
the correct answer is d * ,uinidine is antiarrh"thmia .
5)>) 2) y0o female resented with severe /*" other eDamination normal .
your management is
a. / J+
b. (cs
c. Fysterectomy
d. "lood transfusion
the correct answer is c
5)C) "aby with vesicles on the face and honey comb crust which of the
following organism cause it: Stah aureus
523) 8emale atient resented with migraine headache which is ulsatile5
unilateral 5 increase with activity . /osnSt want to take medication.
7hich of the following is aroriate:
a. "io feedback
b. !+'
c. ""
the correct answer is a
6iofeedbac# has been shown to help some people with miraines.
6iofeedbac# is a techni,ue that can ive people better control over bod"
function indicators such as blood pressure* heart rate* temperature* muscle
tension* and brain waves. The two most common t"pes of biofeedbac# for
miraines are thermal biofeedbac# and electrom"oraphic biofeedbac#.
521) Infant born with hemangioma on the %t eyelid what is aroriate time
to oerate to revent amyloia:
a. 1 day
b. 1 week
c. ) months
d. C months
the correct answer is b
522 ) Moung atient on anti !" medication resented with vertigo which of
the following drug cause this:
a. Stretomycin
b. 4thambutol
c. %ifamcin
the correct answer is a * streptom"cin cause :
th
nerve damae.

52)) !he +-% for child is
a. )3 chest comression 2 ventilation $ (Bone rescuer)
b. 15 chest c 2 ventilation $ (2 or more rescures)
c. m15 chest comression 1 ventilation
the correct answer is a
522) -icture show large ulcer over medial side of the leg . what is your
management
a. Shave biobsy
b. 4levate the legs and stocking
c. !oical steroids
the correct answer is b
KKKKKKKKKKKKKKKKKKKKKKKKKKKKKKKKKKKKKKK
KKKKKKKKKKKKKKKKK..
525) 2months infant with white laTue on tongue and greasy 5ast h0o
clamydia con6unctivitis after birth treated by clinamycin what is ttt:
a. (ral nystatin
b. !oical steroids
c. !oical acyclovair
d. (ral tetracycline
the correct answer is a. oral n"statin ! antifunal
KKKKKKKKKKKKKKKKKKKKKKKKKKKKKKKKKKKKKKK
KKKKKKKKKKKKKKKKK..
52:) child %t ear ain and tenderness on ulling ear 5 no fever 5 (04
inflamed odemateous rt ear canal with yellow discharge
QQQQQQQQQQdD:
a. (titis media
b. (titis eDterna
c. +holesteatoma
the correct answer is b
KKKKKKKKKKKKKKKKKKKKKKKKKKKKKKKKKKKKKKK
KKKKKKKKKKKKKKKKK..
52=) )2y female with FI@ a smear negative5 about cervical cancer
screening :
a. 'fter )m if negative reeat after :m
b. 'fter :m $$$$$$$$$$$ annually
c. 'fter 1y$$$$$$$$$$$$$.annually
The correct answer is c
52>) female about )3y with breast cancer (given cbc Kchem. 'nd reavel
low hb and hematocrite$.) what is the neDt ste in mangment:
a. Staging
b. Bumectomy
c. &astectomy
d. chemotheray

the correct answer is a
KKKKKKKKKKKKKKKKKKKKKKKKKKKKKKKKKKKKKKK
KKKKKKKKKKKKKKKKK..
52C) child with moderate ersistant "ronchial'sthma (n bronch.dilat
inhaler. -resented with acute eDacerbation what will you add in ttt:
a. +orticosteroid inhaler
b. Iratroum bromide inhaler
the correct answer is a
KKKKKKKKKKKKKKKKKKKKKKKKKKKKKKKKKKKKKKK
KKKKKKKKKKKKKKKKK..
553) about head and neck in6ury :
a. Foarsness of voice and stridor can occure with mid facial in6ury
b. !racheostomies contraindicated
c. 8acial in6ury may cause uer air way in6ures
KKKKKKKKKKKKKKKKKKKKKKKKKKKKKKKKKKKKKKK
KKKKKKKKKKKKKKKKK..
551) lactating women 13 days after delivary develoed fever 5malaise5
chills tender Bt breast with hotness and small nodule in uer outer
Tuadrant with aDillary B, .Beucocytic count was 12 L130B dD:
a. Inflammatory breast cancer
b. "reast abscess
c. 8ibrocystic disease
the correct answer is b

552) =3y male with osteoorosis the ! score of bone densometry would
be :
a. #).5
b. #2.5
c. 1
d. 2
e. ).5

the correct answer is b
KKKKKKKKKKKKKKKKKKKKKKKKKKKKKKKKKKKKKKK
KKKKKKKKKKKKKKKKK..
55)) child with eistaDis$$.. management:
a. +omression on nose and leaning forward
b. $$$$$$$$$$$$$$$$$$$.backward
the correct answer is a
KKKKKKKKKKKKKKKKKKKKKKKKKKKKKKKKKKKKKKK
KKKKKKKKKKKKKKKKK..
552) scenario about female underwent abdominal oeration she went to
hysician
8or check$$$.. *0S reveal metal thing inside abdomen (a.e missed during
oeration)
7hat will you do :
a. +all the surgeon and ask him what to do
b. +all attorney and ask about legal action
c. !ell her what you found
d. !ell her that is one of ossible comlications of oeration
e. /onSt tell her what you found
The correct answer is c
KKKKKKKKKKKKKKKKKKKKKKKKKKKKKKKKKKKKKKK
KKKKKKKKKKKKKKKKK..
555) male tn with scaly fine aular rash on fornt of scal5nose and
retroauricular$$..(i think tinea caitis) ttt is:
a. NetoconaGole cream$
b. (ral augmentin
c. $$$ cream
the correct answer is a se
hbrroid dematitis
55:) 'll can cause gastric ulcer eDcet:
a# !ricyclic antideressant.
b# /elay gastric emtying.
c# Sesis.
d# Salicylates.
e# Eastric outlet incometent.

The correct answer is a
Tric"clic antidepressant use in treatment of peptic ulcer.
KKKKKKKKKKKKKKKKKKKKKKKKKKKKKKKKKKKKKKK
KKKKKKKKKKKKKKKKK..
55=) 2>year old female lost her menstruation for 2 cycles5 the method of
contracetion is condom5 eDamination was normal eDcet for dusky
discoloration of the cerviD. 7hat u will do neDt:
a.-rogesterone challenge.
b."eta F+E.
c.-elvic u0s
The correct answer is b
KKKKKKKKKKKKKKKKKKKKKKKKKKKKKKKKKKKKKKK
KKKKKKKKKKKKKKKKK..
55>) ttt of yeast vaginitis: +lotrimaGole 5 8luconaGole 5 NetokonaGole
!reatment of yeast vaginitis ( candida albicans ) : toical aGole or -(
fluconaGole 5 oral aGole should be avoided in regnant.

KKKKKKKKKKKKKKKKKKKKKKKKKKKKKKKKKKKKKKK
KKKKKKKKKKKKKKKKK..
55C) 15 y0o female comlaining of ain during menstruation5 not seDually
active5 medical hD unremarkable5 hysical eDamination normal5 how to
treat:
a# ,S'I/.
b# /anaGole.
c#
The correct answer is a
KKKKKKKKKKKKKKKKKKKKKKKKKKKKKKKKKKKKKKK
KKKKKKKKKKKKKKKKK..
5:3) 7hat is true about alha blocker:
a. +auses hyertension.
b. 7orsen benign rostatic hyerlasia.
c. +ause tachycardia.
d#
the correct answer is c
alpha bloc#er !cause orthostatic h"potension and tach"cardia.
5:1) cyclic menstruation that increase in freTuency:
a. -olymenorrhea.
b. Fyermenorrhea.
c. &enorrhagia.
d. /ysmenorrhea.
e.

The correct answer is a
-olymenorrhea ! fre,uent menestration ( T 10 da" c"cle )
Fyermenorrhea or menorrhagia ! abnormall" heav" and proloned
menstrual period at reular intervals . more than :; ml of blood loss per
c"cle or proloned bleedin * more than : da"s.
(ligomenorrhea ! increase lenth of time between menses ( IQ-U; da"s
bew c"cle)
&etrorrhagia : bleedin bwteen period.
&enometrorrhagia ! e$essive and irrelar bleedin.

KKKKKKKKKKKKKKKKKKKKKKKKKKKKKKKKKKKKKKK
KKKKKKKKKKKKKKKKK..
5:2) ' regnant lady came to you to screen her fetus for down syndrome5
what is the best method:
a. 'mniocentesis. . Naryotying
b. +horiocentesis.
c.
the correct answer is a

KKKKKKKKKKKKKKKKKKKKKKKKKKKKKKKKKKKKKKK
KKKKKKKKKKKKKKKKK..
5:)) 7hat is the most common chromosomal abnormalityO
a. !risomy 1)
b. !risomy 21
the correct answer is b
-down s"ndrome ( trisom" 10 ) is the most common chromosomal
abnormalit".
KKKKKKKKKKKKKKKKKKKKKKKKKKKKKKKKKKKKKKK
KKKKKKKKKKKKKKKKK..
5:2) "est 2 test to screen for heatocellular carcinoma:
a. Biver biosy and alha#fetorotein.
b. Biver ultrasound and alha#fetorotein.
c. 'bdomen +! and
d.
The correct answer is b
5:5) ' child came to 4% with fever5 stridor5 $ 5 D#ray showed swollen
eiglottis5 in addition to oDygen5 what u will doO
a. !hroat eDamination.
b. 'n emergency tracheostomy.
c. 4ndotracheal intubation.
d. ,asoharyngeal intubation.
The correct answer is c
KKKKKKKKKKKKKKKKKKKKKKKKKKKKKKKKKKKKKKK
KKKKKKKKKKKKKKKKK..
5::) 22 y0o female newly diagnosed tye 2 /&5 she is wearing glasses for
13 years5 how freTuent she should follow with ohthalmologist:
a. 4very 5 years.
b. 'nnually
The correct answer is b
#for tye 1 diabetic ! retina screenin annuall" beinnin Q "ears after
onset of diabetes* eneral not before onset of pubert".
# for tye 2 diabetic ! screenin at the time of dianosis then annual
KKKKKKKKKKKKKKKKKKKKKKKKKKKKKKKKKKKKKKK
KKKKKKKKKKKKKKKKK..
5:=) 7hat is the initial management for a atient newly diagnosed knee
osteoarthritis.
a. Intra#articular corticosteroid.
b. %educe weight.
c. 4Dercise.
d. Strengthening of Tuadrices muscle.
The correct answer is b
KKKKKKKKKKKKKKKKKKKKKKKKKKKKKKKKKKKKKKK
KKKKKKKKKKKKKKKKK..
5:>) ' lady came to your clinic said that she doesn?t want to do
mammogram and referred to do breast self# eDamination5 what is your
resonseO
a# &ammogram will detect dee tumor.
b# Self#eDamination and mammogram are comlementary.
c# Self#eDamination is best to detect early tumor

answer is b
KKKKKKKKKKKKKKKKKKKKKKKKKKKKKKKKKKKKKKK
KKKKKKKKKKKKKKKKK..
5:C) 7hat is the best freTuency for breast self#eDaminationO
a. /aily.
b. 7eakly.
c. &onthly.
d. 'nnually.

The correct answer is c
5=3) -atient with left bundle branch block will go for dental rocedure 5
regarding endocarditis rohylaDis:
a. ,o need
b. "efore rocedure.
c. 'fter the rocedure.
d.
The correct answer is a
KKKKKKKKKKKKKKKKKKKKKKKKKKKKKKKKKKKKKKK
KKKKKKKKKKKKKKKKK..
5=1) classical characteristic for genital heres.
-ainful ulcers J vesicles
KKKKKKKKKKKKKKKKKKKKKKKKKKKKKKKKKKKKKKK
KKKKKKKKKKKKKKKKK..
5=2) most common vaginal bleeding :
a. cervical olys
b. menstruation
c. OOOO
the correct answer is b
KKKKKKKKKKKKKKKKKKKKKKKKKKKKKKKKKKKKKKK
KKKKKKKKKKKKKKKKK..
5=)) best stimulant for breast milk secretion:
a. breast feeding
b. oDytocin
c. OOO
the correct answer is a
KKKKKKKKKKKKKKKKKKKKKKKKKKKKKKKKKKKKKKK
KKKKKKKKKKKKKKKKK..
5=2) child with asirin intake overdose ...what kind of acid base balance:
a. metabolic alkalosis wt resiratory
b. metabolic acidosis wt resiratory alkalosis
c. resiratory alkalosis with metabolic acidosid
d. resiratory acidosis with metabolic alkalosis
The correct answer is c
Aspirin toxicity ; in early stages, salicylate will stimulate respiratory center
<< respiratory al*alosis that will be compensated by metabolic acidosis. In
late stage, it will interfere with 02, fat, & protein metabolism as well as
xidation phosphorylation lead to lactate, pyrovate, & *eton bodies. All will lead
to p2. 5igne & symptoms includes; nausea, vomiting, <<, temp, 2<,
sweating, cerebral or pulmonary edema, & coma. =ve anion gap. %%%; hydration,
correct >=, gastric lavage or activated charcoal, urine al*ali,ation, hemodialysis)
5=5) theraeutic range of I,% :
a. 2.5#).5
b. 2.3#).3
c. OOO
The correct answer is b
KKKKKKKKKKKKKKKKKKKKKKKKKKKKKKKKKKKKKKK
KKKKKKKKKKKKKKKKK..
5=:) T about antideressant:
a. start single tye even atient have sever deression
b. start any one of them they all have the same efficacy
c. sto the medication after 2 weeks if no imrovement
The correct answer is a
KKKKKKKKKKKKKKKKKKKKKKKKKKKKKKKKKKKKKKK
KKKKKKKKKKKKKKKKK..
5==) atient with deression started on amitrytaline 5 he had headache or
diGGness 5 vomittingOO im not sure what eDactly was the symtoms
a. change to SS%I
b. OO
A not comlete 5 but amitrityline is !+' .
KKKKKKKKKKKKKKKKKKKKKKKKKKKKKKKKKKKKKKK
KKKKKKKKKKKKKKKKK..
5=>) atient had horsenss of voice for ) weeks... neDt to do:
a. throat swab
b. laryngoscoy
c. OOO
!he correct answer is b
KKKKKKKKKKKKKKKKKKKKKKKKKKKKKKKKKKKKKKK
KKKKKKKKKKKKKKKKK..
5=C) single diagnostic for stroke:
a. high cholesterol
b. high systolic blood ressur
the correct answer is b
5>3) unfaivrable rognosis for scheGohrenia:
a. family FD
b. failed marrige
c. adolscen ageOO not sure
d. resence of sychosis
the correct answer is a
KKKKKKKKKKKKKKKKKKKKKKKKKKKKKKKKKKKKKKK
KKKKKKKKKKKKKKKKK..
5>1) %t lung :
a. have 1 fissure
b. contain = segment
c. OOO read about lung anatomy segment namesZZ
Rt lun has I lobes > 0; sements (medial basal sement onl" in inf. 4obe of Rt
lun)
4ft lun has 1 lobes > U sements (4inular sements in lft lun)
KKKKKKKKKKKKKKKKKKKKKKKKKKKKKKKKKKKKKKK
KKKKKKKKKKKKKKKKK..
5>2) T about secifity:
a. test is .ve in disese oulation
b. test is .ve in healthy olation
c. test is #ve in diseased oulation
d. test is #ve in healthy oulation
The correct answer is d
KKKKKKKKKKKKKKKKKKKKKKKKKKKKKKKKKKKKKKK
KKKKKKKKKKKKKKKKK..
5>)) stage ) colon ca Eive +hemo 'S'-
KKKKKKKKKKKKKKKKKKKKKKKKKKKKKKKKKKKKKKK
KKKKKKKKKKKKKKKKK..
5>2) atient with uer abdominal ain5 nausea vomitting5with back ain5
he is smoker for long time daily5 fecal fat was .ve
a. acute ancreatitis
b. chronic ancreatitis
c. ancreatic +'

The correct answer is b
5>5) in school5 13 of atient had the dis of the first week 5 then )3 t. had
the dieses the neDt week 5 the ercentage infection of this school is:
a. 139
b. 239
c. )39
d. 239
I think the correct answer is b.
KKKKKKKKKKKKKKKKKKKKKKKKKKKKKKKKKKKKKKK
KKKKKKKKKKKKKKKKK..
5>:) atient had headche describe it as aband around his head5 increase
with stress and 5 i dont remeber the Tues /H is:
a. tension headache
b. migraine
c. cluster headache
The correct answer is a
KKKKKKKKKKKKKKKKKKKKKKKKKKKKKKKKKKKKKKK
KKKKKKKKKKKKKKKKK..
+,-. *atient 'as presented (y (oll%s in his !oot , (iopsy sho'ed s%(
dermal lysis , !l%orescent stain sho'ed /&0 , 'hat is the most li1ely
dia&nosis #
'. "olus eidermolysis .
". -emhigoid vulgaris .
+. Feretic multiform .
/. "ullous emhigoid .
The correct answer is d
KKKKKKKKKKKKKKKKKKKKKKKKKKKKKKKKKKKKKKK
KKKKKKKKKKKKKKKKK..

5>>) -atient was resented by difficulties of breathing from one side of
his nose 5 on eDamination there was erythramatus swelling 5 what is the
best initial treatment :
'. /econgestant .
". Steroid .
+. Symathomimetics . OO
!he correct answer is c ( ISm not sure )
4Dlain: this swellin miht be nasophar"neal aniofibroma which is hihl"
vascular swellin > prone to epista$is* so to decrease the si7e we miht use
local or s"stemic deconestants which are s"mpathomimetics* so answer is
not clear.
5>C) -atient with cystic nodule (acne) and scars 5 what is the best
treatment :
'. %etinoin oral .
". 4rythromycin .
+. /oDycyclin .
The correct answer is a
this seems to be an advanced inflammator" stae of acne since there r
scars so if the retinoi/ meant here is s"stemic (isotretinin) so it will be the
choice. If topical it will not be effective as it is effective onl" in the non-
inflammator" stae (comedones). 6oth er"throm"cin (local) > do$"c"clin
(s"stemic) can be used but in advanced stae s"stemic is better.
KKKKKKKKKKKKKKKKKKKKKKKKKKKKKKKKKKKKKKK
KKKKKKKKKKKKKKKKK..
5C3) -atient was resented by constiation5 vomiting 5 abdominal
distension 5 with old scar in the lower abdomen 5 D ray showed dilated
loos with air in the rectum 5 what is the best initial management :
'. ,E! decomression 5 and I@ line .
". %ectal decomression and antibiotics .
+. Suositories .
The correct answer is a
this is a case of intestinal obstruction due to adhesions from previous
abdominal surer" (commonest cause) so the initial treatment would be
decompression of the intestine > replace lost fluids
KKKKKKKKKKKKKKKKKKKKKKKKKKKKKKKKKKKKKKK
KKKKKKKKKKKKKKKKK..
5C1) 7hich of the following is not living vaccine :
'. Featitis " .
". &&% .
+. (ral olio
/. "+E
The correct answer is a
/epatitis 6 is the onl" vaccine prepared b" enetic enineerin so its not livin
vaccine.
5C2) SiD years old child was born to a mother with heatitis " 5 he does
not received any vaccines before 5 what you will give now :
'. /!- 5 &&% 5 Fib
". /!- 5 &&% .
+. /! 5 &&% 5 Fib
/. /!5 &&% .
The correct answer is d
both Pertussis > /.influen7a vaccine are not needed after 0 "ear of ae.
KKKKKKKKKKKKKKKKKKKKKKKKKKKKKKKKKKKKKKK
KKKKKKKKKKKKKKKKK..
5C)) 7hich of the following describes the end of the early inflammatory
hase .
'. 8ormation of eschar .
". 8ormation of ground base of collagen .
+. !he end of angiogenesis .
The correct answer is c
KKKKKKKKKKKKKKKKKKKKKKKKKKKKKKKKKKKKKKK
KKKKKKKKKKKKKKKKK..
5C2) 8emale atient 5 known case of @it/ deficiency 5 smoking 5 and
recurrent fall5 which of the following is the greatest eDogenous risk for
osteoorosis :
'. 'dvanced age .
". %ecurrent fall .
+. @it / .
/. Smoking .
The correct answer is d
%dvanced ae > female ender are the most important factors for
osteoporosis but the" are endoenous. Smo#in is an independent
e$oenous ris# factor for osteoporosis
5C5) "lood sugar in /& tye 1 is best controlled by :
'. Short acting insulin .
". Bong acting .
+. Intermediate .
/. Fyoglycemic agents .
4. "asal and bolus insulin .
The correct answer is e
Ver" vaue ,uestion. Ee can e$clude h"pol"cemic aents. Short acting
insulin is best in emerencies li#e .=% as it can be iven IV. Ee can use
either lon actin alone dail" or a mi$ture of short > intermediate actin
insulin dail". "asal J bolus 5 ( short acting . intermediate or long )*
bolusW of short-actin or ver"-short-actin insulin before meals to deal with
the associated rise in blood-suar levels at these times. In addition* the" ta#e
an evenin inHection of lon- or intermediate-actin insulin that helps
normalise their basal (fastin) lucose levels. This offers reater fle$ibilit"
and is the most commonl" adopted method when intensified insulin therap"
is used to provide optimal l"caemic control.
KKKKKKKKKKKKKKKKKKKKKKKKKKKKKKKKKKKKKKK
KKKKKKKKKKKKKKKKK..
5C:) 7hich of the following is true regarding erths disease :
'. +ommonly seen between 11#1: years of age .
". 'lways unilateral .
+. &ay resent by ainless lim .
/. +haracteristically affect the eDternal rotation of hi .
4. &ore in female .
The correct answer is c
Perthes disease (4e-+alve perthes disease) is a condition affectin the hip
Hoint where there is deenerative avascular necrosis of the femoral head. It
affects children aed I-01 "ears > is more common in males. It is most
commonl" but not alwa"s unilateral ( :Q< is unilateral ). It presents mainl"
b" severe hip pain > limpin that increases b" movement but it can present
b" painless limp. It characteristicall" affects the internal rotation > abduction
of the hip > limits these movements.
5C=) 7hich of the following is true regarding gastric lavage :
'. -atient should be in the right lateral osition .
". It is not effective after > hours of asirin ingestion .
The correct answer is b
lavae is effective onl" 0 hour after inestion of an" poison. %fter that its
ineffective
KKKKKKKKKKKKKKKKKKKKKKKKKKKKKKKKKKKKKKK
KKKKKKKKKKKKKKKKK..
5C>) SS%I was rescribed to a atient with deression 5 the effect is
susected to be within :
'. (ne day .
". !wo weeks .
+. !hree to four weeks .
The correct answer is c
%llow 1 - P wee#s to ta#e effect * and treat for N P months
KKKKKKKKKKKKKKKKKKKKKKKKKKKKKKKKKKKKKKK
KKKKKKKKKKKKKKKKK..
+22. Randomi3ed controlled st%dy 'ill (e stron&er (y #
'. Systemic QQQQ
". 8ollowing at least 53 9 of the articiant .
OOOOOOO
KKKKKKKKKKKKKKKKKKKKKKKKKKKKKKKKKKKKKKK
KKKKKKKKKKKKKKKKK..
:33) -artner lost his wife by '&I : months ago 5 resented by loss of
aetite 5 low mood 5 sense of guilt 5 what is the diagnosis :
'. "everament .
". &a6or deression eisode .
The correct answer is a
- 3aHor depression is a ps"chiatric condition that occurs reardless of events
that happen in life* while normall" most people would have beverament after
death of a close person.
:31) 'n adult was resented by sore throat 5 congestion 5 fatigue 5
etechia in soft alate 5 tender sleen 5 and liver 5 what is the most
likely diagnosis :
'. 4"@ .
(nly 1 choiceO
!his could be %ubella infection as etechia in soft alate
(8orschheimer sots) occur in rubella also. "ut liver J sleen affection
are more common in 4"@. 'lso since it is an adult then we might
susect leukemia as the cause.
KKKKKKKKKKKKKKKKKKKKKKKKKKKKKKKKKKKKKKK
KKKKKKKKKKKKKKKKK..
:32) -atient was resented by bleharitis 5 acne roseca 5 but no
keratitis 5 what is the best treatment :
'. !oical chlorohenicol .
". !oical gentamicin .
+. (ral doDycyclin .
The correct answer is c
KKKKKKKKKKKKKKKKKKKKKKKKKKKKKKKKKKKKKKK
KKKKKKKKKKKKKKKKK..
:3)) /iabetic atient was resented by sastic tongue 5 dysarthria 5
sontenous crying what is the most likely diagnosis :
'. -arkinson .
". "ulbar alsy .
+. -seudobulbar .
/. &yasthenia gravis .
The correct answer is c
This is a bit tric#". 6ulbar pals" is the 43N4 of the last R +N* while
pseudobulbar pals" is the B3N4 of the last R +N. So spasticit" of tonue is
B3N4. 6ut .iabetes causes 43N4 of cranial nerves due to peripheral
neuropath". So ma"be the cuase here is +NS affection due to
atherosclerosis from macroaniopath" of diabetes.
:32) 7hich of the following is describe the normal develomental stage
for : months old child :
'. Sits without suort .
". %olls front to back . (=#> months)
+. ,o head lag . () months)
/. Stand alone . (1 year)
The correct answer is a
KKKKKKKKKKKKKKKKKKKKKKKKKKKKKKKKKKKKKKK
KKKKKKKKKKKKKKKKK..
:35) 7hich of the following ersonality is characteriGed by infleDibility 5
erfectionism O
'. (+/ .
". ,ot otherwise secified .
+. ,arcissistic .
The correct answer is b
&bsessive compulsive personalit" disorder is different from obsessive
compulsive disorder and is characteri7ed mainl" b" perfectionism while &+.
is characteri7ed b" repetitive actions due to compulsion. if (+-/ is resent
in the choices it would be the answer.
KKKKKKKKKKKKKKKKKKKKKKKKKKKKKKKKKKKKKKK
KKKKKKKKKKKKKKKKK..
:3:) 7hat is the commonest cause of otorrhea :
'. 'cute otitis media .
". "asal skull fracture .
+. 4Dternal otitis .
The correct answer is a
KKKKKKKKKKKKKKKKKKKKKKKKKKKKKKKKKKKKKKK
KKKKKKKKKKKKKKKKK..
:3=) -atient was resented by ear ain 5 red tymanic membrane 5
aarent vessels 5 with limited mobility of the tymanic membrane 5
what the most likely diagnosis :
'. 'cute otitis media .
". !ymanic cellulitis .
+. &astoditis .
The correct answer is a
:3>) 7hat is the management of acute congestive glaucoma :
'. I@ acetaGolamide and toical ilocarine . (true)
KKKKKKKKKKKKKKKKKKKKKKKKKKKKKKKKKKKKKKK
KKKKKKKKKKKKKKKKK..
:3C) 4ight years old child with late systolic murmur best heard over the
sterna border 5 high itch 5 crescendo 5 decrescendo 5QQ diagnosis is :
'. -hysiological murmur .
". Innocent murmur .
+. 46ection systolic murmur .
/. Systolic regurgitation murmur .
The correct answer is c
KKKKKKKKKKKKKKKKKKKKKKKKKKKKKKKKKKKKKKK
KKKKKKKKKKKKKKKKK..
:13) +hild was resented by coryGa 5 skin rash 5 con6unctivitis 5 and
multile sots in the mouth :
'. &easles .
". %ubella .
The correct answer is a.
3easles ! due to conHunctivitis which is not present in rubella
KKKKKKKKKKKKKKKKKKKKKKKKKKKKKKKKKKKKKKK
KKKKKKKKKKKKKKKKK..
:11) 7hich of the following could be seen in atient with bulimia :
'. Fyokalmeia .
". &etabolic acidosis .
The correct answer is a
bulimia is a#a bin eatin which means the patient eats a lot then does
forced vomitin so there is loss of acids > electrol"tes which leads to
h"po#alemia > metabolic al#alosis.

:12) &aDimum sinal height is reached after menarche by how many
years O
'. &onths .
". !wo years .
+. !hree years .
The correct answer is b
menarche is at 01-0R "ears and ma$imum heiht is about 0D "ears.not sure
KKKKKKKKKKKKKKKKKKKKKKKKKKKKKKKKKKKKKKK
KKKKKKKKKKKKKKKKK..
:1)) -regnant lady )2 weeks of gestation resented by vaginal bleeding 5
which of the following is relevant to ask about :
'. Smoking .
". /esire of future regnancy .
+. !he result of last a smear .
/. FD of vaginal irritation .
The correct answer is a * because smo#in is an important ris# factor for
antepartum hemorrhae
KKKKKKKKKKKKKKKKKKKKKKKKKKKKKKKKKKKKKKK
KKKKKKKKKKKKKKKKK..
:12) 8emale atient with marginal lacenta revia 5 with : cm dilated
cerviD 5 what is your management :
'. 8etal monitoring .
". /elivery if fully dilated .
+. !ocolytics Q
The correct answer is b
I couldnWt find the answer correctl" so based upon the iven data* fetal
monitorin is a must in an" case of hih ris# prenanc". %lso as the PP is
marinal* there is possibilit" of deliver" without severe PP/ and the cervi$ is
alread" dilated.
:15) 7hich of the following drugs increase the survival in a atient with
heart failure :
'. "eta blocker .
6. '+4 inhibitors .
+. /igoDin .
/. ,itrites .
The correct answer is b
New updated information. %s %+5 inhibitors inhibit aldosterone which if
present in hih concentrations causes modification of the cardiac m"oc"tes
in the lon term.
KKKKKKKKKKKKKKKKKKKKKKKKKKKKKKKKKKKKKKK
KKKKKKKKKKKKKKKKK..
:1:) 4lderly atient resented by S(" 5 rales in auscultation 5 high W@- 5
.2 lower limb edema 5 what is the main athohysiology :
'. Beft ventricular dilatation .
". %ight ventricular dilatation .
+. 'ortic regurgitation.
/. !ricuscid regurgitation .
The correct answer is a ( cofuuuuuuuse )
/ifficult Tuestion. /ere we have both s"mptoms of 4ft ventricular failure
(S&6* Rales) > Rt ventricular failure (/ih LVP > 44 edema). So* more
commonl" lft ventricular failure leads to rt ventricular failure due to overload
and not vice versa . so the most correct is 4ft ventricular dilatation.
KKKKKKKKKKKKKKKKKKKKKKKKKKKKKKKKKKKKKKK
KKKKKKKKKKKKKKKKK..
:1=) -atient with high outut fistula 5 for which !-, was ordered 5 after 2
hours of the central venous catheteriGation 5 the atient become
comatose and unresonsive 5 what is the most likely cause :
'. Setic shock .
". 4lectrolytes imbalance .
+. /elayed resonse of blood mismatch .
/. Fyoglycemia .
4. Fyernatremia .
The correct answer is a
:1>) 8emale atient resented by tender mass below the aDilla 5 with
revious history of ores and black heads in that?s area PPP what is
your management :
'. Eo to surgery immediately .
". (ral antibiotics .
+. !oical antibiotics .
/. Steroid .
The correct answer is a * This is mostl" an infected abscess.
KKKKKKKKKKKKKKKKKKKKKKKKKKKKKKKKKKKKKKK
KKKKKKKKKKKKKKKKK..
:1C) +hild was resented by erythema 5 and swelling in his hand after 1>
hours of bee sting 5 what is your management in this case :
'. 4inehrine .
". 'ntihistaminic drugs .
+. FositaliGation .
The correct answer is b
0: hours is a lon duration so its unli#el" to be anaph"lactic shoc# > more
li#el" a reular reaction to bee stins so antihistaminics is the best solution
KKKKKKKKKKKKKKKKKKKKKKKKKKKKKKKKKKKKKKK
KKKKKKKKKKKKKKKKK..
:23) 7hich of the following is true regarding metformin :
'. &ain comlication is hyoglycemia .
". +an lead to weight gain .
+. It suress the heatic glauconeogenesis .
The correct answer is c
KKKKKKKKKKKKKKKKKKKKKKKKKKKKKKKKKKKKKKK
KKKKKKKKKKKKKKKKK..
:21) +hild was resented by congested throat 5 coryGa 5 high grade
fever 5 which of the following is true regarding this condition :
'. @iral Q bacterial .
". "acterial Q viral .
+. 'ntibiotics should be given any way .
/. It is most likely due to 4"@ .
The correct answer is b
:22) "enign tumors of stomach reresent almost :
'. = 9
". 21 9
+. 53 9
/. C3 9
The correct answer is a
KKKKKKKKKKKKKKKKKKKKKKKKKKKKKKKKKKKKKKK
KKKKKKKKKKKKKKKKK..
:2)) :3 years old atient resented by recurrent venous thrombosis
including suerior venous thrombosis 5 this atient most likely has :
'. SB4 .
". ,ehrotic syndrome .
+. "lood grou ( .
/. 'ntihosholiid syndrome .
The correct answer is d
KKKKKKKKKKKKKKKKKKKKKKKKKKKKKKKKKKKKKKK
KKKKKKKKKKKKKKKKK..
:22) 4lderly male atient was resented by left lower abdomen
tenderness 5 fever 5 elevated 7"+ count 5 what is the most likely
diagnosis :
'. Inflammatory bowel .
". /iverticulitis .
The correct answer is b
KKKKKKKKKKKKKKKKKKKKKKKKKKKKKKKKKKKKKKK
KKKKKKKKKKKKKKKKK..
:25) 22 years old male atient was resented by recurrent attacks of
diarrhea 5 constiation 5 and abdominal ain relieved after defecation 5
but no blood in the stool 5 no weight loss : what is the diagnosis :
'. Irritable bowel Syndrome
:2:) -atient was resented by back ain relieved by ambulation 5 what is
the best initial treatment :
'. Steroid in6ection in the back .
". "ack bracing .
+. -hysical theray .
The correct answer is c
KKKKKKKKKKKKKKKKKKKKKKKKKKKKKKKKKKKKKKK
KKKKKKKKKKKKKKKKK..
:2=) -atient with long history of constiation resented by ainful
defecation 5 followed by bleeding 5 ain ersist for hours after
defecation what is the diagnosis :
'. 'nal fissure .
". !hrombosed eDternal iles .
+. Internal hemorrhoid .
The correct answer is a
KKKKKKKKKKKKKKKKKKKKKKKKKKKKKKKKKKKKKKK
KKKKKKKKKKKKKKKKK..
:2>) 4lderly male atient underwent colectomy for colon cancer in which
micrometastais was detected in the lymh nodes 5 what is the best
eDlaniation :
'. Eood rognosis .
". Biver metastasis .
+. It is sensitive to chemotheray . (/ukes class + cancer best for
chemotheray)
/. It is locally advanced .
The correct answer is c
KKKKKKKKKKKKKKKKKKKKKKKKKKKKKKKKKKKKKKK
KKKKKKKKKKKKKKKKK..
:2C) I@ drug abuser was resented by fever 5 arthralgia 5 con6unctival
hemorrhage 5 what is the diagnosis :
'. "acterial endocarditis .
KKKKKKKKKKKKKKKKKKKKKKKKKKKKKKKKKKKKKKK
KKKKKKKKKKKKKKKKK..
:)3) 8emale atient was resented by dysurea 5 eithelial cells were seen
urine analysis 5 what is the eDlanation in this case :
'. +ontamination .
". Infection .
The correct answer is b
:)1) -atient was resented by tremor 5 fever 5 alitation 5 diagnosed as
case of hyerthyroidism 5 what is your initial treatment :
'. Surgery .
". %adio iodine .
+. "eta blockers .
/. -roylthioracil .
The correct answer is c * firstl" 6-bloc#er then Proph"lthiouracil because we
are afraid of arrh"thmias
KKKKKKKKKKKKKKKKKKKKKKKKKKKKKKKKKKKKKKK
KKKKKKKKKKKKKKKKK..
:)2) Moung female atient resented by ain during the first 2 days of
menses 5 menarche was 2 years back 5 what is your initial treatment :
'. (+- .
". ,S'I/ .
The correct answer is b * NS%I. it is the best initial treatment for
d"smenorrheal
KKKKKKKKKKKKKKKKKKKKKKKKKKKKKKKKKKKKKKK
KKKKKKKKKKKKKKKKK..
:))) 8emale is lanning to get regnant 5 she want to get &&% vaccine
also 5 what is your action :
'. /elay the regnancy 2#) months after vaccination .
". It is safe during regnancy .
The correct answer is a
KKKKKKKKKKKKKKKKKKKKKKKKKKKKKKKKKKKKKKK
KKKKKKKKKKKKKKKKK..
:)2) 7hich of the following is true regarding varicella vaccine during
breast feeding :
'. It is safe .
". ,o breast feeding eDcet after ) days of the immuniGation .
'nswer is a
There are no data on the e$cretion of varicella virus vaccine in human mil#.
The manufacturer recommends caution if varicella vaccine is administered to
nursin women. The %dvisor" +ommittee on Immuni7ation Practices (%+IP)
considers the administration of live virus vaccines compatible with breast-
feedin
KKKKKKKKKKKKKKKKKKKKKKKKKKKKKKKKKKKKKKK
KKKKKKKKKKKKKKKKK..
:)5) -atient underwent abdominal surgery due to intestinal erforation
many years back 5 resented by abdominal ain 5 distension 5
constiation 5 what is the best investigation in this case :
'. "arium enema .
". *ltrasound .
+. Small bowel barium study .
The correct answer is c
:):) !rue negative test is best described as following :
'. ,ot susected to have the disease who actually do not have .
KKKKKKKKKKKKKKKKKKKKKKKKKKKKKKKKKKKKKKK
KKKKKKKKKKKKKKKKK..
45-. *atient 'ith AM/ and m%ltiple *6C , is yo%r treatment !or this
arrhythmia #
'. 'miadrone .
". ,o treatment .
KKKKKKKKKKKKKKKKKKKKKKKKKKKKKKKKKKKKKKK
KKKKKKKKKKKKKKKKK..
:)>) 'denosine dose should be reduced in which of the following cases :
'. +hronic renal failure .
". -atients on thiohyline .
The correct answer is a
KKKKKKKKKKKKKKKKKKKKKKKKKKKKKKKKKKKKKKK
KKKKKKKKKKKKKKKKK..
:)C) 7hich the following is the commonest comlication of atient with
chronic atrial fibrillation :
'. Sudden death .
". +erebra vascular accidents .
The correct answer is b * due to multiple atrial thrombi
KKKKKKKKKKKKKKKKKKKKKKKKKKKKKKKKKKKKKKK
KKKKKKKKKKKKKKKKK..
:23) 7hich of the following is most likely seen in case of active
glomerlonehritis :
'. %"+ casts .
KKKKKKKKKKKKKKKKKKKKKKKKKKKKKKKKKKKKKKK
KKKKKKKKKKKKKKKKK..
:21) !arget Fb'1c is less than :
'. :.5
". >
+. C
The correct answer is a
:22) 8emale atient with candida most likely has :
'. /& .
". SB4 .
The correct answer is a
KKKKKKKKKKKKKKKKKKKKKKKKKKKKKKKKKKKKKKK
KKKKKKKKKKKKKKKKK..
:2)) 7hich of the following indicates good rognosis in schiGohrenia :
'. 8amily history of schiGohrenia .
". Eradual onset .
+. 8lat mood .
/. -rominent affective symtoms .
4. ,o reciitating factors .
The correct answer is d
KKKKKKKKKKKKKKKKKKKKKKKKKKKKKKKKKKKKKKK
KKKKKKKKKKKKKKKKK..
:22) -regnant woman with susected /@! 5 what is the best initial
investigations :
'. /uleD *S .
". / dimer .
+. -latysomgrahy .
/. @enogram .
The correct answer is a
KKKKKKKKKKKKKKKKKKKKKKKKKKKKKKKKKKKKKKK
KKKKKKKKKKKKKKKKK..
:25) 7hich of the following is true regarding crohns disease :
'. -artial thickness involvement .
". 8istula formation .
+. +ontinuous area of inflammation .
/. &ainly involve the recto sigmoid area .
The correct answer is b
:2:) -atient serology showed antibodies to the surface antigen for
heatitis " 5 what is your diagnosis :
'. -revious infection or immuniGation .
". +hronic carrier .
+. Fighly infective .
The correcr answer is a
KKKKKKKKKKKKKKKKKKKKKKKKKKKKKKKKKKKKKKK
KKKKKKKKKKKKKKKKK..
:2=) Moung male atient resented by acute scrotal ain 5 *S showed
reduced blood flow 5 what is the diagnosis :
'. !esticular torsion .
". !rauma .
+. Infection .
/. Fernia .
The correct answer is a
KKKKKKKKKKKKKKKKKKKKKKKKKKKKKKKKKKKKKKK
KKKKKKKKKKKKKKKKK..
:2>) 8emale atient is sure that she is regnant for 2 months 5 on
eDamination 5 the uterus is larger than susected 5 "#hcg is very high 5
the doctor diagnosed her as having tumor which is chemo sensitive 5
what is the diagnosis :
'. (varian cancer
". 4ndometrial cancer .
+. Eestational trohoblastic .
The correct answer is c
KKKKKKKKKKKKKKKKKKKKKKKKKKKKKKKKKKKKKKK
KKKKKKKKKKKKKKKKK..
:2C) 7hich of the following is true regarding infertility :
'. It is 8ailure to conceive withing : months . (1 year)
". &ale factor Q female factors . (the reverse)
+. It could be due to high rolactin levels .
/. %are to be due anovulotion . (common)
4. (nly diagnosed by FSE . (need full lab J imaging investigations)
The correct answer is c
:53) )2 years old female atient resented by irregular menses 5 menses
occurs every two months 5 on eDamination every thing is normal 5
which of the following is the B4'S! imortant test to ask about first :
'. +"+ .
". -elvic *S .
+. +oagulation rofile .
/. /F4S .
The correct answer is c
KKKKKKKKKKKKKKKKKKKKKKKKKKKKKKKKKKKKKKK
KKKKKKKKKKKKKKKKK..
:51) -atient with early rheumatoid arthritis 5 what is your management to
decrease the limitation of movement :
- /o not use analgesics or steroids5refer him to %heumatologist to use
/&'%/s like methotreDate or anti!,85 hydroDychloroTuine
KKKKKKKKKKKKKKKKKKKKKKKKKKKKKKKKKKKKKKK
KKKKKKKKKKKKKKKKK..
:52) -atient with truncal obesity 5 easy bruising 5 hyertension 5 buffalo
hum 5 what is the diagnosis :
'. +ushing .
KKKKKKKKKKKKKKKKKKKKKKKKKKKKKKKKKKKKKKK
KKKKKKKKKKKKKKKKK..
:5)) -atient is known case of cervical sondylolysis 5 resented by
arasthesis of the little finger 5 with atrohy of the hyothenar
muscles 5 4&E showed cubital tunnel comression of the ulnar nerve 5
what is your action now :
'. *lnar nerve decomression .
". Steroid in6ection .
+. +! scan of the sine .
The correct answer is a
:52) 7ell known case of S+/ resented by lueritic chest ain 5 fever 5
tachynea 5 resiratory rate was )3 5 oDygen saturation is C3 9 what is
the diagnosis :
'. 'cute chest syndrome .
". -ericarditis :
+. @(+ .
The correct answer is a * or pneumonia would be more correct if it was the
answer
KKKKKKKKKKKKKKKKKKKKKKKKKKKKKKKKKKKKKKK
KKKKKKKKKKKKKKKKK..
:55) +hild with hemangioma around the eye 5 oeration should be done
within $$$ :
'. 1 7eek
". ) months .
+. : months .
The correct answer is a
Ee onl" fear of reduction in the function of the e"e due to hemanioma so it
should be removed.
KKKKKKKKKKKKKKKKKKKKKKKKKKKKKKKKKKKKKKK
KKKKKKKKKKKKKKKKK..
:5:) 7hich of the following is the recommended diet to revent IF/ :
'. /ecrease the intake of meat and dairy .
". /ecrease the meat and bread .
+. Increase the intake of fruit and vegetabl
The correct answer is c
KKKKKKKKKKKKKKKKKKKKKKKKKKKKKKKKKKKKKKK
KKKKKKKKKKKKKKKKK..
:5=) 7hich of the following can lead to olyhydrmnios :
'. /uodenal atresia .
". %enal agenesis . (ligohydramnios
+. -ost term regnancy . (ligohydramnios
/. /iabetes insidious .

The correct answer is a
:5>) 'dult olycystic kidney disease is inherited as:
'. 'utosomal dominant .
". 'utosomal recessive .
+. H linked .
The correct answer is a
KKKKKKKKKKKKKKKKKKKKKKKKKKKKKKKKKKKKKKK
KKKKKKKKKKKKKKKKK..
:5C) !he best treatment for bacteroid :
'. +lindamycin .
KKKKKKKKKKKKKKKKKKKKKKKKKKKKKKKKKKKKKKK
KKKKKKKKKKKKKKKKK..
::3) Mou have an aointment with your atient at 13 am who is newly
diagnosed /& 5 you came late at 11 am because you have another
comlicated atient 5 what are you going to say to control his anger :
'. /o not say any thing .
6. !old him that there is another atient who really need your hel
The correct answer is b
KKKKKKKKKKKKKKKKKKKKKKKKKKKKKKKKKKKKKKK
KKKKKKKKKKKKKKKKK..
::1) 7ell known case of /& was resented to the 4% with drowsiness 5
in the investigations : "lood sugar X 233 mg0dl 5 F X =.35 5 what is
your management O
'. 13 units insulin . 233 cc of deDtrose .
". 3.1 unit0kg of insulin 5 subcutaneous .
+. ,aF+( .
/. (ne liter of normal saline .
The correct answer is d
KKKKKKKKKKKKKKKKKKKKKKKKKKKKKKKKKKKKKKK
KKKKKKKKKKKKKKKKK..
::2) 7hat is the best method for history taking :
'. Mes or no Tuestions .
". (en ended .
+. Silent listening .
The correct answer is b
::)) 7hat you will give to revent hemorrhagic disease of newborns :
'. @itamin N .
KKKKKKKKKKKKKKKKKKKKKKKKKKKKKKKKKKKKKKK
KKKKKKKKKKKKKKKKK..
::2) 'rterial in6ury is characteriGed by :
'. /ark in color and steady .
". /ark in color and surting .
+. "right red and steady .
/. "right red and surting .
The correct answer is d
KKKKKKKKKKKKKKKKKKKKKKKKKKKKKKKKKKKKKKK
KKKKKKKKKKKKKKKKK..
::5) 12 years old girl comlaining of ainless vaginal bleeding for 2#2 days
every )7eeks to 2 months ranging from sotting to 2 acks er dayV
she had 2ry seDual ccc 1 year ago and had her menstruation since :
months on clinical eDamination she is normal seDual ccc5 normal elvic
eDam aroriate action
a. (+- can be used
b. Mou should ask for 8SF and rolactin level
c. /on?t do anything J eDlain this is normal
the correct answer is c
KKKKKKKKKKKKKKKKKKKKKKKKKKKKKKKKKKKKKKK
KKKKKKKKKKKKKKKKK..
:::) 2 years old child what can he do
a. +oy sTuare and triangle
b. Seak in sentences
the correct answer is a , if copy s-uare ) years , but if copu circle 7 years
KKKKKKKKKKKKKKKKKKKKKKKKKKKKKKKKKKKKKKK
KKKKKKKKKKKKKKKKK..
::=) baby can sit without suort5 walk by holding fourniture5 -incer gras5
ull to stand how old is he
a. > months
b. 13 months
c. . 12 month
d. 1> month
%he correct answer is b
PP:) -atient came after dee laceration at the anterior art of the wrist:
a. 7rist dro
b. Sensory loss only
c. +law hand
d. *nable to do thumb oosition
The correct answer is d
1- %adial nerve in6ury ! wrist drop * common with humers inHur" (humers
roove)
I- *nlar nerve in6ury ! claw hand * common with elbow inHur"
R- &edian nerve inury !unable to do thumb opposition * common with wrist
inHur"
KKKKKKKKKKKKKKKKKKKKKKKKKKKKKKKKKKKKKKK
KKKKKKKKKKKKKKKKK..
::C) "est way to diagnose ost stretococcus glomerulonehritis (sot
diagnosis):
a) Bow +)
b) Oor %"+ casts
c) O
The correct answer is a
Eith these 3+2s * I thin# the correct answer is a * but most sensitive in
dianosis of post streptococcus lomerulonephritis is increase 'S( titer !
lump"-bumb" immunofluorescence. or (stretoGyme test 5 which tests
antibodies to 'S() ( if %S& titer written with 3+2s * it is correct answer .
KKKKKKKKKKKKKKKKKKKKKKKKKKKKKKKKKKKKKK
KKKKKKKKKKKKKKKKKK..
:=3) +(-/ atient with emhysema has low oDygen rolonged chronic
high +(25 the resiratory drive is maintained in this atient by:
a. FyoDemia
b. Fyercanemia
c. -atient effort voluntary
d. O
The correct answer is a
KKKKKKKKKKKKKKKKKKKKKKKKKKKKKKKKKKKKKKK
KKKKKKKKKKKKKKKKK..
:=1) +hild with ear ain with ositive um test for tymanic membrane5
treatment is:
a. &aryngiotomy
b. 'moDicillin0-otassium $$.
O
The correct answer is a
:=2) -atient has fatigue while walking last night. Fe is on atrovastatin for
> months5 +irofluDacin5 /ialtiGem and alhaco$.. the cause of this
fatigue is:
a. /ialtiGem and 'trovastatin
b. 'trovastatin and +irofloDacin
c. 'trovastatin and 'lhaco
d. O
the correct answer is b
KKKKKKKKKKKKKKKKKKKKKKKKKKKKKKKKKKKKKKK
KKKKKKKKKKKKKKKKK..
:=)) -atient with ischemic stroke resent after : hours5 the best
treatment is:
a) 'S'
b) !-'
c) +loidogril
d) I@ hearin
e) (ther anticoagulant
The correct answer is a
0- !-' ! administered within Ih of s"mptoms onset ( if no
contraindication)
1- 'S'! use with R:h of ischemic stro#e to reduce ris# of death.
I- +aloidogril ! can be use in acute ischemic
R- Fearin J other anticoagulant ! in patient has hih ris# of .VT or
%8

KKKKKKKKKKKKKKKKKKKKKKKKKKKKKKKKKKKKKKK
KKKKKKKKKKKKKKKKK..
:=2) -atient with lateral and vertical dilobia5 he can?t abduct both eyes5
the affected nerve is:
a) II
b) III
c) @I
d) @
I thin# this is diplopa because nerve R * but with this 2 I choose c

:=5) Infant with heamangioma on forehead obscuring the vision5 best
thing to do is to remove the lesion to avoid amblyobia5 after:
a) (ne day
b) (ne week
c) (ne month
d) : months
e) (ne year
The correct answer is b
KKKKKKKKKKKKKKKKKKKKKKKKKKKKKKKKKKKKKKK
KKKKKKKKKKKKKKKKK..
:=:) -hotohobia5 blurred vision5 keratic OO behind cornea and cells in
anterior chamber5 the best treatment is :
a) !oical antifungal
b) !oical 'cyclovir
c) 'ntibiotic
d) O
The correct answer is b
KKKKKKKKKKKKKKKKKKKKKKKKKKKKKKKKKKKKKKK
KKKKKKKKKKKKKKKKK..
:==) &ost common medical roblems faced in rimary health care is:
a) +oryGa
b) *!I
c) Fyertension
d) /iabetes
The correct answer is a
KKKKKKKKKKKKKKKKKKKKKKKKKKKKKKKKKKKKKKK
KKKKKKKKKKKKKKKKK..
:=>) (ld lady afraid of (steorosis5 to avoid the risk5 you should advise
her to do:
a) 7eight bearing eDercise
b) O
c) O
d) O
The correct answer is a
:=C) (bese lady with essential hyertension5 lab work showed: high ,a5
Figh N5 !he reason for Fyertension is:
a) (besity
b) Figh ,a intake
c) Figh N intake
d) O
The correct answer is a * more than :Q< of essential h"pertension
with 63I N 1Q .
KKKKKKKKKKKKKKKKKKKKKKKKKKKKKKKKKKKKKK
KKKKKKKKKKKKKKKKKK..
:>3) -atient with erutive ururic rash5 heatoslenomegaly5 $$$$
a) O
b) O
c) O
d) O
KKKKKKKKKKKKKKKKKKKKKKKKKKKKKKKKKKKKKKK
KKKKKKKKKKKKKKKKK..
:>1) Single thyroid nodule showed high iodine utake5 best treatment is:
a) %adio Iodine 1)1
b) Send home
c) 'ntithyriod medication
d) 4Dcision if resent
The correct answer is c
KKKKKKKKKKKKKKKKKKKKKKKKKKKKKKKKKKKKKKK
KKKKKKKKKKKKKKKKK..
:>2) 'thletic with tina edis best treatment is:
a) !oical antifungal
b) Systemic antifungal
c) /rug starts with trebenafine
d) O
The correct answer is a
:>)) Moung atient with congested nose5 sinus ressure5 tenderness and
green nasal discharge5 has been treated three times with broad
sectrum antibiotics reviously5 what is your actionO
a) Eive antibiotic
b) ,asal corticosteroid
c) Eive anti histamine
d) /econgestant
The correct answer is b
KKKKKKKKKKKKKKKKKKKKKKKKKKKKKKKKKKKKKKK
KKKKKKKKKKKKKKKKK..
:>2) !hyrotoDicosis include all of the following5 4Dcet:
a) ,euroathy
b) Fyerglycemia
c) -eriheral -roDimal myoathy
d)
2 is not complete * all 3+2s are wron * ma" be choice d is correct if
mention .
KKKKKKKKKKKKKKKKKKKKKKKKKKKKKKKKKKKKKKK
KKKKKKKKKKKKKKKKK..
:>5) &ale atient has hair loss started as fronto#temoral and moving
toward the verteD (to of the head) the diagnosis is:
a) 'ndrogenic aloecia
b) !inea +atus
c) O
d) O
The correct answer is a
KKKKKKKKKKKKKKKKKKKKKKKKKKKKKKKKKKKKKKK
KKKKKKKKKKKKKKKKK..
:>:) Moung healthy male has abdominal ain after basketball.
4Damination fine eDcet for Beft araumbilical tenderness5 what to do:
a) 'bdominal *S
b) 8lat late grah
c) Send home J reassess within 2> hours
d) O
The correct answer is choice a
:>=) 13 year#old boy with $.to tell that sinal cord length will sto after:
a) O
b) O
Bength sto at B1 R B2
KKKKKKKKKKKKKKKKKKKKKKKKKKKKKKKKKKKKKKK
KKKKKKKKKKKKKKKKK..
:>>) +hild with atoic dermatitis at night has stridor lus barking cough
on and off from time to time5 diagnosis is:
a) "'
b) +rou
c) Sasmadic +rou
d) O
The correct answer is c
Sasmadic crou ! recurrent sudden upper airwa" obstruction which
present as sridor and couh . appro$imatel" Q;< of children have
atopic disease.
KKKKKKKKKKKKKKKKKKKKKKKKKKKKKKKKKKKKKKK
KKKKKKKKKKKKKKKKK..
:>C) Sickle cell anemia atient resented with asymtomatic unilateral
hi ain5 most likely diagnosis is:
a) Setic arthritis
b) 'vascular ,ecrosis
c) O
The correct answer is b
KKKKKKKKKKKKKKKKKKKKKKKKKKKKKKKKKKKKKKK
KKKKKKKKKKKKKKKKK..
:C3) "est drug to treat deression in children and adolescent is:
a) 8luDetine (-roGac)
b) O
KKKKKKKKKKKKKKKKKKKKKKKKKKKKKKKKKKKKKKK
KKKKKKKKKKKKKKKKK..
:C1) (ld male with neck stiffness5 numbness and arasthesia in the little
finger and ring finger and ositive raised hand test5 diagnosis is:
a) !horacic outlet syndrome
b) Imingement syndrome
c) *lnar artery thrombosis
d) /o +! scan for +ervical sine
The correct answer is a
:C2) -atient diagnosed with obstructive 6aundice best to diagnose
common bile duct obstruction:
a) 4%+-
b) *S
c) O
The correct answer is a
KKKKKKKKKKKKKKKKKKKKKKKKKKKKKKKKKKKKKKK
KKKKKKKKKKKKKKKKK..
:C)) 4ndometriosis best diagnosed by
a) *S
b) Baroscoy
c) Baratomy
d) O
The correct answer is b
KKKKKKKKKKKKKKKKKKKKKKKKKKKKKKKKKKKKKKK
KKKKKKKKKKKKKKKKK..
:C2) +hild was sick 5 days ago culture taken showed ositive for
meningococcal. -atient now at home and asymtomatic your action
will be:
a) %ifamicin
b) I& +eftriDone
c) O
d) O
2 not complete ( i.m ceftria$on sinle dose or oral rifampicin 1-Q da"s )
KKKKKKKKKKKKKKKKKKKKKKKKKKKKKKKKKKKKKKK
KKKKKKKKKKKKKKKKK..
:C5) Infant with bright blood5 black stool and foul smelling stool. "est
way to know the diagnosis:
a) *S
b) %adio Isoto scan
c) 'ngiogram
d) O
%ll 3+2s are wron * if stool anal"sis is present * it is correct.
:C:) -rimary hyeraldosteronism associated with:
a) Fyernatremia
b) Fyomagnesemia
c) Fyokalemia
d) Fyerkalemia
The correcr answer is c
KKKKKKKKKKKKKKKKKKKKKKKKKKKKKKKKKKKKKKK
KKKKKKKKKKKKKKKKK..
:C=) ' vaccination for regnant lad with /!
a) Eive vaccine and delivery within 22 hrs
b) +ontraindicated in regnancy
c) ,ot contraindicated in regnancy
d) O
The correct answer is c
KKKKKKKKKKKKKKKKKKKKKKKKKKKKKKKKKKKKKKK
KKKKKKKKKKKKKKKKK..
:C>) ' study done to assess the risk of long taking +a in two grous the
diseased grou with long +a lus control according to geograhical
location5 site5 and oulation. It adds (OO) this tye of study:
a) +ohort
b) +ase +ontrol (retrosective)
c) +orrelation study
d) O
The correct answer is a
+ross#Sectional Study:
status of individual with respect to presence and absence of both
e$posure and disease assessed at one point in time
case control study ( retrosective ) :
samples a roup of people who alread" have a particular outcome
(cases) and
compares them to a similar sample roup without that outcome (controls)
cohort study : ( rosective5 incidence 5 longitudinal )
subHects are sampled and as a roup are classified on the basis of
presence or
absence of e$posure to a particular ris# factor
KKKKKKKKKKKKKKKKKKKKKKKKKKKKKKKKKKKKKKK
KKKKKKKKKKKKKKKKK..
:CC) /efine 4idemiology
a) O ;!he study of the distribution and determinants of disease
revalence in man<
=33) +ase +ontrol descrition
a) Start with the outcome the 8* risk factors
b) O
KKKKKKKKKKKKKKKKKKKKKKKKKKKKKKKKKKKKKK
KKKKKKKKKKKKKKKKKK..
=31) *sing gastric lavage :
a) *seless after > hours of 'S' ingestion
b) ,o benefit after : hours of !+' ingestion
c) O
The correct answer is a
KKKKKKKKKKKKKKKKKKKKKKKKKKKKKKKKKKKKKKK
KKKKKKKKKKKKKKKKK..
=32) 13 years old child with rheumatic fever treated early5 no cardiac
comlication. "est to advice the family to continue rohylaDis for:
a) 1 month
b) ) ys
c) 2 ys
d) 15 ys
The correct answer is d ( ICm not sure * if there is 00 "ears choose it )
%merican heart associated has recommended all patient with histor"
rheumatic fever be placed on lone term penicillin proph"la$ic.
.uration of proph"la$is depends on presence or absent of carditis* but
for children without carditis * duration minimum Q "ears or at ae 10*
whichever is loner.
=3)) -icture of large neck mass only no other manifestations or
organomegaly or lymhadenoathy5 diagnosis is:
a) &ononucleosis
b) I would say Eoiter
c) O
d) Bymhoma
The correct answer is b
KKKKKKKKKKKKKKKKKKKKKKKKKKKKKKKKKKKKKKK
KKKKKKKKKKKKKKKKK..
=32) -atient with nausea5 vomiting5 and diarrhea develoed ostural
hyotension. 8luid deficit is:
a) Intracellular
b) 4Dtracellular
c) Interstitial
The correct answer is b
KKKKKKKKKKKKKKKKKKKKKKKKKKKKKKKKKKKKKKK
KKKKKKKKKKKKKKKKK..
=35) +ardiac syncoe:
a) Eradual onset
b) 8ast recovery
c) ,eurological seTuence after
d) O
The correct answer is b
KKKKKKKKKKKKKKKKKKKKKKKKKKKKKKKKKKKKKKK
KKKKKKKKKKKKKKKKK..
=3:) "est way to decrease ain in elderly with bilateral knee ain and
creitation is:
a) ,S'I/
b) /ecrease weight
c) 4Dercise
d) O
The correct answer is b
=3=) Moung female with whitish grey vaginal discharge N(F test OO smell
fish like diagnosis is:
a) Eonorrhea
b) "acterial @aginosis
c) !rachomanous @aginalis
d) O
The correct answer is b
KKKKKKKKKKKKKKKKKKKKKKKKKKKKKKKKKKKKKKK
KKKKKKKKKKKKKKKKK..
=3>) (ld lady with osteoorosis asked for treatment for revention:
a) @I!. /
b) @I!. 4
c) %etonic 'cid
d) O
The correct answer is a
KKKKKKKKKKKKKKKKKKKKKKKKKKKKKKKKKKKKKKK
KKKKKKKKKKKKKKKKK..
=3C) Moung male with morning stiffness at back relieved with activity and
uveitis:
a) 'nkylosing Sondylitis
b) O
KKKKKKKKKKKKKKKKKKKKKKKKKKKKKKKKKKKKKKK
KKKKKKKKKKKKKKKKK..
=13) "est way to revent infection in medical ractice in ediatric
a) 7ear gloves
b) 7ash hand
c) 7ear mask
d) 7ear gown
The correct answer is b
KKKKKKKKKKKKKKKKKKKKKKKKKKKKKKKKKKKKKKK
KKKKKKKKKKKKKKKKK..
=11) Figh risk for develoing colon cancer in young male is:
a) Smoking5 high alcohol intake5 low fat diet
b) Smoking5 low alcohol intake5 high fat diet
c) %ed meat diet5 garden?s disease (Eardner syndrome)
d) Inactivity5 smoking
The correct answer is c
=12) 'lternative theray for severe deression and resistance to anti#
deressant medications are:
a) SS%I
b) !+'
c) 4+!
The correct answer is c
KKKKKKKKKKKKKKKKKKKKKKKKKKKKKKKKKKKKKKK
KKKKKKKKKKKKKKKKK..
=1)) -atient had history of ancreatic cancer on chemotheray then
imroved comletely5 came to doctor concerning about recurrence of
cancer and a history of many hosital visits. !his atient has:
a) &alingering
b) Fyochondriasis
c) 8actitious
d) +onversion
The correct answer is b
KKKKKKKKKKKKKKKKKKKKKKKKKKKKKKKKKKKKKKK
KKKKKKKKKKKKKKKKK..
=12) -atient came with neck swelling5 5moves when atient rotrude his
tongue. /iagnosis is:
a) Eoiter
b) !hyroglossus +yst
c) +ystic Fygroma
d) O
The correct answer is b
KKKKKKKKKKKKKKKKKKKKKKKKKKKKKKKKKKKKKKK
KKKKKKKKKKKKKKKKK..
=15) -regnant atient came with neck swelling and multile nodular non#
tender goiter the neDt evaluation is:
a) !hyroid biosy
b) Eive anti#thyroid medication
c) %adiation Iodine
d) !SF J 8ree !25 or 6ust follow u
The correct answer is d
=1:) Moung atient with F!, came comlaining of high blood ressure
and red5 tender5 swollen big left toe5 tender swollen foot and tender
whole left leg. /iagnosis is:
a) +ellulitis
b) @asculitis
c) Eout 'rthritis
d) OO
The correct answer is a * because tender and swollen whole left le.
KKKKKKKKKKKKKKKKKKKKKKKKKKKKKKKKKKKKKKK
KKKKKKKKKKKKKKKKK..
=1=) 7hat is the in6ection that is routinely given to newborn to inhibit
hemorrhage:
a) @it. N
b) @it. +
c) @it. /
d) @it. 4
The correct answer is a
KKKKKKKKKKKKKKKKKKKKKKKKKKKKKKKKKKKKKKK
KKKKKKKKKKKKKKKKK..
=1>) -atient with strong genetic factor for colon cancer5 what is the
medication that could decrease the risk of colon cancer:
a) 8olic 'cid.
b) @it. +
c) @it. N or '
d) @it. 4
The correct answer is a ( folaic acid and vit. + both are prevent colon
cancer * but folat reduce ris# in people who enic predisposin )
KKKKKKKKKKKKKKKKKKKKKKKKKKKKKKKKKKKKKKK
KKKKKKKKKKKKKKKKK..
=1C) -atient with asthma5 well controlled by albutarol5 came comlaining
of asthma symtoms not resond to albutarol5 what medication could
be added:
a) +orticosteroid inhaler
b) Bong acting "#agonist
c) (ral corticosteroid
d) !heohyline
!he correct answer is a
=23) Fenosch#Scholen urura affect:
a) +aillary
b) +aillary and venule
c) 'rteriole5 caillary and venule
d) 'rtery to vein
The correct answer is c
KKKKKKKKKKKKKKKKKKKKKKKKKKKKKKKKKKKKKKK
KKKKKKKKKKKKKKKKK..
=21) +ontracetive ill that contain estrogen increase risk of:
a) "reast +a
b) (vary +a
c) +ervical +a
d) O
The correct answer is a
KKKKKKKKKKKKKKKKKKKKKKKKKKKKKKKKKKKKKKK
KKKKKKKKKKKKKKKKK..
=22) -atient came with uer resiratory tract infection with red
con6unctiva5 the cause is:
a) @iral infection
b) "acterial infection
c) 8ungal infection
d) O
The correct answer is a
KKKKKKKKKKKKKKKKKKKKKKKKKKKKKKKKKKKKKKK
KKKKKKKKKKKKKKKKK..
=2)) Fealthy atient with family history of /& tye 25 the most factor that
increase chance of /& are:
a) F!, and (besity
b) Smoking and (besity
c) -regnancy and F!,
d) -regnancy and Smoking
The correct answer is a
=22) -atient comlaining of back ain and hyersensitive skin of the
back5 on eDamination5 atient had rashes in the back5 tender5 red base
distributed as blunt shae on the back5 diagnosis is:
a) Feres Ioster
b) +&@
c) O
d) O
The correct answer is a ( , not complete )
KKKKKKKKKKKKKKKKKKKKKKKKKKKKKKKKKKKKKKK
KKKKKKKKKKKKKKKKK..
=25) (ld atient comlaining of hematuria5 on investigation5 atient has
bladder calculi5 most common causative organism is:
a) Schistosoma
b) +&@
c) O virus
d) O virus
The correct answer is a
KKKKKKKKKKKKKKKKKKKKKKKKKKKKKKKKKKKKKKK
KKKKKKKKKKKKKKKKK..
=2:) "lood culture show gram negative rod shae that grow only on
charcoal free fungal organism is:
a) Stah. 'ureus
b) +hlamydia
c) Nlebsiella
d) &ycolasma
%ll above are wron * le&ionella ! rame neative rod rowth on charcoal aar
KKKKKKKKKKKKKKKKKKKKKKKKKKKKKKKKKKKKKKK
KKKKKKKKKKKKKKKKK..
=2=) Nlebesilla feacalis cause the following disease:
a) -neumonia
b) O
c) O
d) O
4nterococcus faecalis +ause Eastroenteritis
KKKKKKKKKKKKKKKKKKKKKKKKKKKKKKKKKKKKKKK
KKKKKKKKKKKKKKKKK..
=2>) /ermatomyositis came with the following symtoms:
a) -roDimal muscle weakness
b) -roDimal muscle tenderness
c) O
d) O
The correct answer is a
KKKKKKKKKKKKKKKKKKKKKKKKKKKKKKKKKKKKKKK
KKKKKKKKKKKKKKKKK..
=2C) "ursitis of the elbow 6oint caused by:
a) 4lbow trauma
b) 'utoimmune disease
c) Stah. 'ureus
d) O ruture of bursa
The correct answer is a
KKKKKKKKKKKKKKKKKKKKKKKKKKKKKKKKKKKKKKK
KKKKKKKKKKKKKKKKK..
=)3) -atient came with symtoms of anDiety including alitation5
agitation5 and worry. !he first best line for treatment is:
a) SS%I
b) !+'
c) "#blocker
d) &'(I
The correct answer is a
KKKKKKKKKKKKKKKKKKKKKKKKKKKKKKKKKKKKKKK
KKKKKKKKKKKKKKKKK..
=)1) -regnant diagnosed with *!I. !he safest antibiotic is:
a) +irofloDacin
b) 'miciln
c) !etracycline
d) O
Eith these 3+2s * the correct answer is b * but if present nitrofurantion
is more accurate answer . BTI in prenanc" treated b" ! nitrofurantion or
cehalosorine ( I- D da"s ) in s"mptomatic or as"mptomatic BTI .
avoid fluroro,uinolone ( which include ! ciproflo$acin* atiflo$acine*
levoflo$acin* norflo$acin ).
=)2) -atient is comlaining of right side harynD tenderness on
eDamination atient had inflamed right tonsil and redness around tonsil
with normal left tonsil. !he diagnosis is:
a) -arenchymal tonsillitis
b) Auinsy araharyngeal abscess
c) O
d) O
The correct answer is b
KKKKKKKKKKKKKKKKKKKKKKKKKKKKKKKKKKKKKKK
KKKKKKKKKKKKKKKKK..
=))) -atient came comlaining of fever5 night sweating5 and hemotysis
with ositive --/ test. 4Damination was normal5 +H% shows infiltrate
of left aical lung but in lateral H#ray showed nothing the reeated --/
test showed normal result diagnosis is:
a) Sarcoidosis
b) %eactivated !"
c) &ycolasma infection
d) @iral infection
The correct answer is b
KKKKKKKKKKKKKKKKKKKKKKKKKKKKKKKKKKKKKK
KKKKKKKKKKKKKKKKKK..
=)2) 8emal atient came with lower abdominal ain5 fever on eDam
atient has lower abdominal tenderness and tender cervical forniD5 the
most aroriate way to diagnose the roblem is:
a) Baroscoy
b) Feterosalingograhy
c) 'bdominal +!
d) %adionuclar Study
The correct answer is a
KKKKKKKKKKKKKKKKKKKKKKKKKKKKKKKKKKKKKKK
KKKKKKKKKKKKKKKKK..
=)5) !he best non#medical theray is roven to be of benefit for
osteoarthritis is:
a) &uscle strength eDercise
b) Eive ,S'I/
c) "ack sla
d) O
The correct answer is a
=):) Fematological disease occurs in children5 treated with hearin and
fresh froGen lasma what is the disease:
a) Femohilia '
b) Femohilia "
c) @on#wille brand disease
d) /I+ thrombosis
The correct answer is d
KKKKKKKKKKKKKKKKKKKKKKKKKKKKKKKKKKKKKKK
KKKKKKKKKKKKKKKKK..
=)=) +hild with *%!I is comlaining of bleeding from nose5 gum and
bruising the diagnosis is:
a) Femohilia '
b) I!!
c) O
d) O
The correct answer is b
KKKKKKKKKKKKKKKKKKKKKKKKKKKKKKKKKKKKKKK
KKKKKKKKKKKKKKKKK..
=)>) -atient came comlaining of vague abdominal ain for : hours then
shifted to right lower Tuadrant diagnosis is :
a) 'cute aendicitis
b) /iverticulitis
c) O
d) O
The correct answer is a
KKKKKKKKKKKKKKKKKKKKKKKKKKKKKKKKKKKKKKK
KKKKKKKKKKKKKKKKK..
=)C) 8emale atient is comlaining of abdominal distension5 fever and
nausea abdominal D#ray showed (Badder sign) management is:
a) +olostomy
b) Ileus treatment
c) %ectal de#obstruction
d) O
The correct answer is b
=23) !he best stimulus for breast milk secretion is :
a) 4strogen
b) "reast feeding
c) O
The correct answer is b
KKKKKKKKKKKKKKKKKKKKKKKKKKKKKKKKKKKKKKK
KKKKKKKKKKKKKKKKK..
=21) 8emale atient did urine analysis shows eithelial cells in urine5 it
comes from:
a) @ulva
b) +erviD
c) *rethra
d) *reter
The correct answer is c
KKKKKKKKKKKKKKKKKKKKKKKKKKKKKKKKKKKKKKK
KKKKKKKKKKKKKKKKK..
=22) 'll of the following are risk factors for heart disease eDcet:
a) Figh F/B
b) &ale
c) (besity
The correct answer is a
KKKKKKKKKKKKKKKKKKKKKKKKKKKKKKKKKKKKKKK
KKKKKKKKKKKKKKKKK..
=2)) !he most signs and symtoms of abrution of lacenta is:
a) @aginal bleed
b) 8etal distress
c) *terus ain and back ain
d) 'bnormal uterine contraction
The correct answer is a
=22) Sign of severe hyokalemia is:
a) -#wave absence
b) -eak !#wave
c) 7ide A%S comleD
d) SeiGure
The correct answer is d
severe hyokalemia is defined as a level less than 2.5 m4T0B.
Severe hyokalemia is not linked with any symtoms5 but may
cause:
0- muscle
1- m"alia or muscle pain
I- disturbed heart rh"thm includin ectop" (disturbance of the electrical
conduction s"stem of the heart where beats arise from the wron part of
the heart muscle)
R- serious arrh"thmias (electrical faster or slower than normal)
Q- reater ris# of h"ponatremia (an electrol"te disturbance in humans when
the sodium concentration in the plasma decreases below 0IQ mmolO4) with
confusion and sei7ures
4+E changes in hyokalemia :
0-T-wave flattenin
1-B-wave ! ( additional wave after the T wave )
I-ST - sement depression
4+E changes in hyerkalemia :
0- pea# T wave
1- wide 2RS ( in severe case )
I- PR prolon ( in severe case )
R- loss of P wave
KKKKKKKKKKKKKKKKKKKKKKKKKKKKKKKKKKKKKKK
KKKKKKKKKKKKKKKKK..
=25) +hild came with his father and high "&I and look older than other
children with same age5 on eDam child has QC5
th
ercentile of weight
and tall5 management is:
a) (bserve and aoint
b) Bife style change
c) Eive rogram to decrease the weight
d) O
The correct answer is a

=2:) -regnant on ):
th
week came with = cm cervical width at 3 station.
/uring birth5 +!E shows late deceleration5 management is:
a) Eive (Dytocin
b) (2 and change mother osition
c) Eive &g sulfate
d) O
The correct answer is b
!ye of deceleration etiology management
early /ead compression from
uterine contraction
(normal )
No treatment
late Bteroplacenta
insufficienc" and fetal
h"po$ima
Place patient on side
.iscontinue o$"tocin.
+orrect an" h"potension
IV h"dration.
If decelerations are associated with
tach"s"stole consider terbutaline ;.1Q m S+
%dminister &1
If late decelerations persist for more than I;
minutes despite the above maneuvers* fetal
scalp p/ is indicated.
Scalp p/ N D.1Q is reassurin* p/ D.1-D.1Q
ma" be repeated in I; minutes.
.eliver for p/ T D.1 or minimal baseline
variabilit" with late or proloned decelerations
and inabilit" to obtain fetal scalp p/
variable Bmbilical cord
compression
+hane position to where 8/R pattern is
most improved. Trendelenbur ma" be
helpful.
.iscontinue o$"tocin.
+hec# for cord prolapse or imminent
deliver" b" vainal e$am.
+onsider amnioinfusion
f %dminister 0;;< &1
=2=) !he way to determine the accuracy of occult blood test for 115333
old atients is by measuring:
a) Sensitivity
b) Secificity
c) -ositive redictive value
d) ,egative redictive value
The correct answer is a
KKKKKKKKKKKKKKKKKKKKKKKKKKKKKKKKKKKKKKK
KKKKKKKKKKKKKKKKK..
=2>) Sickle cell atient5 asymtomatic with history of recurrent gall#
stones and recurrent crisis the management is:
a) +holecystectomy
b) FydroDyurea
The correct answer is a
KKKKKKKKKKKKKKKKKKKKKKKKKKKKKKKKKKKKKKK
KKKKKKKKKKKKKKKKK..
=2C) -atient came with F!,5 N*" shows small left kidney5 arteriograhy
shows renal artery stenosis5 what is the neDt investigation:
a) %enal biosy
b) %enal +! scan
c) %enal barium
d) %etrograde yelograhy
The correct answer is a
KKKKKKKKKKKKKKKKKKKKKKKKKKKKKKKKKKKKKKK
KKKKKKKKKKKKKKKKK..
=53) !he way to differentiate between low iron level from iron deficiency
anemia and anemia of chronic disease is:
a) 8erritin
b) !I"+
c) Serum Iron
d) Serum !ransferrin
The correct answer is a
=51) -atient came with hallucination and illusion the medication that
should be given is:
a) +arbameGain
b) Faloeridol
c) O
The correct answer is b
2 not complete* but with this scenario and 3+2s * the correct answer is b
KKKKKKKKKKKKKKKKKKKKKKKKKKKKKKKKKKKKKKK
KKKKKKKKKKKKKKKKK..
=52) 's doctor if you see atient and you face difficulty to get accurate
information from him the best tactic to do it is:
a) 'sk direct Tuestion
b) 'sk oen Tuestion
c) +ontrol way of discussion
d) O
The correct answer is a
KKKKKKKKKKKKKKKKKKKKKKKKKKKKKKKKKKKKKKK
KKKKKKKKKKKKKKKKK..
=5)) (long Tuestion) atient came with &// so during communication
with atient you will find :
a) Fyomania
b) Bate morning awake
c) Boss of eye contact
d) O
The correct answer is b
KKKKKKKKKKKKKKKKKKKKKKKKKKKKKKKKKKKKKKK
KKKKKKKKKKKKKKKKK..
=52) +hild atient after swimming in ool came comlaining of right ear
tenderness on eDamination atient has eDternal auditory canal redness5
tender5 and discharge the management is:
a) 'ntibiotics ear dros
b) Systemic antibiotics
c) Steroid dros
d) O
The correct answer is a
=55) +hild came with inflammation and infection of the ear the most
comlication is:
a) Babrynthitis
b) &eningitis
c) 4ncehalitis
d) &astoiditis
The correct answer is d
KKKKKKKKKKKKKKKKKKKKKKKKKKKKKKKKKKKKKKK
KKKKKKKKKKKKKKKKK..
=5:) 4lderly atient comlaining of urination during night and describe
when he feel the bladder is full and need to wake u to urinate5 he
suddenly urinate on the bed this is:
a) *rgency incontinence
b) *rge incontinence
c) Stress incontinence
d) 8low incontinence
The correct answer is d
tyes FD of urine loss mechanism treatment
total Bncontrolled loss at all
times and in all position
4oss of sphincter efficienc"
or abnormal connection
bew urinar" tract and
s#in( fistula)
surer"
stress %fter increase abdominal
pressure (couhin*
snee7in* liftin )
Brethral sphincter
insufficienc" due to la$it"
of pelvic floor
musculature * common in
multiparous women
=eal e$erciseand
pessar"
surer"
urge Stron* une$pected ure
to void that is unrelated
to position or activit"
.etrusor h"perrefle$ia or
sphincter d"sfunction or
neuroloic disorder
%nticholineric -
medication or
T+% ! behavior
trainin
overflo
w
+hronic urinar" retention Increase intravesical
pressure that Hust e$ceed
the outlet resistance * allow
small amount of dribble out
Placement of
urethra catheter *
treat underl"in
causes
=5=) ,ewborn came with red#lum on left shoulder5 it is:
a) Femangioma
b) O
c) O
KKKKKKKKKKKKKKKKKKKKKKKKKKKKKKKKKKKKKKK
KKKKKKKKKKKKKKKKK..
=5>) +hild came to ohthalmology clinic did cover test5 during eye
cover 5 his left eye move sontaneously to left5 the most comlication
is:
a) Strabismus
b) Elaucoma
c) &yobloma
d) O
The correct answer is a
KKKKKKKKKKKKKKKKKKKKKKKKKKKKKKKKKKKKKKK
KKKKKKKKKKKKKKKKK..
=5C) ,ewborn came with congenital heatomegaly5 high B8!5 6aundice
the most organism cause this symtoms is:
a) +ongenital !"
b) %ubella
c) FI@
d) +&@
The correct answer is d
KKKKKKKKKKKKKKKKKKKKKKKKKKKKKKKKKKKKKKK
KKKKKKKKKKKKKKKKK..
=:3) Eross motor assessment at age of : months to be asked is:
a) Sitting without suort
b) Standing
c) %ole from rone to suine osition
d) %ole from suine to rone osition
The correct answer is a
=:1) 8emale child came with recocious uberty the most cause is:
a) Idioathic
b) 'drenal tumor
c) "rain tumor
d) O ovarian tumor
The correct answer is a
KKKKKKKKKKKKKKKKKKKKKKKKKKKKKKKKKKKKKKK
KKKKKKKKKKKKKKKKK..
=:2) Femorrhoid usually occurs in:
a) -regnancy and ortal F!,
b) ?
c) ?
d) ?
2 is not complete
&ost common causes of hemorrhoid ! increase strainin ( constipation
) * portal /TN * increase abdominal pressure ( chromin couh * pelvic
tumor ) * obesit" * prenanc" * smo#in
KKKKKKKKKKKKKKKKKKKKKKKKKKKKKKKKKKKKKKK
KKKKKKKKKKKKKKKKK..
=:)) !he immediate urgent referral of child that take
a) 13 ills contracetive
b) 13 ills antibioticsO
c) =5 mg
KKKKKKKKKKKKKKKKKKKKKKKKKKKKKKKKKKKKKKK
KKKKKKKKKKKKKKKKK..
=:2) &ost unwanted side effect of anti#cholinergic drugs is :
a) +onstiation
b) O
c) O
more than Q;< of patients ta#in anticholineric have side effects ! dr"
mouth * blurr" vision * constipation and urinar" retention .
=:5) -atient with /& II with good vision5 to revent eye disease (%etinal
back ground) to develo is to avoid:
a) F!,5 Smoking
b) (besity5 Smoking
c) F!,5 (besity
d) O
The correct answer is c
!he risk factors that increase diabetic retinoathy background
are:
0- /TN
1- Poor lucose control or lon case ..3
I- Raised level of fat ( cholesterol)
R- Renal disease
Q- Prenanc" ( but not in diabetes caused b" prenanc" )

KKKKKKKKKKKKKKKKKKKKKKKKKKKKKKKKKKKKKKK
KKKKKKKKKKKKKKKKK..
=::) "est medication to be given for E/& (gestational) is:
a) Insulin
b) &etformin
c) O
The correct answer is a

=:=) ' child is comlaining of severe headache which is unilateral5
throbbing and aggravated by light5 diagnosis is:
a) &igraine
b) +luster Feadache
c) Stress Feadache
d) O
The correct answer is a
KKKKKKKKKKKKKKKKKKKKKKKKKKKKKKKKKKKKKKK
KKKKKKKKKKKKKKKKK..
=:>) !he most imortant factor for smoker to Tuit is :
a) -atient desire
b) Eive nicotine ills
c) Eive rogrammed lan
d) +hange life style
The correct answer is a
KKKKKKKKKKKKKKKKKKKKKKKKKKKKKKKKKKKKKKK
KKKKKKKKKKKKKKKKK..
=:C) -atient is comlaining of irritation5 tachycardia5 night sweating5 labs
done showed !SF: ,ormal5 !2: Figh5 diagnosis is:
a) Erave?s disease
b) Secondary Fyothyriodism
c) Fashimoto?s thyroiditis
d) O
%ll 3+2s are wron * choice d ma" be correct if mention
,ormal !SF and increase !2 : in thyroid troin secretary itutay
adenoma X thyroid resistance .
KKKKKKKKKKKKKKKKKKKKKKKKKKKKKKKKKKKKKKK
KKKKKKKKKKKKKKKKK..
==3) !he most active form is:
a) !2
b) !)
c) !SF
d) !%F
The correct answer is b
==1) &iddle age man found to have heaviness in his groin. (n hysical
eDamination there was swelling 6ust above his testis which aarent
with valsalva maneuver. 7hat is the diagnosis:
a) /irect inguinal hernia
b) Indirect inguinal hernia
c) 8emoral Fernia
d) !esticular mass
e) Fydrocele
f) @aricocele
The correct answer is b
KKKKKKKKKKKKKKKKKKKKKKKKKKKKKKKKKKKKKKK
KKKKKKKKKKKKKKKKK..
==2) Eardener has recurrent con6unctivitis. Fe can?t avoid eDosure to
environment. In order to decrease the symtoms in the evening5 E-
should advise him to:
a) +old comression
b) 4ye irrigation with @inegar Solution
c) +ontact lenses
d) 'ntihistamines
The correct answer is d
KKKKKKKKKKKKKKKKKKKKKKKKKKKKKKKKKKKKKKK
KKKKKKKKKKKKKKKKK..
==)) 2> year#old male comlaining of lower back ain with morning
stiffness for )3 minutes only. (n eDam he was having sasm centrally
on the lower back. 7hat is the aroriate management :
a) 4idural steroids in6ection
b) "ack brace
c) 8acet lysis
d) -hysiotheray
The correct answer is d
KKKKKKKKKKKKKKKKKKKKKKKKKKKKKKKKKKKKKKK
KKKKKKKKKKKKKKKKK..
==2) 21 weeks regnant lady last biohysical rofile showed
oligohydroamnios. She has no comlaints eDcet mild F!,. 7hat is the
aroriate management :
a) 7ait
b) Induce labor ost 22 wks
c) Induce labor
d) /o biohysical rofile twice weekly
The correct answer is c
==5) 8ull term wide elvis lady5 on delivery station .25 verteD5 +!E
showed late deceleration5 the most aroriate management:
a) +0S
b) Suction
c) 8orces /elivery
d) Sontaneous /elivery
The correct answer is c
KKKKKKKKKKKKKKKKKKKKKKKKKKKKKKKKKKKKKKK
KKKKKKKKKKKKKKKKK..
==:) %ecent study revealed that anti sychotic meds cause the following
comlication:
a. wt gain
b. aloecia
c. cirrhosis
the correct answer is a
KKKKKKKKKKKKKKKKKKKKKKKKKKKKKKKKKKKKKKK
KKKKKKKKKKKKKKKKK..
===) comilication of raid correction of hyernatremia :
a.brain edema
in hyernatremia ! raduall" correction to prevent cerebral edema
in hyonatremia ! raduall" correction to prevent *m"elinol"sis
( include paraparesis e ,uadriparesis * d"sarthria and coma
KKKKKKKKKKKKKKKKKKKKKKKKKKKKKKKKKKKKKKK
KKKKKKKKKKKKKKKKK..
==>) rohylaDis of cholera :
+holera is an infection of the small intestine that is caused b" the bacterium
6i(rio cholerae. The main s"mptoms are profuse water" diarrhea ( rice-
water" diarrhea ) vomitin and deh"dration . infection b" fecal-oral rout .
treatment ! 0- reh"dration * 1-antibiotic ! "oun > adult ! do$c"cline or
tetrac"cline * for children ! S3d-T3P * for prenant ! fura7olidone .
rhylaDis ! ood h"iene and sanitation and oral vaccine * in epidemic
public ! mass sinle dose of vaccine and tetrac"cline.

==>)which one of the anti !" medications cause tinnitus5 imbalance..
a. stretomycin
b. isoniaGide
c. yriGinamide
the correct answer is a * sterptom"cine ! cause :
th
nerve damae
KKKKKKKKKKKKKKKKKKKKKKKKKKKKKKKKKKKKKK
KKKKKKKKKKKKKKKKKK..
==C) the following combination drug should be avoided :
a. levodoa J digoDin
2 is not complete
KKKKKKKKKKKKKKKKKKKKKKKKKKKKKKKKKKKKKKK
KKKKKKKKKKKKKKKKK..
=>3)young male had haryngitis5then cough 5fever5most likely org
a. stah aureus
b. stret neumonia
the correct answer is b
KKKKKKKKKKKKKKKKKKKKKKKKKKKKKKKKKKKKKKK
KKKKKKKKKKKKKKKKK..
=>1)most effective measure to revent sread of infection among health
care workers J ts in a nursery:
a. wash hand befor and after eDamining each t
b. wear gown and gloves before entering the nursery
c. wear shoe cover
the correct answer is a

=>2) 2= yrs old female with erianal ain for 2 days tender erythematous
fluctuating
a. 'bD
b. local +S
c . SitG bath
e. evac J drain
the correct answer is e
KKKKKKKKKKKKKKKKKKKKKKKKKKKKKKKKKKKKKKK
KKKKKKKKKKKKKKKKK..
=>))female t develoed eDtreme from Goo5ark5 sorting events5 the fear
revented her from going out:
a. agorahobia
b. social hobia
c. schiGohrenia
the correct answer is a * ( agorahoia ! fear oin out from the home )
KKKKKKKKKKKKKKKKKKKKKKKKKKKKKKKKKKKKKKK
KKKKKKKKKKKKKKKKK..
=>2)child starts to smile:
a. at birth
b. 2month
c. 1month
The correct answer is b
KKKKKKKKKKKKKKKKKKKKKKKKKKKKKKKKKKKKKK
KKKKKKKKKKKKKKKKKK..
=>5)child recogniGe 2 colors5 5 words5 hos on one foot5 consistent with
which age:
a. 12 mons
b. 22 mons
c. ): mons
d. 1> mons
the correct answer is c
=>:)t with hD of 5 yrs F!, on thiaGide5 came to 4% midnight screaming
holding his Bt foot5 o0e t afebrile5Bt foot tender erythema5 swollen big
toe most tender and ainful5 no other 6oint involvement
a. cellulitis
b. gouty arthritis
c. setic arthritis
the correct answer is b
KKKKKKKKKKKKKKKKKKKKKKKKKKKKKKKKKKKKKKK
KKKKKKKKKKKKKKKKK..
=>=)2) yrs old married for ) mons5 c0o not getting regnant5 they have
intercourse )#2 times0week5 normal gynecologic hD5 husband 25 yrs old
healthy wt would you advice:
a. cont. trying
b. obtain serm analysis
c. study of tubes atency
the correct answer is a
KKKKKKKKKKKKKKKKKKKKKKKKKKKKKKKKKKKKKKK
KKKKKKKKKKKKKKKKK..
=>>)t with '8 what is the most common comlication:
a. cerebrovascular events
b. v.tach
c. '&I
d. v.fib
the correct answer is a
KKKKKKKKKKKKKKKKKKKKKKKKKKKKKKKKKKKKKKK
KKKKKKKKKKKKKKKKK..
=>C)case scenario : oral.genital ulcer .arthritis
a. behcet disease
b. syhilis
c.heres simleD
the correct answer is a
=C3) 23 yrs old lady5 regnant5 eDosed to rubella virus since ) days 5 never
was vaccinated against rubella mums or measles 5 whatSs the best
thing to do:
a. give IE
b. vaccine
c. do nothing
d. terminate the regnancy
the correct answer is c * onl" supportive . ( 0;; < correct )
KKKKKKKKKKKKKKKKKKKKKKKKKKKKKKKKKKKKKKK
KKKKKKKKKKKKKKKKK..
=C3)which of the following not comatible with head engagement:
a. verteD at Gero station
b. crowning of the head
c. )05 head felt in the abdomen
d. "-/ at ischial sines
the correct answer is c * 7hen the fetal head is engaged5 205 or less of
the head is alable above the elvic
=C1)5> yrs old female5 known case of osteoenia5 sheSs asking you abt the
best way to revent comression vertebral fracture5 what would you
advice her:
a. avoid obesity
b. vit./ daily
c. wt bearing eDercise
the correct answer is c * cnfuse with choice b
mild osteoenia ( ! score #1 to #2 )! life st"le modification ( stop
smo#in ) * dail" calcium inta# ( 0Q;; m dail" ) * e$ercise * and
reassessed after I - Q "ears .
severe osteoenia ( ! score #2 to K 2.5 ) and osteoorosis ( ! score Q
2.5 ) ! same mild pus pharmacoloical .
KKKKKKKKKKKKKKKKKKKKKKKKKKKKKKKKKKKKKKK
KKKKKKKKKKKKKKKKK..
=C2) &other comlains of shar ain on radial styloid when carrying her
baby. !he ain increase with eDtension of the thumb against
resistance5 8inkelstein test was ositive5 /D :
a. (steoarthritis of radial styloid
b. /e Auervain !enosynovitis
the correct answer is b
KKKKKKKKKKKKKKKKKKKKKKKKKKKKKKKKKKKKKK
KKKKKKKKKKKKKKKKKK..
=C)) 1
st
line in !rigeminal ,euralgia management:
a. +arbamaGeine
KKKKKKKKKKKKKKKKKKKKKKKKKKKKKKKKKKKKKK
KKKKKKKKKKKKKKKKKK..
=C2) +ontraindicated in acute glaucoma management:
a. -ilocarine
b. !imolol
c. "#blockers5 +' inhibitors5 ,S'I/5 &annitol
d. O
%ll of 3+2s can be use in acute laucoma . ma" be choice d is correct if
mention.
=C5) !rue about systolic hyertension:
a. could be caused by mitral %egurg
b. &ore serious than diastolic hyertension
c. systolicQ123 and diastolic PC3
d. O
the correct answer is c
KKKKKKKKKKKKKKKKKKKKKKKKKKKKKKKKKKKKKK
KKKKKKKKKKKKKKKKKK..
=C:) -regnant )>E'5 resented in labor5 dilated cerviD5 station 225 late
deceleration on +!E5 management:
a. continuo sontaneous labor
b. 8orce delevary
c. @acuum /elevary
d. +S
!he correct answer is a
KKKKKKKKKKKKKKKKKKKKKKKKKKKKKKKKKKKKKKK
KKKKKKKKKKKKKKKKK..
=C=) -t5 febrile5 tender rostate on -%:
a#'cute -rostitis
KKKKKKKKKKKKKKKKKKKKKKKKKKKKKKKKKKKKKKK
KKKKKKKKKKKKKKKKK..
=C>) -roven to revent some cancers:
a. +a
b. 8olic 'cid
c. @it./
the correct answer is c
KKKKKKKKKKKKKKKKKKKKKKKKKKKKKKKKKKKKKKK
KKKKKKKKKKKKKKKKK..
=CC) -atient with cont. &urmur:
a. -/'
b. +oarctation of 'orta
the correct answer is a
>33) (ld man5 ost (%5 +omlains of unilateral face swelling:
a. S6or6an syndrome
b. Sarcoidosis
c. Salivary gland tumor
d. Salivary gland stone
the correct answer is d
KKKKKKKKKKKKKKKKKKKKKKKKKKKKKKKKKKKKKKK
KKKKKKKKKKKKKKKKK..
>31) 't what level B- done:
a. B2#B)
b. B)#B2
c. B5#S1
The correct answer is b
KKKKKKKKKKKKKKKKKKKKKKKKKKKKKKKKKKKKKKK
KKKKKKKKKKKKKKKKK..
>32) 2months old with scaling lesion on scal and forehead5 /D:
a. Seberrhoic /ermatitis
b. 4rythema multiforme
the correct answer is a
KKKKKKKKKKKKKKKKKKKKKKKKKKKKKKKKKKKKKKK
KKKKKKKKKKKKKKKKK..
>3)) -t have high "lood -ressure on multible visits5 so he was diagnosed
with hyertension5 what is the -athohysiology:
a. increased eriheral resistance
b. increased salt and water retention
the correct answer is a
KKKKKKKKKKKKKKKKKKKKKKKKKKKKKKKKKKKKKKK
KKKKKKKKKKKKKKKKK..
>32) 1)years old with hD of neumonia and managed with abD 2 weeks
back5 now he came with diarrhea5 abdominal ain5 and .ve 7"+ in
stool5 the causative organism is:
a. +lostridium dificile
KKKKKKKKKKKKKKKKKKKKKKKKKKKKKKKKKKKKKKK
KKKKKKKKKKKKKKKKK..
>35) "est &anag. (f +lost./eficile is:
a. &etronidaGole
b. /oDacycline
the correct answer is a
>3:) -rohylaDis of arrhythmia ost &I :
a. Auinidine
b. Auinine
c. Bidocaine
d. -rocainamide
the correct answer is c * if b-bloc#er is present choose it
KKKKKKKKKKKKKKKKKKKKKKKKKKKKKKKKKKKKKKK
KKKKKKKKKKKKKKKKK..
>3=) +hronic /iarrhea is a feature of:
a. Fyer,atremia
b. Fyer+alcemia
c. Fyo&agnesemia
d. &etabolic 'lkalosis
The correct answer is d
KKKKKKKKKKKKKKKKKKKKKKKKKKKKKKKKKKKKKKK
KKKKKKKKKKKKKKKKK..
>3>) -t with have -olyuria and thirst5 he had FD of biolar on lithium5 /D:
a. ,ehrogenic /iabetes Insiidus
b. central /iabetes Insiidus
the correct answer is a
KKKKKKKKKKKKKKKKKKKKKKKKKKKKKKKKKKKKKKK
KKKKKKKKKKKKKKKKK..
>3C) In city with oulation of 15333 eole J 135 birth 0yr 5 2 are still
birth 5) died within months 52 died before their 1
st
birthday 5 with =53
come out J 253 come in what is the birth mortality rate in this city :
a. 2
b. :
c. >
d. C
A not comlete ( I didSt finde any thing about bierth mortality rate )
>13) 8emale with recently inserted I*+/ coming with watery brownish
vaginal discharge J abdominal ain what is the most likely /D :
a. *terine ruture
b. (varian torsion
c. "acterial vaginosis
d. 4ctoic regnancy
The correct answer is d
KKKKKKKKKKKKKKKKKKKKKKKKKKKKKKKKKKKKKKKKK
KKKKKKKKKKKKKKK..
>11) )> y0o male calling you comlaining of fear from going outside J
fear from seeing the front door J don?t sit in the room containing the
front door what is most likely /D:
a. /eression
b. 'gorahobia
c. &alingering
d. 'nti#social ersonality
The correct answer is b
KKKKKKKKKKKKKKKKKKKKKKKKKKKKKKKKKKKKKKKK
KKKKKKKKKKKKKKKK..
>12) :3 y0o male known to have ( "-F) digital rectal eDamination shows
soft rostate with multile nodularity J no hard masses 5 the t
reTuest for ( -S') for screening for rostatic ca what will you do :
a. Sit with the t to discuss the cons J rods in -S' test
b. /o trans#rectal *S because it is better than -S' in detection
c. /o multile biosies for different sites to detect rostatic ca
d. $$$$$$$$.
The correct answer is a
KKKKKKKKKKKKKKKKKKKKKKKKKKKKKKKKKKKKKKKKK
KKKKKKKKKKKKKKK..
>1)) 7hat is the most effective ttt for rocasea :
a. +lindamycine
b. 4rythromycin
c. !oical steroids
d. $$..
2 is not clear or not complete. ( if mention tetrac"cline choose it )
!reatment of %osacea : &ral tetrac"clines > topical metronida7ole or
topical erth"rom"cin or topical clinam"cine . for severe case ! isotretinoin .
surgical treatment for rhinohyma .
>12) = y0o child coming with S(" and wheeGing he was sitting in bed 5
leaning forward 5 with drooling J strider what is /D :
a# 4iglottits
b# "ronchial asthma
c# $$$
d# $$$
The correct answer is a
KKKKKKKKKKKKKKKKKKKKKKKKKKKKKKKKKKKKKKKKK
KKKKKKKKKKKKKKK..
>15) 13 y0o child brought by his arents because they where concern
about his weight 5 he eats a lot of fast food and 8rench fries 5 your main
concern to manage this t is :
a# Fis arents concerning about his weight
b# Fis "&I Q ))
c# 8amily FD of heart disease
d# 4ating habit ( fast food 5 8rench fries )
KKKKKKKKKKKKKKKKKKKKKKKKKKKKKKKKKKKKKKKKK
KKKKKKKKKKKKKKK..
>1:) 7hat is an absolute contraindication of (+- :
a. FD of revious /@!
b. (varian ca
c. "reast ca
d. $$$.
The correct answer is a * both h$ of .VT and breast cancer are absolute
contraindication * but in h$ .VT is more accurate
KKKKKKKKKKKKKKKKKKKKKKKKKKKKKKKKKKKKKKKKK
KKKKKKKKKKKKKKK..
>1=) (+- is roven to :
a. /ecrease ovarian ca
b. /ecrease endometrial ca
c. Increase breast ca
d. Increase risk of ectoic regnancy
The correct answer is c

>1>) "aby born J discharge with his mother 5 )weeks later he started to
develo difficulty in breathing J become cyanotic what is most likely
/H :
a# @+/
b# Fyolastic Bf ventricle
c# +oarctaion of aorta
d# Subaortic hyertrohy
The correct answer is b
KKKKKKKKKKKKKKKKKKKKKKKKKKKKKKKKKKKKKK
KKKKKKKKKKKKKKKKKK..
>1C) :3 y0o male /D to have acute ancreatitis ( there was values for
ranson criteria that I couldn?t memoriGe ) what is the aroriate
nutrition :
a# !-,
b# %egular diet with low sugar
c# Figh rotein 5high ca 5 low sugar
d# ,aso#6u6enal tube
The correct answer is d
KKKKKKKKKKKKKKKKKKKKKKKKKKKKKKKKKKKKKKK
KKKKKKKKKKKKKKKKK..
>23) 15 y0o boy brought by his mother with lab values shows microcytic
hyochromic anemia what is most likely /D :
a# !halassemia
b# I/' ( iron deficiency anemia )
c# S+/
d# $$.
The correct answer is b
KKKKKKKKKKKKKKKKKKKKKKKKKKKKKKKKKKKKKKK
KKKKKKKKKKKKKKKKK..
>21) 8emale with FD of lt flank ain radiating to groin 5 symtoms of *!I
J ,.@ what is /H:
a# 'endicitis
b# /iverteculitis
c# %enal colic
d# $$..
The correct answer is c
>22) (ld t with neck swelling 5 nodular 5disfiguring 5 with FD of muscle
weakness 5 cold intolerance 5 hoarseness 5 what is your management :
a# BevothyroDine
b# +arbamaGole
c# !hyroid lobectomy
d# %adio#active iodine
The correct answer is a
KKKKKKKKKKKKKKKKKKKKKKKKKKKKKKKKKKKKKKK
KKKKKKKKKKKKKKKKK..
>2)) -regnant woman with symtoms of hyerthyroidism 5 !SF low :
a# -roylthyiouracil
b# %adio#active iodine
c# -artial thyroidectomy
d# $$$$.
The correct answer is a
KKKKKKKKKKKKKKKKKKKKKKKKKKKKKKKKKKKKKKK
KKKKKKKKKKKKKKKKK..
>22) -t with thyroid mass 5 firm 52D2 cm what is most aroriate for /D :
a# ,eck *S
b# 8,'
c# ,eck +!
d# Surgery
The correct answer is b
KKKKKKKKKKKKKKKKKKKKKKKKKKKKKKKKKKKKKKK
KKKKKKKKKKKKKKKKK..
>25) 5: y0o female comlaining of back ain 5 her mother suffer from
vertebral comression 8D 5 investigation shows early osteoorosis
what is the best ttt :
a# 'dvice with eDercise
b# @it / 5 ca sulement 5 $$.
c# $$$$$.
d# $$$$$
The correct answer is b
KKKKKKKKKKKKKKKKKKKKKKKKKKKKKKKKKKKKKKK
KKKKKKKKKKKKKKKKK..
>2:) 7hat is the most common non#traumatic 8D caused by osteoorosis
:
a# +olleSs 8D
b# 8emoral 8D
c# @ertebral comression 8D
The correct answer is c * because most common non traumatic .
>2=) 2 or 5 ( not sure ) brought by his arents with weight Q C5
th

ercentile 5 height P 5
th
ercentile J bowing of both legs what is the
aroriate management :
a# Biver J thyroid function tests
b# Bower limb H#ray
c# -elvis H#ray
d# !hyroid or ( liver not sure ) function test
The correct answer is a
KKKKKKKKKKKKKKKKKKKKKKKKKKKKKKKKKKKKKK
KKKKKKKKKKKKKKKKKK..
>2>) 't which stage saration of chromatoids occur :
a# &etahase
b# !elohase
c# $$$$.
d# $$$$$
The correct answer is a
KKKKKKKKKKKKKKKKKKKKKKKKKKKKKKKKKKKKKKK
KKKKKKKKKKKKKKKKK..
>2C) 4lderly t with /&2 5 F!, 5 smoker 5 sedentary lifestyle 5 inestigation
low F/B5 high B/B 5 ( - ,' - '- ^-- ) which of what will be effective
in this t :
a# Bevel of F/B 5 smoking 5 obesity
b# !riglyceride level 5 $$$$$$$$
c# $$$$$$$$$$$$$$$$$$$.
d# $$$$$$$$$$$$$$$$$$$.( all were long sorry )
&+As are not comlete.
KKKKKKKKKKKKKKKKKKKKKKKKKKKKKKKKKKKKKKK
KKKKKKKKKKKKKKKKK..
>)3) t known case of ulcerative colitis with erythematous rash in lower
limb what is most likely /H:
a# erythema nodusum

>)1) -t known to have ulcerative colitis coming with skin lesion around
!ibia which is with irregular margins what is most likely /D:
a# -yoderma gangirenosum

KKKKKKKKKKKKKKKKKKKKKKKKKKKKKKKKKKKKKK
KKKKKKKKKKKKKKKKKK..
>)2) Moung t victim of %!' coming with S(" ( or not breathing not
sure ) absent of breath sound J chest movement in %! side but
resonance in ercussion what is your neDt ste :
a# /o stat chest H#ray
b# 4mergency surgery
c# $$$$$$$$$$$.
d- KKKKKKKKKKKK
Needle aspiration then chest tube
KKKKKKKKKKKKKKKKKKKKKKKKKKKKKKKKKKKKKKK
KKKKKKKKKKKKKKKKK..
>))) -sychiatric t non#comliance to his medication what is the ttt :
a# $$$$. Eiven I&
b# $$$$$$$.. given I@
c# $$$$$$..oral
d# $$$$$$$..oral
+orrect answer is a
KKKKKKKKKKKKKKKKKKKKKKKKKKKKKKKKKKKKKKK
KKKKKKKKKKKKKKKKK..
>)2) 7hich sychiatric disease is treated with electroconvulsive
theray :
a# -aranoia
b#ma6or deression
!he correct answer is : &a6or deression
>)5) -t with bilateral eye discharge 5 watery 5 red eyes5 corneal ulceration
what is the most common cause :
a# /ust J ollen
b# Fyertension J $$$$$
c# *ltra#violet light J stress
d# $$$$$$$$$$$$$.
The correct answer is a
KKKKKKKKKKKKKKKKKKKKKKKKKKKKKKKKKKKKKKK
KKKKKKKKKKKKKKKKK..
>):) 2 y0o baby felt down his mother ulled him by his arm J since then
he ket his arm in ronation osition what is your management:
a# Slint $$$$.
b# /o D#ray for the arm before any intervention
c# (rthoedic surgery
d# $$$$$$$$$$$$.
The correct answer is b
KKKKKKKKKKKKKKKKKKKKKKKKKKKKKKKKKKKKKKK
KKKKKKKKKKKKKKKKK ..
>)>) diabetic t on medication found un conscious his blood suger was :3
what is the most common to cause this roblem:
a# Sufonylurease
b# "igunides
c# $$$$$$$$
The correct answer is a
KKKKKKKKKKKKKKKKKKKKKKKKKKKKKKKKKKKKKKK
KKKKKKKKKKKKKKKKK..
>)C) what is most sensitive indicator for factitious fever :
a# -ulse rate
b# $$$$$$$$.
c# $$$$$$$$$.
8actitious fever! 8ever produced artificiall" b" a patient. This is done b"
artificiall" heatin the thermometer or b" self-administered p"roenic
substances. %n artificial fever ma" be suspected if the pulse rate is much
less than e$pected for the deree of fever noted. This dianosis should be
considered in all patients in whom there is no other plausible e$planation for
the fever. Patients who pretend to have fevers ma" have serious ps"chiatric
problems.
>23) young male ( I think he was a student ) with headache involving the
whole head J bilateral eye glob that increase with stress what is most
likely /H:
a# !ension headache
b# &igraine
c# $$$$$$$$.
d# $$$$$$$$$.
A not cmlete
KKKKKKKKKKKKKKKKKKKKKKKKKKKKKKKKKKKKKKK
KKKKKKKKKKKKKKKKK..
>21) t turns to be erratic 5 for 2 month he said that?s eole in !@ knows
what eole are thinking about 5 in last 2 month he claim that he has
secial ower that no one has what is the most likely /H :
a# *ni#olar $.. deression
b# "iolar $. &ania
c# S+I
d# $$$$$$$$
The correct answer is c
KKKKKKKKKKKKKKKKKKKKKKKKKKKKKKKKKKKKKKK
KKKKKKKKKKKKKKKKK..
>22) common site of anal fissure is :
a# 'nterior
b# -osterior
c# Bateral
d# $$$$$$$$.
The correct answer is ! b
KKKKKKKKKKKKKKKKKKKKKKKKKKKKKKKKKKKKKKK
KKKKKKKKKKKKKKKKK ..
>2)) common site of hernia sac in relation to the cord content :
a# 'ntro#lateral
b# &edial
c# 'nywhere
d# -osterior
The correct answer is a
>22) what a 2 years child can do :
a# /raw sTuare J triangle
b# Say comlete sentence
c# !ie his shoes
d# $$$$$$$$$$$..
The correct answer is a
KKKKKKKKKKKKKKKKKKKKKKKKKKKKKKKKKKKKKKK
KKKKKKKKKKKKKKK
>25) Mou received a call from a father how has a son diagnosed recently
with /&#I for siD months5 he said that he found his son lying down
unconscious in his bedroom5 7hat you will tell him if he is seeking for
advise:
a. "ring him as soon as ossible to 4%
b. +all the ambulance
c. Eive him his usual dose of insulin
d. Eive him I& Elucagone
e. Eive him Sugar in 8luid er oral
The correct answer is e
KKKKKKKKKKKKKKKKKKKKKKKKKKKKKKKKKKKKKKK
KKKKKKKKKKKKKKKKK..
>2:) factor which determine recurrence of breast cancer :
a# Site J siGe of breast mass
b# ,o of lymh nodes
c# -ositive estrogen recetor
d# -ositive rogesterone recetor
The correct answer is b
KKKKKKKKKKKKKKKKKKKKKKKKKKKKKKKKKKKKKKK
KKKKKKKKKKKKKKKKK..
>2=) teacher in school resented with ) days FD of 6aundice ( they wrote it
ictric as if it will a differacne) J abdominal ain 5 ,.@ 5 2 of school
student had the same illness in lab what is true regarding this t :
a# -ositive for heatitis ' IgE
b# -ositive heatitis ' Ig&
c# -ositive heatitis " core
d# -ositive heatitis " c anti#body
The correct answer is b
>2>) regarding weight gain in regnancy what is true :
a# -regnant woman should consume an average calorie )33#533 er
day
b# %egardless her "&I or body weight she should gain from 1.5 K ) lb
which reresent the babySs growth
c# $$$$$$$$$$$$$$$$$$$$$$..
d# $$$$$$$$$$$$$$$$$$$$$$$$.
A is not comlete
7eight gain during regnancy :
- 133 K )33 Ncal 0 day 5 533 Ncal 0 day in breastfeeding
- 7t. gain : 1 K 1.5 kg 0 month 5 11 K 1: kg gain during regnancy.
( for chane from Ib to = ! divided b" 1.1)
KKKKKKKKKKKKKKKKKKKKKKKKKKKKKKKKKKKKKKK
KKKKKKKKKKKKKKKKK..
>2C) what condition is an absolute contraindication of lactation :
a# &other with oen ulmonary !" for ) month
b# Feres Goster in !13 drematome
c# 'symtomatic FI@
d# $$$$$$$$$$$$$$$$$.
The correct answer is c
KKKKKKKKKKKKKKKKKKKKKKKKKKKKKKKKKKKKKKK
KKKKKKKKKKKKKKKKK..
>53) what is most sensitive for /H of duodenal ulcer :
a# 4igasteric ain starting )3#:3 min after the meal
b# 4igasteric ain staring immediately after a meal
c# Increasing of ain when lying suine
d# -ain radiating to the back
The correct answer is d
KKKKKKKKKKKKKKKKKKKKKKKKKKKKKKKKKKKKKKK
KKKKKKKKKKKKKKKKK..
>51) female at 23 yr with "&I Q2> what is your management :
a# %educe calorie in take to >33 0day
b# In general reduce calorie intake
c# $$$$$$$$$$$$$.
d# $$$$$$$$$$$$$$$..
2 is not complete* but with these 3+2s * the correct answer is b
>52) =3 y0o female say that she lay uGGle but for a short eriod she canSt
lay because as she develo headache when laying what u will eDam
for
a. astigmatism
b. glaucoma
the correct answer is a
KKKKKKKKKKKKKKKKKKKKKKKKKKKKKKKKKKKKKKK
KKKKKKKKKKKKKKKKK..
>52) rohylaDis for meningitis ttt contact
a. cimitidine
b. rifamicin
the correct answer is b
KKKKKKKKKKKKKKKKKKKKKKKKKKKKKKKKKKKKKKK
KKKKKKKKKKKKKKKKK..
>5))rheumatic fever t has stretococcal haringitis risk to develo
another attack
a. trimesmore than normal
b. 1339
c. 539
the correct answer is c
KKKKKKKKKKKKKKKKKKKKKKKKKKKKKKKKKKKKKKK
KKKKKKKKKKKKKKKKK..
>52)child with unilateral nasal obstruct with bad odor
a. unilateral adenoid hyertrohy
b. 8"
the correct answer is b
KKKKKKKKKKKKKKKKKKKKKKKKKKKKKKKKKKKKKKK
KKKKKKKKKKKKKKKKK..
>55)-!. do colectomy for colon cancer routine follow u every
a. : months
b. ) months
c. C months
d. 1 years
the correct answer is d
>5:) for atient with osteoarthritis 5 what is your initial action :
a. Intraarticular corticosteroid
b. Auadrices strengthening eDercise
c. +limb stairs eDercise
The correct answer isb
KKKKKKKKKKKKKKKKKKKKKKKKKKKKKKKKKKKKKKK
KKKKKKKKKKKKKKKKK..
>5=) =3#y old female atient with osteoorosis 5 what is her ! score :
a. (#2.5)
b. (#1)
c. (1)
d. (2)
The correct answer is a
KKKKKKKKKKKKKKKKKKKKKKKKKKKKKKKKKKKKKKK
KKKKKKKKKKKKKKKKK..
>5> ) diabetic atient 5 diagnosed 2 weeks back came to your clinic at
scheduled aointment suosed to be at 13:33 '& but because you
were having another comlicated case 5 he had to wait for more than an
hour 5 and he was eDtremely angry 5 what u will do :
a. be emathetic as this anger is mostly because of the new morbidity
diagnosed at this atient
b.you start your talk with him by saying ;I was having a hard case ;
c. /on?t say anything regarding being late unless he brings it u
d.you star M(*% !'BN 7I!F FI& "M S'MI,F ;you seem furious<
KKKKKKKKKKKKKKKKKKKKKKKKKKKKKKKKKKKKKKK
KKKKKKKKKKKKKKKKK..
>5C) 1
st
ste in eidemic study is :
a. verifying diagnosis
2 is not complete
>:3) %andomiGed control trials become stronger if :
a. you follow more than 539 of those in the study
b. systematic assignment redictability by articiants
the correct answer is b
KKKKKKKKKKKKKKKKKKKKKKKKKKKKKKKKKKKKKKK
KKKKKKKKKKKKKKKKK..
>:1) child with multile ainful swellings on the dorsum of hands 5 feet 5
fingers and toes 5his +"+ showed Fb X=5%"+?s on eriheral smear are
crescent shaed 5 what is your long#term care :
a. corticosteroids
b. enicillin @
c. antihistaminic
the correct answer is b * this pt. is sic#le cell anemia
KKKKKKKKKKKKKKKKKKKKKKKKKKKKKKKKKKKKKKK
KKKKKKKKKKKKKKKKK..
>:2) 52 y old atient 5 farmer 5 coming comlaining of dry eye 5 he is smoker
for 23 years and smokes 2 acks0 day 5 your recommendation :
a. advise him to eDercise
b. sto smoking
c. wear sunscreen
The correct answer is b * smo#in increase ris# of dr" e"e (international
stud" )
KKKKKKKKKKKKKKKKKKKKKKKKKKKKKKKKKKKKKKK
KKKKKKKKKKKKKKKKK..
>:)) atient is wearing contact lenses for vision correction since ten years 5
now coming c0o eDcessive tearing when eDosed to bright light 5 what
will be your advice to him :
a. wear hat
b. wear sunglasses
c. remove the lenses at night
d. saline eye dros 2 times 0 day
The correct answer is d

>:2) management of mild eistaDis :
a. inching the fleshy art of nose while telting head foreword
KKKKKKKKKKKKKKKKKKKKKKKKKKKKKKKKKKKKKKK
KKKKKKKKKKKKKKKKK..

>:5) most common site of malignancy in aranasal sinuses :
a. &aDillary sinus ( correct )
KKKKKKKKKKKKKKKKKKKKKKKKKKKKKKKKKKKKKK
KKKKKKKKKKKKKKKKKK..
>::) atient with hyertension 5 what is the best non#harmacological
method to lower the elevated blood ressure :
a. weight reduction ( correct )
KKKKKKKKKKKKKKKKKKKKKKKKKKKKKKKKKKKKKKK
KKKKKKKKKKKKKKKKK..
>:=) newly diagnosed atient with hyer tension having ,aX12= 5 NX) 5
what is the most likely cause of his secondary hyertension :
a. hyeraldosteronism ( correct )
KKKKKKKKKKKKKKKKKKKKKKKKKKKKKKKKKKKKKKK
KKKKKKKKKKKKKKKKK
>:>) 53 year old atient 5 diagnosed with hyertension 5 he is used to drink
one glass of wine every day 5 he is also used to get high ,a and high N
intake 5 his "&I is )3kg0m 5 what is the strongest risk factor for having
hyertension in this atient :
a. wine
b. high ,a intake
c. high N intake
d. "&IX)3
the correct answer is d
>:C) all of the following tests are necessary to be done before initiating
lithium eDcet:
a. liver function tests
KKKKKKKKKKKKKKKKKKKKKKKKKKKKKKKKKKKKKKK
KKKKKKKKKKKKKKKKK..
>=3) in dementia 5 best drug to use :
a. haloeridol
b. galantamine
the correct answer is b
treatment of dementia is cholinesterase inhibitor ( alantamine* donepe7il*
rivastimine* and tracin )
KKKKKKKKKKKKKKKKKKKKKKKKKKKKKKKKKKKKKKK
KKKKKKKKKKKKKKKKK..
>=1) most imortant oint to redict a rognosis of SB4 atient :
a. degree of renal involvement
b. seD of the atient
c. leucocyte count
the correct answer is a
KKKKKKKKKKKKKKKKKKKKKKKKKKKKKKKKKKKKKK
KKKKKKKKKKKKKKKKKK..
>=2) what is the antibiotic that you will give if you will do aendectomy :
a. doDycycline
b. ceftriaDone
c. metronidaGole
d. cefuroDime
!he correct answer is d


>=)) most common site for ectoic regnancy :
a. falloian tubes
KKKKKKKKKKKKKKKKKKKKKKKKKKKKKKKKKKKKKK
KKKKKKKKKKKKKKKKKK..
>=2)child with low grade fever 5 rash 5 haringitis 5 and tender
slenomegally 5 .ve 4"@ 5 /D :
a. infectious mononeucleosis
KKKKKKKKKKKKKKKKKKKKKKKKKKKKKKKKKKKKKK
KKKKKKKKKKKKKKKKKK..
>=5) best early sign to detect tension neumothoraD :
a. tracheal shift
b. distended neck veins
c. hyotensionv
the correct answer is b * ICm not sure * I thin# tracheal shift is the late
sin .
KKKKKKKKKKKKKKKKKKKKKKKKKKKKKKKKKKKKKKK
KKKKKKKKKKKKKKKKK..
>=:) 8irst sign of magnisium sulfate toDicity is :
a. Boss of dee tendon refleD
KKKKKKKKKKKKKKKKKKKKKKKKKKKKKKKKKKKKKKK
KKKKKKKKKKKKKKKKK..
>==) -atient with mashroom toDicity will resent with
a. +onstiation
b. Fallucination
c. 'nhydrosis
the correct answer is b
>=>) +hild ate overdose of iron 5 best immediate management
a. Eastric lavage
b. Induce vomiting manually
c. 4metic drugs
d. Iecac
the correct answer is a
KKKKKKKKKKKKKKKKKKKKKKKKKKKKKKKKKKKKKKK
KKKKKKKKKKKKKKKKK..
>=C) Status eileticus is :
a. +ontinuous siGure activity more than )3 min without regaining
consciousness
KKKKKKKKKKKKKKKKKKKKKKKKKKKKKKKKKKKKKK
KKKKKKKKKKKKKKKKKK..
>>3) 8irst sign of increased I+- is :
a. contralateral aillary dilatation
b. Isilateral aillary dilatation
c. /ecrease level of consciousness
the correct answer is c
KKKKKKKKKKKKKKKKKKKKKKKKKKKKKKKKKKKKKKK
KKKKKKKKKKKKKKKKK..
>>3) -atient with history of head trauma unable to move the fork to his
mouth 5 where is the lesion:
a. cerebellum
b. frontal lobe
c. temoral
the correct answer is a
KKKKKKKKKKKKKKKKKKKKKKKKKKKKKKKKKKKKKKK
KKKKKKKKKKKKKKKKK..
>>1) -atient with lumbar canal stenosis 5 I forget the resentation 5 what
is the ttt:
a. -hysiotheray
b. Bumbar dicsectomy
2 IS N&T +&3P45T5
&ild to moderate : ,S'I/ . hysiotheray
'dvance : eidural corticosteroid
%efractory : surgical laminectomy
>>2) &ost common cause of otorrhia :
a. +sf leak
b. 'cute otitis media
c. +erumen leak
The correct answer is b
KKKKKKKKKKKKKKKKKKKKKKKKKKKKKKKKKKKKKKK
KKKKKKKKKKKKKKKKK..
>>)) &ost common symtoem of acute otitis media :
a. -ain
b. /ischarge
c. !innitus
d. @ertigo
The correct answer is a
KKKKKKKKKKKKKKKKKKKKKKKKKKKKKKKKKKKKKKK
KKKKKKKKKKKKKKKKK..
>>2) 12 yo boy is mocked at school because he is obese 5 ate a lot of ill
to slee and never wake u again 5 best management is :
a. %efer to mental rofessional
b. !ell him that most kid grow out before they grow u
c. 'dvice healthy food
KKKKKKKKKKKKKKKKKKKKKKKKKKKKKKKKKKKKKKK
KKKKKKKKKKKKKKKKK..
>>5) 12 yo boy brought by his arent for routine evaluation 5 his is obese
but otherwise healthy 5 his arents want to measure his cholesterol
level 5 what is the best indicator of measuring this child cholesterol :
a. Fis arent desire
b. 8amily hD of early cva
c. Figh bmi
The correct answer is b
>>:) /iabetic t. have neovasclariGation and vetrous hemorrhage 5 neDt
ste :
a. %efer to ohthalmologist
KKKKKKKKKKKKKKKKKKKKKKKKKKKKKKKKKKKKKKK
KKKKKKKKKKKKKKKKK..
>>=) Fistoatholgy of resected inflamed aendiD will show :
a. ,eutrohil in muscular wall
b. Bymhoid hyerlasia with multineucleated gaint cel
The correct answer is a
KKKKKKKKKKKKKKKKKKKKKKKKKKKKKKKKKKKKKKK
KKKKKKKKKKKKKKKKK..
>>>) ' child had bee bite resented after 1> hour with left arm erythema
and itching and$. Bong scenario 5 what to do:
a. 'ntihistaminic
b. (ral steroid
c. Subcutaneous einehrine
The correct answer is a
KKKKKKKKKKKKKKKKKKKKKKKKKKKKKKKKKKKKKKK
KKKKKKKKKKKKKKKKK..
>>C) (ld man with urinary incontinence 5 laable bladder after voiding 5
urgency and sense of incomlete voiding dDV
a. Stress incontenece
b. (verflow
c. %efleD
The correct answer is b
KKKKKKKKKKKKKKKKKKKKKKKKKKKKKKKKKKKKKKK
KKKKKKKKKKKKKKKKK..
>C3) !tt of scabis :
a. -ermethrin
>C1) -icture 5 hyerkeratotic 5 scaly lesion over the eDtensor surface of
knee and elbow 5 what to do to avoid eDacerbation :
a. Steroid
b. 'void sun eDosure
c. 'void trauma
The correct answer is a
KKKKKKKKKKKKKKKKKKKKKKKKKKKKKKKKKKKKKKK
KKKKKKKKKKKKKKKKK..
>C2) 7hich is true about allouranol :
a. Eood if given during acute gout
b. *ricisoric
c. %educe the chance of uric acid stone
d. +an be antagoniGe by salysilate
The correct answer is c
KKKKKKKKKKKKKKKKKKKKKKKKKKKKKKKKKKKKKKK
KKKKKKKKKKKKKKKKK..
>C)) man walking in street and saying bad words to stranger 5 he is not
aware of his conditiond 5 what is the descrition :
a. flight of idea
b. insertion of idea
c. loosening of association
KKKKKKKKKKKKKKKKKKKKKKKKKKKKKKKKKKKKKKK
KKKKKKKKKKKKKKKKK..
>C2) which of the following suggest that thyroid nodule is bengin rather
that malignant:
a. history of childhood head and neck radation
b. hard consistency
c. lemhadenoathy
d. resense of multile nodules
the correct answer is d
th"roid cancer ! ( cold and solid nodule )
>C5) child resented after sting bite with abd ain 5 arthalgia 5 lower
eDtremity and buttock rash 5 normal %8! 5 .ve fecal occult stool 5 dD :
a. FS-(human scal arasit)
b. Byme diases
The correct answer is a
KKKKKKKKKKKKKKKKKKKKKKKKKKKKKKKKKKKKKKK
KKKKKKKKKKKKKKKKK..
>C:) best indicator for labor rogress is :
a. freTuency of contractions
b. strength of contractions
c. descent of the resenting art
the correct answer is c
KKKKKKKKKKKKKKKKKKKKKKKKKKKKKKKKKKKKKKK
KKKKKKKKKKKKKKKKK..
>C=) lactating mother is taking -henobarbital for siGure rohylaDsis 5
what to advice her :
a. sto breast feeding immediately
b. breast feed after taking the ill by > hour
c. start weaning
d. does not matter
the correct answer is a
KKKKKKKKKKKKKKKKKKKKKKKKKKKKKKKKKKKKKKK
KKKKKKKKKKKKKKKKK..
>C>) best single way to reduce high blood ressure is :
a. smoke cessation
b. decrease liid level
c. reduce weight
the correct answer is c
KKKKKKKKKKKKKKKKKKKKKKKKKKKKKKKKKKKKKKK
KKKKKKKKKKKKKKKKK..
>CC) drug of choice for suraventricular tachycardia is :
a.adenosine

C33) atient with blood grou ' had blood transfusion grou " 5 the best
statement that describe the result is :
a. tye I@ hyersensitivity
b. Faemolytic inflammatory reaction $.
c. $$$.
The correct answer is b! It is t"pe 1 h"persensitivit"
KKKKKKKKKKKKKKKKKKKKKKKKKKKKKKKKKKKKKKK
KKKKKKKKKKKKKKKKK..
C31) a nurse gave blood transfusion through +@- 5 2 hours later the
atient is comatose and unresonsive 5 dD :
a. Setic shock
b. "lood grou mismatch
The correct answer is a
KKKKKKKKKKKKKKKKKKKKKKKKKKKKKKKKKKKKKKK
KKKKKKKKKKKKKKKKK..

C32) 1339 (2 given for rolonged eriods can cause all eDcet:
a. %etrosternal -ain
b. SeiGures
c. /eression
d. (cular !oDicity
e. OOO
The correct answer is c ( ICm not sure because choice e is not
mention )
C3)) (rganohoshorus oisoning5 what is the antidoteO
a. 'troine
b. -hysostigmine
c. ,eostigmine
d. -ilocarine
e. 4ndrohonium
The correct answer is a
KKKKKKKKKKKKKKKKKKKKKKKKKKKKKKKKKKKKKKK
KKKKKKKKKKKKKKKKK..
C32) 7hich of the following ulse character goes with the diseaseO
a. +ollasing ulse Severe anemia
b. -ulsus 'lternans -remature ventricular comleDes
c. Slow rising -ulse &itral stenosis
d. -ulsus "isferens &itral %egurgitation
e. -ulsus aradoDus I think 'ortic stenosis or regurgitation
The correct answer is a
-B*S4 /48I,I!I(, +'*S4S
+ollasing ulse Pulse with fast upstro#e and fast
downstro#e
0- Sever anemia
1- %ortic reure
I- Th"roto$icosis
-ulse alternans %ltrnans wea# and stron pulse 0- 4V d"sfunction
-ulsus aradoDus .ecline s"stolic more than 0; mm
/ durin inspiration
0- +ardiac
tamponade
1- %sthma > +&P.
-ulsus arvus et
tardus
Eea# and dela"ed pulse 0- %ortic stenosis
Slow rising ulse Slow upstro#e pulse 0- %ortic stenosis
-ulsus bisferiens pulse characteri7ed b" two stron
s"stolic pea#s separated b" a
mids"stolic dip
0- %ortic reuire
1- %R A %S
C35) 4ntamoeba histolytica cysts are destroyed best by:
a. "oiling
b. Iodine added to water
c. +hlorine added to water
d. 8reeGing
The correct answer is a
KKKKKKKKKKKKKKKKKKKKKKKKKKKKKKKKKKKKKKK
KKKKKKKKKKKKKKKKK..
C3:) ' scenario about an old male with symtoms suggesting
arkinsonism such as difficulty walking5 resting tremors and rigidity in
addition to hyotension. !hen he asks about what is the most common
resenting symtom of this disease
a. %igidity
b. !remors
c. *nsteady Eait
d. Fyotension
e. OOO
The correct answer is b
KKKKKKKKKKKKKKKKKKKKKKKKKKKKKKKKKKKKKKK
KKKKKKKKKKKKKKKKK..
C3=) %egarding ostartum -sychosis:
a. %ecurrences are common in subseTuent regnancies
b. It often rogresses to frank schiGohrenia
c. It has good rognosis
d. It has insidious onset
e. It usually develos around the )
rd
week ostartum
The correct answer is a
KKKKKKKKKKKKKKKKKKKKKKKKKKKKKKKKKKKKKKK
KKKKKKKKKKKKKKKKK..
C3>) ' regnant female develos lesions on the vulva and vagina and
she was diagnosed as genital heres5 what should be included in her
future health careO
a. +esarian section should be done if the lesions did not disaear
before 2 weeks of delivery date
b. (ral acyclovir to treat heres
c. !ermination of regnancy because of the risk of fetal
malformations
d. 'voidance of seDual intercourse for 1 month after the healing of
the lesions
e. OOOOO
The correct answer is b
FS@ in regnant treated by ! oral ac"clovir R;; m TI. for Q-D da"s . if
FS@ was resent at time of labor ! c - section
C3C) ' icture of 2 warts in the indeD finger.
/iagnosis is @iral 7arts
KKKKKKKKKKKKKKKKKKKKKKKKKKKKKKKKKKKKKKK
KKKKKKKKKKKKKKKKK..
C13) 7hat is the most common seTualae of acute ancreatitis O
a. 'bscess
b. -seudocyst
c. "owel obstruction
The correct answer is b
&ost common comlication of acute ancreatitis! pancreatic
pseudoc"t and fistula formation
KKKKKKKKKKKKKKKKKKKKKKKKKKKKKKKKKKKKKKK
KKKKKKKKKKKKKKKKK..
C11) 'n old atient with history of cerebrovascular disease J Ischemic
heart disease5 resents with a attern of breathing described as: '
eriod of anea followed by slow breathing which accelerates J
becomes raid with hyernea J tachycardia then anea again. 7hat is
this tye of breathingO
a. Fiocrates
b. +hyene#stokes breathing
c. Nussmaul breathing
d. (ne tye beginning with ( letter and contains ) letters only
e. OOOO
The correct answer is b
1# +hyene#stokes resiration : rapid deep breathin phase followed b"
period of apnea * present with heart failure * stro#e* brain trauma * also can
be with sleep or hih altitude
2#kusmmaulSs breathing : rapid and deep breathin . present with
metabolic acidosis particularl" in diabetic #etoacidosis
C12) ' atient with miDed 1
st
J 2
nd
degree burns in head J neck region5
what is the most aroriate managementO
a. 'ly silver sulfadiaGine and cream to all burned areas5cover them
and admit to hosital
b. 'ly cream to 2
nd
degree burns and cover them5 give I@ fluids
c. /ebridement of 2
nd
degree burns and $.
d. 'ly silver sulfadiaGine then @aseline ointment to all areas then
discharge the atient
The correct answer is a
1sr degree ! epidermis A painful ( no blister )
2
nd
degree ! epidermis A partial dermis A painful [ blister
)
rd
degree ! epidermis A full thic#ness dermis A painless
%ule of nine to estimate "S' :
0- /ead and nec# ! U<
1- 5ach upper arm ! U<
I- 5ach lower limb ! 0:<
R- 5ach post. > ant. Trun# ! 0:<
Q- Perineum > enitalia ! 0<
-Silver sulfadia7ine and mafenide used for all t"pes of burn
-supportive ! tetanus and I.V nacrotic can be used for all t"pes of burn
- I.V fluid use onl" in ( N 0;< of 6S%) in 1
nd
> I
rd
deree of burn accordin
par#land formula ! ( fluid in first 1Rh V R \ wt. in # \ 6S% . Q;< in first :
hours and another in ne$t 0P hours )
In this patient 0
st
deree and 1
nd
deree T 0;< ( head and nec# U<) ! so*
not use fluid
KKKKKKKKKKKKKKKKKKKKKKKKKKKKKKKKKKKKKKK
KKKKKKKKKKKKKKKKK..

C1)) 2 years old child with ear ain J bulging tymanic membrane5 what
is the diagnosisO
a. (titis media
b. (titis eDterna
c. (tomycosis
d. "ullous myringitis
The correct answer is a
C12) "esides I@ fluids5 what is the most imortant drug to be given in
anahylaDisO
a. 4inehrine
b. Steroids
c. OOO (ther choices
The correct answer is a
KKKKKKKKKKKKKKKKKKKKKKKKKKKKKKKKKKKKKKK
KKKKKKKKKKKKKKKKK..
C15) 'n adult atient in 23s or )3s of age resents by history of 1 month
of fever5 5 days of headache J 2 days of altered sensorium. (n
eDamination there is nuchal rigidity5 then there is a table showing
investigations which include
Fb: 13 g0dl
"lood 7"+: 1>5333
+S8 4Damination: 7"+s elevated: ==9 lymhocytes5 ))9 ,eutrohils.
-rotein OOO. Elucose)3mg
7hat is the diagnosisO
a. @iral meningioencehalitis
b. !uberculous meningitis
c. -h$. &eningitis
d. OOO
2 is not complete * but with this scenario and 3+2s * The correct
answer is b
C1:) %egarding chronic fatigue syndrome5 which is trueO
a. 'ntibiotics may reduce the symtoms
b. 'ntideressants may reduce the symtoms
c. %est may reduce the symtoms
d.
e. OOOO
the correct answer is b.
chronic fatigue syndrome:
characteri7es b" profound mental and ph"sical e$haustion. In associated
with multiple s"stem and neurosps"chiatric s"mptoms that last at least P
mounth. 3ust be new ( not life lon ) * must not be relived b" rest* and
must result in reater than Q;< reduction in previous activit".
Presentation with R or more of the followin ! poor memor" O
concentration* m"alia* arthalaia* sore throat* tender l"mph node* recent
onset headach* unrefreshin sleep* e$cessive tiredness with e$ercise.
Treatment b" ! conitive and e$cercise therap" .also* diet* ph"siotherap" *
dietar" supplements * antidepressants .
KKKKKKKKKKKKKKKKKKKKKKKKKKKKKKKKKKKKKKK
KKKKKKKKKKKKKKKKK..
C1=) ' atient comlains of dry eyes5 a moisturiGing eye dros were
rescribed to him 2 times daily. 7hat is the most aroriate method
of alication of these eye drosO
a. 1 dro in the lower forniD
b. 2 dros in the lower forniD
c. 1 dro in the uer forniD
d. 2 dros in the uer forniD
KKKKKKKKKKKKKKKKKKKKKKKKKKKKKKKKKKKKKKK
KKKKKKKKKKKKKKKKK..
C1>) ' atient with enetrating abdominal stab wound. @itals are: F% C>5
"- 1230>35 %% 1>. ' art of omentum was rotruding through the
wound. 7hat is the most aroriate neDt ste:
a. 8%ST Bltrasound
b. .P4 (.ianostic peritoneal lavae)
c. 5$plore the wound
d. %rrane for a +T Scan
e. 5$plorator" laparotom"
The correct answer is e
C1C) ' atient resents with loin ain radiating to the groin. %enal stones
are susected. 7hat is the test that has the most secificity J
sensitivity in diagnosing this conditionO
a. ,oncontrast siral +! scan of the abdomen
b. *ltrasound
c. N*"
d. Intravenous yelograhy (I@-)
e. ,uclear Scan
The correct answer is a
KKKKKKKKKKKKKKKKKKKKKKKKKKKKKKKKKKKKKKK
KKKKKKKKKKKKKKKKK..
C23) 'n old atient resents with history diGGiness J falling down 1 day
ago accomanied by history of 4igastric discomfort. Fe has very high
tachycardia I think around 1)3#123 and "- 1330:3. 7hat is the
diagnosisO
a. -etic ulter
b. E4%/
c. Beaking aortic aneurysm
d. OOO
The correct answer is c
KKKKKKKKKKKKKKKKKKKKKKKKKKKKKKKKKKKKKKK
KKKKKKKKKKKKKKKKK..
C21) ' child is about to be given flu vaccine5 what allergy should be
eDcluded before giving the vaccineO
a. +hicken
b. egg
c. 8ish
d. OOO
The correct answer is b
KKKKKKKKKKKKKKKKKKKKKKKKKKKKKKKKKKKKKKK
KKKKKKKKKKKKKKKKK..
C22) 7hich of the following is roven to reduce the incidence of cancerO
a. Salt
b. &ineral water
c. @itamin /
d. 8iber
The correct answer is c
C2)) ' female atient with history of cyclic abdominal ain5 inability to
conceive5 heavy menses5 and eDamination showed tenderness J
nodularity in uterosacral ligaments. 7hat is the diagnosisO
a. 4ndometriosis
KKKKKKKKKKKKKKKKKKKKKKKKKKKKKKKKKKKKKKK
KKKKKKKKKKKKKKKKK..
C22) ' female atient on the )
rd
week ostartum. She says to the
hysician that the freTuently visualiGes snakes crawling to her baby?s
bed. She knows that it is imossible but she cannot remove the idea
from her head. She says she wakes u around 53 times at night to
check her baby. !his roblem revents her from getting good slee and
it started to affect her marriage. 7hat is this roblem she is
eDeriencingO
a. 'n obsession
b. ' hallucination
c. ' ostartum sychosis
d. ' /elusion
The correct answer is a
(bsession : persistent* unwanted* and intrusive ideas * thouhts*
impulses or imaes
KKKKKKKKKKKKKKKKKKKKKKKKKKKKKKKKKKKKKKK
KKKKKKKKKKKKKKKKK..
C25) %egarding screening for cancer5 which of the following is trueO
a. Screenin for cervical cancer had decreased in recent "ears
b. Screenin for breast cancer had decreased in recent "ears
c. Screenin for +olorectal cancer is inade,uate for the hih-ris# roups
d. Screenin for lun cancer has reduced the mortalit" rate of lun cancer
e. Screenin for tobacco use is now ade,uatel" done b" health
professionals
KKKKKKKKKKKKKKKKKKKKKKKKKKKKKKKKKKKKKKK
KKKKKKKKKKKKKKKKK..
C2:) %egarding ostartum deression5 what is the most aroriate
intervention to reduce the symtomsO
a. Include family in the theray
b. Isolation theray
c. 'dd very low doses of imiramine
d. 4ncourage breastfeeding
The correct answer is a
C2=) 7hich of the following is a side effect of "urione5 adrug used to
hel smoking cessation:
a. 'rrhythmia
b. Herostomia
c. Feadache
d. SeiGure
The correct amswer is d
KKKKKKKKKKKKKKKKKKKKKKKKKKKKKKKKKKKKKKK
KKKKKKKKKKKKKKKKK..
C2>) ' :3 years old atient with history of heart attack : weeks ago5
comlaining of not getting enough slee. -sychiatric evaluation is
unremarkable for deression or anDiety5 what should be given to this
atientO
a. %m"triptiline
b. 6uspirone
c. 6uprione
d. Golbidim
The correct answer is d
KKKKKKKKKKKKKKKKKKKKKKKKKKKKKKKKKKKKKKK
KKKKKKKKKKKKKKKKK..
C2C) ' atient is asked to face the wall5 bend his waist5 and let his hands
hang down without suort. !his test is used as a screening tool for
which of the followingO
a. Bower limb asymmetry
b. %ectal rolased
c. Scoliosis
d. OOOO
The correct answer is c
This test is called for ('damSs 8orward "end !est )

C)3) ' atient resents with sore throat5 cervical lymhadenoathy5 mild
slenomegaly5 4"@ antibodies are .ve. 7hat is the diagnosisO
a. Infectious &ononucleosis (4"@ )
KKKKKKKKKKKKKKKKKKKKKKKKKKKKKKKKKKKKKKK
KKKKKKKKKKKKKKKKK..
C)1) 'n old atient resents with knee ain5 limitation of movement and
Tuadrices muscle atrohy. (n eDamination there is limited range on
movement5 ain on movement J +reitus. 7hat is the diagnosisO
a. (steoarthritis
b. %heumatoid arthritis
c. 'nkylosing Sondylitis
The correct answer is a
KKKKKKKKKKKKKKKKKKKKKKKKKKKKKKKKKKKKKKK
KKKKKKKKKKKKKKKKK..
C)2) ' atient resents with long time history of knee ain suggestive of
osteoarthritis. ,ow he comlains of unilateral lower limb swelling and
on eDamination there is .ve edal J tibial itting edema. 7hat is the
neDt aroriate investigationO
a. +H%
b. 4+E
c. 4chocardiograhy
d. /uleD ultrasound of lower limb
The correct answer is d
(steoarthritis relief by rest . So5 immobility t. can lead to /@!
KKKKKKKKKKKKKKKKKKKKKKKKKKKKKKKKKKKKKKK
KKKKKKKKKKKKKKKKK..
C))) 'loecia is a side effect of which antieileticO
a. -henytoin
b. +arbamaGeine
c. ,a @alroate
d. OOO
The correct answer is c
1#henytoin ! inival h"perplasia* hirsuteism* ata$ia
2#carbamaGeine ! aranuloc"tosis* hepatoto$icit"* aplastic anemia
)# ,a @alroate ! transient hair loss.

C)2) 7hich of the following is an indication for tonsillectomyO
a. Slee anea
b. 'symtomatic large tonsils
c. -eriharygeal abscess
d. %etroharyngeal abscess
The correct answer is a
KKKKKKKKKKKKKKKKKKKKKKKKKKKKKKKKKKKKKKK
KKKKKKKKKKKKKKKKK..
C)5) 7hich of the following is a gram Kve rods that grow on charcoal
yeast agarO
a. Begionella
KKKKKKKKKKKKKKKKKKKKKKKKKKKKKKKKKKKKKKK
KKKKKKKKKKKKKKKKK..
C):) ' = years old child had history of chest infection which was treated
with antibiotics. !he atient resented : weeks after cessation of
antibiotics with abdominal ain5 fever and rofuse watery diarrhea for
the ast month. 7hich of the following organisms is resonsible for the
atient?s conditionO
a. Eiardia Bamblia
b. +lostridium /ifficile
c. 4scherichia coli
d. +lostridium -erfringens
The correct answer is b
KKKKKKKKKKKKKKKKKKKKKKKKKKKKKKKKKKKKKKK
KKKKKKKKKKKKKKKKK..
C)=) 7hich of the following features of ulcerative colitis distinguishes it
from crohn?s disease
a. -ossible malignant transformation
b. 8istula formation
c. 'bsence of granulomas
d. +olon involvement
The correct answer is a
C)>) ' very very long scenario about a female atient with vaginal
discharge ;malodorous watery in character< with F of : J .ve clue
cells but there is no branching seudohyhe. (Fe is telling you the
diagnosis is vaginosis J there is no fungal infection) !hen he asks
about which of the following drug regimens should ,(! be used in this
aitent:
a. &etronidaGole (-( 533 gm for = days)
b. &etronidaGole (-( 2 large dose tablets for 1 or 2 days)
c. &etronidaGole (I@ or I& $..)
d. &iconaGole ( -( $..)
e. +lindamycin (-( $..)
The correct answer is d
This case is F bacteria vaginosisF and treatment b" ! metradina7ole or
clindam"cine . avoid micona7ole because it is antifunal
KKKKKKKKKKKKKKKKKKKKKKKKKKKKKKKKKKKKKKK
KKKKKKKKKKKKKKKKK..
C)C) 7hat is the condition that roduces malodorous watery vaginal
discharge with .ve clue cells in wet mount rearation slides:
a. "acterial vaginosis
b. @aginal +andidiasis
c. !richomonas vaginalis
d. Eonorrhea
The correct answer is a
KKKKKKKKKKKKKKKKKKKKKKKKKKKKKKKKKKKKKKK
KKKKKKKKKKKKKKKKK..
C23) 7hich of the following is the best treatment for Eiardiasis:
a. &etronidaGole
KKKKKKKKKKKKKKKKKKKKKKKKKKKKKKKKKKKKKKK
KKKKKKKKKKKKKKKKK..
C21) ' woman with Cweeks Fistory of elevated erythematous wheals
overall her body 5 she also has li swelling. !here is no Fistory of
recent travel or food allergy or drug ingestion5 /iagnosis:
a. +hronic 'ngioedema J *rticaria
b. +ontact /ermatitis
c. Solar *rticaria
d. +holinergic *rticaria
e. +old *rticaria
The correct answer is a
+hronic urticaria ! if more than P months
Solar urticaria ! due to sunliht
+holinergic urticariae : due to brief increase in bod" temperature.
+old urtiaria : due to e$posure to cold
C22) ' 1= years old male with history of mild intermittent asthma. 'ttacks
occur once or twice weekly in the morning and no attacks at night.
7hat should be the initial drug to giveO
a. Inhaled short acting "2 agonist as needed
b. Inhaled high dose corticosteroid as needed
The correct answer is a
KKKKKKKKKKKKKKKKKKKKKKKKKKKKKKKKKKKKKKK
KKKKKKKKKKKKKKKKK..
C2)) ' child resented with sore throat. +ulture from the throat revealed
.ve meningiococci. !he atient is now asymtomatic. 7hich of the
following should be doneO
a. %eassurance
b. %ifamicin oral for = days
c. I& ceftriaDone 1 dose
d. +eftriaDone oral
e. OOOOOO
The correct answer is c ( ICm not sure )
1-da" reimen of oral rifampin or a sinle dose of I3 ceftria$one or a
sinle dose of oral ciproflo$acin).
KKKKKKKKKKKKKKKKKKKKKKKKKKKKKKKKKKKKKKK
KKKKKKKKKKKKKKKKK..
C22) 7hich of the following increases the Tuality of the randomiGed
controlled study J make it stronger:
a. Systemic 'ssignment redictability by articiants
b. (en 'llocation
c. Including only the articiants who received the full intervention
d. 8ollowing at least 53 9 of the articiants
e. Eiving similar intervention to similar grous
The correct answer is a
KKKKKKKKKKKKKKKKKKKKKKKKKKKKKKKKKKKKKKK
KKKKKKKKKKKKKKKKK..
C25) ' scenario about an infant who resented with difficulty breathing
and sweating5 eDamination shows hyeractive recordium with loud S2
and ansystolic murmur grade )0: best heard in the left )rd arasternal
region. 7hat is the /iagnosisO
a. @S/
b. &itral %egurgitation
c. -/'
d. Barge 'S/
The correct answer is a

C2:) -ost IF/ 5 your advise :
a. Moga
b. isotonic eDercise
???????????
KKKKKKKKKKKKKKKKKKKKKKKKKKKKKKKKKKKKKKK
KKKKKKKKKKKKKKKKK..
C2=) very obese girl 5 whatSs the first ste to reduce weight :
a. increase water intake
b. decrease calorie intake
the correct answer is b
KKKKKKKKKKKKKKKKKKKKKKKKKKKKKKKKKKKKKKK
KKKKKKKKKKKKKKKKK..

C2>) 23 y0o male 5 resented with large hands 5 heatomegaly $.. diagnosis
:
a. acromegaly
b. gigantism
the correct answer is a
KKKKKKKKKKKKKKKKKKKKKKKKKKKKKKKKKKKKKK
KKKKKKKKKKKKKKKKKK..
C2C) cousre of facial nerve when comes out from stylomastoid $
a. /ee to arotid gland J lateral to eDternal carotid J
retromandibular vein
KKKKKKKKKKKKKKKKKKKKKKKKKKKKKKKKKKKKKKK
KKKKKKKKKKKKKKKKK..
C53) old female with hemorrhoids for 13 years 5 no comlication 5 your
action O
a. observe
b. surgery
c. increase fiber diet
the correct answer is c
KKKKKKKKKKKKKKKKKKKKKKKKKKKKKKKKKKKKKKK
KKKKKKKKKKKKKKKKK..
C51) femal about )3y c0o abdominal ain related to menses (scenario
going with endometriosis)$$$$neDt ste in dD:
a. Baaroscoy
b. *0S
c. +!
The correct answer is a
+onfirm dianosis b" laparoscop" * BS > 3RI can use but if normal can
not e$clude endometriosis .
UQ1) female about I;" with breast cancer (iven cbc -chem. %nd reavel low
hb and hematocriteK.) what is the ne$t step in manment!
a. Stain
b. 4umpectom"
c. 3astectom"
d. +hemotherap"
KKKKKKKKKKKKKKKKKKKKKKKKKKKKKKKKKKKKKKK
KKKKKKKKKKKKKKKK.....

C5)) t with oen angle glaucoma and k0c of +(-/ and /& ttt:
a. !imelol
b. betaDolol
c. 'cetaGolamide
the correct answer is c
KKKKKKKKKKKKKKKKKKKKKKKKKKKKKKKKKKKKKKK
KKKKKKKKKKKKKKKK.....

C52) infant with runny nose and fever after that develo dysnea
5wheeGing with working accessory muscles and chest retractions with
cyanosis5 tem )> dD:
a. @iral numonia
b. "acterial neumonia
c. "ronchiolitis
The correct answer is c
C55) )2y female with FI@ a smear negative5 about cervical cancer
screening :
a. 'fter )m if negative reeat after :m
b. 'fter :m $$$$$$$$$$$ annually
c. 'fter 1y$$$$$$$$$$$$$.annually
The correct answer is c .
creening should begin no later than age !".
creening should begin earlier than age !" if the patient is sexually active. In this
case, it should start # years after initiation of vaginal intercourse.
$nce initiated, screening should be performed annually if a traditional, glass%
slide%based technique is used. If liquid medium is used, &ap smear screening may
be performed every other year.
'fter age #(, for women who have had # consecutive, normal &ap smears,
screening frequency may be reduced to every two to three years.
)omen who are *I+ positive, immunocompromised due to disease or
medication, or are ,- daughters, should continue annual screening.
creening may stop following a total hysterectomy (including the cervix), if the
the patient is at low risk, and has had three consecutive normal &ap smears within
the last "( years.
*igh risk patients, including those with a history of cervical cancer, ,-
exposure in%utero, *I+ positive, immunocompromised from medication, and
those tested positive for *&+, should continue to be screened indefinitely.
creening may stop after age .(, if the patient is low risk, and has had three
normal &ap smears over the last "( years.
creening may be omitted in the case of women with life%threatening or other
serious illness
KKKKKKKKKKKKKKKKKKKKKKKKKKKKKKKKKKKKKKK
KKKKKKKKKKKKKKKK.....
C5:) which drug increase incidence of refluD oesohagitis:
a. !heohylline
b. 'moDicillin
c. &etocloramide
d. %antidine
e. BansoraGole
The correct answer is a

C5=) most effective ttt of cluster headach:
a. 4rgotamine nebuliGer
b. S0+ Sumatritan
c. 1339 (2
d. I@ @eraamil
The correct answer is c
KKKKKKKKKKKKKKKKKKKKKKKKKKKKKKKKKKKKKKK
KKKKKKKKKKKKKKKK.....
C5>) old t. with F!, and migraine ttt:
a. " blockers
b. '+4 I
c. +a blockers
The correct answer is a
KKKKKKKKKKKKKKKKKKKKKKKKKKKKKKKKKKKKKKK
KKKKKKKKKKKKKKKK.....
C5C) lactating women 13 days after delivery develoed fever 5malaise5
chills tender Bt breast with hotness and small nodule in uer outer
Tuadrant with aDillary B, .Beukocyte count was 12 L130B dD:
a. Inflammatory breast cancer
b. "reast abscess
c. 8ibrocystic disease
d. OOOO
OOOOOOO
KKKKKKKKKKKKKKKKKKKKKKKKKKKKKKKKKKKKKKK
KKKKKKKKKKKKKKKK.....
C:3) newborn with fracture mid clavicle what is true:
a. &ost cases cause serious comlication
b. 'rm sling or figure > sling used
c. &ost atient heal without comlications
The correct answer is c
-most clavicles fracture in newborn no need to treatment apart from
careful handlin. If the fracture is displaced and bab" in pain*
simple slin is re,uire.
C:1) =3y male with osteoorosis the ! score of bone densometry would
be :
a. ).5
b. #2.5
c. #1
d. 2
e. ).5
The correct answer is b
- The T score is the number of standard deviation (S. ) a patientCs 63.
deviated from the mean.
- ! score Q (# 1) ! normal 63.
- ! score (#1 to #2.5) ! ostopenia
- ! score P ( #2.5 ) ! osteoprosis
KKKKKKKKKKKKKKKKKKKKKKKKKKKKKKKKKKKKKKK
KKKKKKKKKKKKKKKK.....
C:2) what is the most comlication after hesterectomy:
a. *reteral in6ury
b. -ulmonary embolism
c. Faemorrhage
The correct answer is c
KKKKKKKKKKKKKKKKKKKKKKKKKKKKKKKKKKKKKKK
KKKKKKKKKKKKKKKK.....
C:)) young female with "&I 1> 5 fine hair allover body 5 feeling of she is
fat 5doesn?t eat well with eDcessive eDercise$$$$$$..
a. 'noreDia nervosaO
b. "ody dysmorhic disorder
c. "ulimia nervosa
The correct answer is a
KKKKKKKKKKKKKKKKKKKKKKKKKKKKKKKKKKKKKKK
KKKKKKKKKKKKKKKK.....
C:2) whatSs advantage of mature human milk over cowSs milk:
a. &ore rotein
b. &ore Iron content
c. &ore calories
d. &ore fat
The correct answer is b
KKKKKKKKKKKKKKKKKKKKKKKKKKKKKKKKKKKKKKK
KKKKKKKKKKKKKKKK.....
C:5) young male c0o of deformity of 6aw .ast h0o of nasolasty and
bleharolasty (04 nothing abnormal $$$..dD:
a. "ody dysmorhic syndrome
C::) the most common cause of non#traumatic subarachnoid
haemorrhage:
a. %uture aneurysm
b. @essels abnormality
c. Fyertension
The correct answer is a
-3ost common causes of subarachnoid hemorrhae are trauma and
berr" aneur"sm
KKKKKKKKKKKKKKKKKKKKKKKKKKKKKKKKKKKKKKK
KKKKKKKKKKKKKKKK.....
UPD) -t. with chest ain and s.o.b decreased by leaning forword . (04
friction rub and increased W@- QQQQ (a case of ericardial effusion)
4+E will show:
a. S! changes
b. Bow voltage
c. Increase -% interval
5+@ chanes durin pericarditis ! diffuse ST elevation * PR depression
followed b" t-wave intervention * and low voltae.
KKKKKKKKKKKKKKKKKKKKKKKKKKKKKKKKKKKKKKK
KKKKKKKKKKKKKKKK.....
C:>) !he causative organism of sudomembranous colitis is:
a. +lostridium difficile
KKKKKKKKKKKKKKKKKKKKKKKKKKKKKKKKKKKKKKK
KKKKKKKKKKKKKKKK.....
UPU) scenario about arthritis (I couldnSt remember the details ) but Woint
asirate reveal Eram negative dilococci $..dD:
a. ,isseria gonorrhea
N. gonorrhoeae / gram negative diplococci and can also cause con0unctivitis,
pharyngitis, proctitis or urethritis, prostatitis and orchitis.
KKKKKKKKKKKKKKKKKKKKKKKKKKKKKKKKKKKKKKK
KKKKKKKKKKKKKKK.....
C=3) @ertigo J $$.. is caused by which of the following drug O
a. 'hotercine "
b. -nicillin reaction
c. I,F
KKKKKKKKKKKKKKKKKKKKKKKKKKKKKKKKKKKKKKK
KKKKKKKKKKKKKKK.....
C=1) "aby with red macule J dilated caillary on the
%t side of the face
"turge#7eber Syndrome or ,evus 8lammeus 7one o! them 'ill come.
Don8t choose milia or caverno%s haeman&ioma
C=2) ' 25 year old secondary school teacher that every time enters the
class starts sweating and having alitation5 she is a fired to give
wrong information and be unarsed. 7hat is the diagnosis:
a. Secific -hobia
b. Social -hobia
The correct answer is b
KKKKKKKKKKKKKKKKKKKKKKKKKKKKKKKKKKKKKKK
KKKKKKKKKKKKKKK.....
C=)) 't which chromosome is the cystic fibrosis gene:
a. Bong arm chromosome =
b. Short arm chromosome =
c. Bong arm chromosome >
d. Short arm chromosome >
e. Bong arm chromosome 1=
The correct answer is a
KKKKKKKKKKKKKKKKKKKKKKKKKKKKKKKKKKKKKKK
KKKKKKKKKKKKKKK.....
C=2) %egarding heatocellular carcinoma (Featoma) 7hich is true:
a. &ore common in females
b. ..
c. ..
d. !he most common cancer in 'frica and 'sia
e. Increase risk in chronic liver disease
The correct answer is e * chronic disease especiall" /epatitis "
KKKKKKKKKKKKKKKKKKKKKKKKKKKKKKKKKKKKKKK
KKKKKKKKKKKKKKK.....
C=5) ' atient is having a 2 year history of low interest in live5 he doesn?t
slee well and can?t find 6oy in life5 7hat is the most likely diagnosis:
a. /ysthymia
b. &a6or deressive disorder
c. "iolar disorder
d. ..
e. ..
The correct answer is a
C=:) 7hat is the mechanism of (+/ :
a. Increase availability of Serotonin
b. /ecrease roduction of Serotonin
c. Increase roduction of Serotonin
d. .. Serotonin
e. .. Serotonin
The correct anser is a
KKKKKKKKKKKKKKKKKKKKKKKKKKKKKKKKKKKKKKK
KKKKKKKKKKKKKKK.....
C==) 7hat is the &(S! accurate test to detect early regnancy:
a. ..Serum "F+E
b. ..
c. *rine regnancy test
d. *ltrasound
e. ..
The correct answer is a
KKKKKKKKKKKKKKKKKKKKKKKKKKKKKKKKKKKKKKK
KKKKKKKKKKKKKKK.....
C=>) ' case of a atient with thin cerviD and little amount of cervical
mucus5 how would you treat her: OOOOO
a. 4strogen in6ections

KKKKKKKKKKKKKKKKKKKKKKKKKKKKKKK
KKKKKKKKKKKKKKKKKKKKKKK.....
C=C) !he most common cause on chronic interruted rectal bleeding is:
a. /iverticulosis
b. Femorrhoids
The correct answer is b
KKKKKKKKKKKKKKKKKKKKKKKKKKKKKKKKKKKKKKK
KKKKKKKKKKKKKKK.....
C>3) ' child is having a crou early morning5 the most common cause is:
a. -ost nasal dri
C>1) ' burn atient is treated with Silver SulfadiaGine5 the toDicity of this
drug can cause:
a. Bycosytosis
b. ,eutroenia
c. 4lectrolyte disbalance
d. Fyokalemia
e. ..
The correct answer is b * side effect of sulfadia7ine
KKKKKKKKKKKKKKKKKKKKKKKKKKKKKKKKKKKKKKK
KKKKKKKKKKKKKKK.....
C>2) ' case scenario about a atient who had aendectomy5 after that
he has abdominal ain and constiation and absent bowel sound5 the
most likely cause is:
a. Ilus araticus
2 is not complete
KKKKKKKKKKKKKKKKKKKKKKKKKKKKKKKKKKKKKKK
KKKKKKKKKKKKKKK.....

C>)) ' case scenario describing a atient who had a right chest trauma
than develoed tension neumothoraD5 the immediate management is:
a. Insert a needle in the second intercostal sace
b. Insert a needle in the fifth intercostal sace
c. Eive I@ antibiotics
d. ..
e. Insert a tube in the fifth intercostal sace
The correct answer is a
KKKKKKKKKKKKKKKKKKKKKKKKKKKKKKKKKKKKKKK
KKKKKKKKKKKKKKK.....
C>2) ' atient that have a enile ulcer that healed after that he develoed
a alm and sole rash5 the most likely cause is:
a. Syhilis
2 is not complete
C>5) ' case scenario about a atient who has on and off eisodes of
abdominal ain and was found to have multile gallstones5 the largest
is 1 cm and they are not blocking the duct5 7hat will you do:
a. Eive ain killers medication
b. %emove gallbladder by surgery
the correct answer is b

KKKKKKKKKKKKKKKKKKKKKKKKKKKKKKK
KKKKKKKKKKKKKKKKKKKKKKK.....
C>:) &ost common symtoms of soft tissue sarcoma :
a. -aralysis
b. (n growing mass
c. -ain
d. ..
The correct answer is b
KKKKKKKKKKKKKKKKKKKKKKKKKKKKKKKKKKKKKKK
KKKKKKKKKKKKKKK.....
C>=) -atient use illegal drug abuse and the blood show %,' virus. 7hich
heatitis
a. '
b. " (/,')
c. +
d. 4
The correct answer is c * RN% virus transmitted b" blood id /+V
KKKKKKKKKKKKKKKKKKKKKKKKKKKKKKKKKKKKKKK
KKKKKKKKKKKKKKK.....
C>>) ' case scenario about bronchial carcinoma5 which is true: OOOO
a. !he most common cancer in females
b. STuamous cell carcinoma sreads faster
c. 'denocarcinoma is usually in the uer art
d. 4levation of the diahragm on the D#ray means that the carcinoma
has metastasiGe outside the chest
e. "ronchoscoy should be done
The correct answer is c
C>C) old female with ubic itching with bloody discharge5 then she
develoed ea shaed swelling in her labia5 most likely:
a. "artholin cyst
b. "artholin gland carcinoma
c. "artholin abscess
d. ..
The correct answer is c
KKKKKKKKKKKKKKKKKKKKKKKKKKKKKKKKKKKKKKK
KKKKKKKKKKKKKKK.....
CC3) -atient has decrease visual acuity bilateral 5 but more in rt side 5
visual field is not affected 5 in fundus there is irregular igmentations
and early cataract formation . what you will do
a. %efer to ohthalmologist for laser theray
b. %efer to ohthalmologist for cataract surgery
c. ..
The correct answer is a
KKKKKKKKKKKKKKKKKKKKKKKKKKKKKKKKKKKKKKK
KKKKKKKKKKKKKKK.....
CC1) Inflammatory bowel disease is idioathic but one of following is
ossible underlying cause:
a. Immunological

KKKKKKKKKKKKKKKKKKKKKKKKKKKKKKK
KKKKKKKKKKKKKKKKKKKKKKK.....
CC2) regant lady deliverd 'nencehalytillbirth occurance of neural tube
defect in neDt regnancy
a. >9
b. 29
c. 139
d. 239
!he correct answer is b 5 ( 2 K 2 9 )
$$$$$$$$$$$$$$$$$$$$$$$$$$$$$$$$$$$$$$$
$$$$$$$$$$$$$$$$
CC)) tyist theaner atrohy 5.ve tehlen sign
a. &edian nerve
b. *lnar
c. "rachial
!he correct answer is a
CC2) ainful ile
a. 4Dison daringe
b. SitG ath and steriod su
c. 'b
d. 8iber food and analgesics
!he correct answer is a
$$$$$$$$$$$$$$$$$$$$$$$$$$$$$$$$$$$$$$$
$$$$$$$$$$$$$$$$
CC5) )C years old FI@ atient with !" recive 2 drugs of tt after one month
a. +ontinue 2 drugs for 1 years
b. +intinue isoniaGide for Cmonths
c. +ontiue isoniGide for 1 year
!he correct answer is a
$$$$$$$$$$$$$$$$$$$$$$$$$$$$$$$$$$$$$$$
$$$$$$$$$$$$$$$$
CC:) femaIe not married with normaI investigation except FBS=142.
RBS196. so ttt
a. give insIin sctaneos
. advice not ecome married
c. arrier contraceptive is good
d. B controI
!he correct answer is d

$$$$$$$$$$$$$$$$$$$$$$$$$$$$$$$$$$$$$$$
$$$$$$$$$$$$$$$$
CC=) 4ntamoeba histolytica cysts are destroyed best by:
a. "oiling
b. Iodine added to water
c. +hlorine added to water
d. 8reeGing
The correct answer is a
CC>)' scenario about an old male with symtoms suggesting arkinsonism
such as difficulty walking5 resting tremors and rigidity in addition to
hyotension. !hen he asks about what is the most common resenting
symtom of this disease
a. %igidity
b. !remors
c. *nsteady Eait
d. Fyotension
4. OOO
The correct answer is b
KKKKKKKKKKKKKKKKKKKKKKKKKKKKKKKKKKKKKKK
KKKKKKKKKKKKKKKKK..
CC>) %egarding ostartum -sychosis:
a. %ecurrences are common in subseTuent regnancies
b. It often rogresses to frank schiGohrenia
c. It has good rognosis
d. It has insidious onset
e. It usually develos around the )
rd
week ostartum
The correct answer is a
KKKKKKKKKKKKKKKKKKKKKKKKKKKKKKKKKKKKKKK
KKKKKKKKKKKKKKKKK..
CCC) Eastrictomy ost#o 1 day. Fe have temerature )>.> J ulse 112.
7hat is the most common cause O
a. wound infection.
b. inflammatory mediator in the circulation.
c. *!I
d. normal
the correct answer is a or b
most common causes of post-operation fever ! ( P w )
0- E ound
1- Eine V pneumonia or atlectasia
I- Eater V BTI
R- Ehat happen V transfusion
Q- Eal# V .VT
P- Eonder dru V asprin*er"throm"cin * isoni7ide
Immediatel" ( 1R h ) ! post-op inflammation * wound infection * transfusion *
streptococcus
%cute ( I - D da"s ) ! pneumonia

1333) &alaria O
# the most common cases is caused by -lasmodium falciarum.
KKKKKKKKKKKKKKKKKKKKKKKKKKKKKKKKKKKKKK
KKKKKKKKKKKKKKKKKK..
1331 ) -olyartheralgia rhumatica. 7hat is the thing that suggest it rather
than 4S% J +#reactive rotein:
a. roDimal muscle weakness
b. roDimal muscle tenderness
the correct answer is a
KKKKKKKKKKKKKKKKKKKKKKKKKKKKKKKKKKKKKKK
KKKKKKKKKKKKKKKKK..
1332) !he cause of insulin resistance in obese is:
a. insulin recetors kinase activity
b. number of insulin recetor
c. circulation of anti#insulin
d. insulin roduction from the ancreas
e. ost#recetor action
the correct answer is e

133)) -t came with total bilirubin: 5.> J direct bilirubin: 3.2
a. EuillainK"arr^ syndrome
b. /ubin#6unson syndrome
c. "iliary sclerosis
d. +rigler#,a66ar syndrome
e. Eilbert Syndrome
the correct answer is e
KKKKKKKKKKKKKKKKKKKKKKKKKKKKKKKKKKKKKKK
KKKKKKKKKKKKKKKKK
1332) -t comlaining of hyotension J bradycardia. 4lectrolytes show:
,a5 N5 +l5 *rea. So the cause of this is:
a. hyonatremia
b. hyerkalemia
c. hyerchloremia
d. uremia
the correct answer is a

:
Orthopaedic Qassim University

You might also like